Reproductive

Pataasin ang iyong marka sa homework at exams ngayon gamit ang Quizwiz!

The nurse is teaching a client with benign prostatic hyperplasia (BPH) about ways to avoid loss of bladder muscle tone. What instructions does the nurse provide? Select all that apply.

"Empty your bladder as soon as you feel the urge." "Avoid drinking large amounts of fluids in a short amount of time." "Avoid the intake of alcohol and caffeine."

A client has had a right modified radical mastectomy and axillary lymph node dissection. The nurse is teaching the client about measures to reduce the risk of complications. The client demonstrates understanding of the instructions when she states which of the following? a) "I need to use an electric shaver when shaving my right armpit." b) "I can lift with my right arm objects that weigh as much as 15 pounds." c) "I should tell my manucurist that it is okay to trim the cuticles on my right hand." d) "Anytime I need blood drawn, they should get the sample from my right arm."

"I need to use an electric shaver when shaving my right armpit." To prevent complications after a right modified radical mastectomy and axillary lymph node dissection, the client should shave her right axillary area with an electric razor rather than a straight razor to reduce the risk of trauma to or breaks in the skin. Blood sampling should be done with the client's left arm, not the right one. The client should not lift objects more than 5 to 10 pounds with the right arm, and cuticles on the affected hand should be pushed back, not cut.

The nurse is educating a group of young men about testicular self-examination (TSE). Which statement by a member of the group indicates teaching has been effective?

"I will look and feel for any lumps or changes to my testes."

The nurse is planning the discharge of a client who had a radical prostectomy. What instruction does the nurse provide to the client about caring for the catheter at home?

"Replace the old drainage bag with a new one at least once a week."

The nurse is explaining testicular self-examination to a client. What does the nurse explain about the examination?

"Roll and feel each testicle between your thumb and fingers."

The nurse is teaching a client how to perform a testicular self-examination. What does the nurse instruct the client to do?

"Roll and feel each testicle between your thumb and fingers."

A client with prostate cancer asks why he must have surgery instead of radiation, even if his cancer is the least-invasive type. What is the nurse's best response?

"Surgery is the most common intervention to cure the disease."

A nurse is meeting with a woman scheduled to have a modified radical mastectomy to remove an aggressive breast tumor. The woman tells the nurse that she agreed to have the surgery before considering alternative options. Which of the following statements is the nurse's best response? a) "If I were you, I would consider a second opinion." b) "You might want to consider a less invasive surgical procedure." c) "You have a very competent surgeon and you should move forward as planned." d) "Tell me more about your fears and concerns."

"Tell me more about your fears and concerns." The type of surgery recommended depends on the stage of the tumor and the client's informed decision about treatment options. The client should be encouraged to express her concerns. Surgery should not be performed until the client is comfortable with the scheduled procedure. The type of surgery recommended depends on the stage of the tumor. A less invasive procedure may not remove all of the affected tissue. The nurse should not share her opinion with the client but rather support the client in making the best decision.

The nursing instructor is teaching a group of nursing students how to prepare a client with benign prostatic hyperplasia (BPH) for a digital rectal examination (DRE). What statement by a student nurse indicates a need for further teaching?

"The residual urine will be measured during the examination."

What are risk factors for breast cancer?

#1-female, #2-obesity, #3-age. others-personal and family history including genetic mutations, hormonal factors, exposure to radiation, history of benign breast disease, high fat dies (lead to obesity), alcohol intake

A 16-year-old male is to receive testosterone cypionate (Depo-Testosterone), 50 mg IM every 2 weeks. The medication is available in 100-mg/mL containers. How many mL will the nurse draw up in the syringe to administer for each dose?

0.5mL per dose

53. The nurse should instruct the client who is being discharged to home 3 days after transurethral resection of the prostate (TURP) to do which of the following? Select all that apply. 1. Drink at least 3,000 mL water per day. 2. Increase calorie intake by eating six small meals a day. 3. Report bright red bleeding to the health care provider. 4. Take deep breaths and cough every 2 hours. 5. Report a temperature over 99°F (37.2°C).

1, 3, 5. The nurse should instruct the client to drink a large amount of fuids (about 3,000 mL/day) to keep the urine clear. The urine should be almost without color. About 2 weeks after TURP, when desiccated tissue is sloughed out, a secondary hemorrhage could occur. The client should be instructed to call the surgeon or go to the emergency department if at any time the urine turns bright red. The nurse should also instruct the client to report signs of infection such as a temperature over 99°F (37.2°C). The client is not specifically at risk for nutritional problems after TURP and can resume a diet as tolerated. The client is not specifically at risk for airway problems because the procedure is done under spinal anesthesia and the client does not need to take deep breaths and cough.

38. An adult male client has been unable to void for the past 12 hours. The best method for the nurse to use when assessing for bladder distention in a male client is to check for: 1. A rounded swelling above the pubis. 2. Dullness in the lower left quadrant. 3. Rebound tenderness below the symphysis. 4. Urine discharge from the urethral meatus.

1. A rounded swelling above the pubis. The best way to assess for a distended bladder in either a male or female client is to check for a rounded swelling above the pubis. This swelling represents the distended bladder rising above the pubis into the abdominal cavity. Dullness does not indicate a distended bladder. The client might experience tenderness or pressure above the symphysis. No urine discharge is expected; the urine flow is blocked by the enlarged prostate. C

116. The nurse should teach the client with erectile dysfunction (ED) to alter his lifestyle to: 1. Avoid alcohol. 2. Follow a low-salt diet. 3. Decrease smoking. 4. Increase attempts at sexual intercourse.

1. Avoidance of alcohol can improve the outcome of therapy. Alcohol and smoking can affect a man's ability to have and maintain an erection. The client should be encouraged to follow a healthy diet, but no specific diet is associated with improvement of sexual function. The client should cease smoking, not just decrease smoking.

41. The primary reason for taping an indwelling catheter laterally to the thigh of a male client is to: 1. Eliminate pressure at the penoscrotal angle. 2. Prevent the catheter from kinking in the urethra. 3. Prevent accidental catheter removal. 4. Allow the client to turn without kinking the catheter.

1. Eliminate pressure at the penoscrotal angle. The primary reason for taping an indwelling catheter to a male client so that the penis is held in a lateral position is to prevent pressure at the penoscrotal angle. Prolonged pressure at the penoscrotal angle can cause a ureterocutaneous fistula.

43. When caring for a client with a history of benign prostatic hypertrophy (BPH), the nurse should do which of the following? Select all that apply. 1. Provide privacy and time for the client to void. 2. Monitor intake and output. 3. Catheterize the client for postvoid residual urine. 4. Ask the client if he has urinary retention. 5. Test the urine for hematuria.

1. Provide privacy and time for the client to void. 2. Monitor intake and output. 4. Ask the client if he has urinary retention. 5. Test the urine for hematuria. Because of the history of BPH, the nurse should provide privacy and time for the client to void. The nurse should also monitor intake and output, assess the client for urinary retention, and test the urine for hematuria. It is not necessary to catheterize the client.

How many lymph nodes may be taken out with breast cancer?

10-13 of the surrounding lymph or nodes or just a few (depends on what the physician sees when he gets in there and what labs show)

In which stage of breast development does breast budding occur? a) 3 b) 2 c) 1 d) 4

2 Correct Explanation: Breast budding occurs in Stage 2 of breast development. Stage 1 describes a prepubertal breast. Stage 3 involves further enlargement of breast tissue and the areola. Stage 4 occurs when the nipple and areola form a secondary mound on top of the breast tissue

51. A physician has prescribed amoxicillin (Ampicillin) 100 PO b.i.d. The nurse should teach the client to do which of the following? Select all that apply. 1. Drink 300 to 500 mL of fuids daily. 2. Void frequently, at least every 2 to 3 hours. 3. Take time to empty the bladder completely. 4. Take the last dose of the antibiotic for the day at bedtime. 5. Take the antibiotic with or without food.

2, 3, 4, 5. Ampicillin may be given with or without food, but the nurse should instruct the client to obtain an adequate fuid intake (2,500 to 3,000 mL) to promote urinary output and to flush out bacteria from the urinary tract. The nurse should also encourage the client to void frequently (every 2 to 3 hours) and empty the bladder completely. Taking the antibiotic at bedtime, after emptying the bladder, helps to ensure an adequate concentration of the drug during the overnight period.

114. A male client reports having impotence. The nurse examines the client's medication regimen and is aware that a contributing factor to impotence could be: 1. Aspirin. 2. Antihypertensives. 3. Nonsteroidal antiinfammatory drugs. 4. Anticoagulants.

2. Antihypertensives. Antihypertensives, especially beta-blockers such as propranolol (Inderal), can cause impotence. When a male client has impotence, the nurse should always examine his medication regimen as a potential contributing factor. Aspirin, nonsteroidal anti-infammatory drugs, and anticoagulants do not cause erectile dysfunction.

44. The nurse should specifically assess a client with prostatic hypertrophy for which of the following? 1. Voiding at less frequent intervals. 2. Difficulty starting the flow of urine. 3. Painful urination. 4. Increased force of the urine stream.

2. Difficulty starting the flow of urine. Signs and symptoms of prostatic hypertrophy include diffculty starting the flow of urine, urinary frequency and hesitancy, decreased force of the urine stream, interruptions in the urine stream when voiding, and nocturia. The prostate gland surrounds the urethra, and these symptoms are all attributed to obstruction of the urethra resulting from prostatic hypertrophy. Nocturia from incomplete empty-ing of the bladder is common. Straining and urine retention are usually the symptoms that prompt the client to seek care. Painful urination is generally not a symptom of prostatic hypertrophy.

52. In discussing home care with a client after transurethral resection of the prostate (TURP), the nurse should teach the male client that dribbling of urine: 1. Can be a chronic problem. 2. Can persist for several months. 3. Is an abnormal sign that requires intervention. 4. Is a sign of healing within the prostate.

2. Dribbling of urine can occur for several months after TURP. The client should be informed that this is expected and is not an abnormal sign. The nurse should teach the client perineal exercises to strengthen sphincter tone. The client may need to use pads for temporary incontinence. The client should be reassured that continence will return in a few months and will not be a chronic problem. Dribbling is not a sign of healing, but is related to the trauma of surgery.

42. Many older men with prostatic hypertrophy do not seek medical attention until urinary obstruction is almost complete. One reason for this delay in seeking attention is that these men may: 1. Feel too self-conscious to seek help when reproductive organs are involved. 2. Expect that it is normal to have to live with some urinary problems as they grow older. 3. Fear that sexual indiscretions in earlier life may be the cause of their problem. 4. Have little discomfort in relation to the amount of pathology because responses to pain stimuli fade with age.

2. Expect that it is normal to have to live with some urinary problems as they grow older. Some older men tend to believe it is normal to live with urinary problems. As a result, these men often overlook symptoms and simply attribute them to aging. As part of preventive care for men older than age 40, the yearly physical examination should include palpation of the prostate via rectal exami-nation. Prostate-specific antigen screening also is done annually to determine elevations or increasing trends in elevations. The nurse should teach male clients the value of early detection and adequate follow-up for the prostate.

40. The primary reason for lubricating the urinary catheter generously before inserting the catheter into a male client is that this technique helps reduce: 1. Spasms at the orifice of the bladder. 2. Friction along the urethra when the catheter is being inserted. 3. The number of organisms gaining entrance to the bladder. 4. The formation of encrustations that may occur at the end of the catheter.

2. Friction along the urethra when the catheter is being inserted. Liberal lubrication of the catheter before catheterization of a male reduces friction along the urethra and irritation and trauma to urethral tissues. Because the male urethra is tortuous, a liberal amount of lubrication is advised to ease catheter passage. The female urethra is not tortuous, and, although the catheter should be lubricated before insertion, less lubricant is necessary. Lubrication of the catheter will not decrease spasms. The nurse should use sterile technique to prevent introducing organisms. Crusts will not form immediately. Irrigating the catheter as needed will prevent clot and crust formation.

115. A 65-year-old male client with erectile dysfunction (ED) asks the nurse, "Is all this just in my head? Am I crazy?" The best response by the nurse is based on the knowledge that: 1. ED is believed to be psychogenic in most cases. 2. More than 50% of the cases are attributed to organic causes. 3. Evaluation of nocturnal erections does not help differentiate psychogenic or organic causes. 4. ED is an uncommon problem among men older than age 65.

2. More than 50% of the cases are attributed to organic causes. ED is multifactorial in origin, and more than 50% of the cases can be attributed to organic causes, which include alteration in vascular supply, hormonal changes, neurologic dysfunction, medications, and associated systemic diseases, such as diabetes mellitus or alcoholism. The presence of nocturnal erections is the first evaluation to differentiate between organic and psychogenic causes. ED is a common problem among men older than age 65.

113. The client is taking sildenafil (Viagra) PO for erectile dysfunction. The nurse should instruct the client about which of the following? 1. Sildenafl (Viagra) may be taken more than one time per day. 2. The health care provider should be notified promptly if the client experiences sudden or diminished vision. 3. Sildenafil (Viagra) offers protection against some sexually transmitted diseases (STDs). 4. Sildenafol (Viagra) does not require sexual stimulation to work.

2. The health care provider should be notified promptly if the client experiences sudden or diminished vision. The client should notify his health care provider promptly if he experiences sudden or decreased vision loss in one or both eyes. Sildenafil (Viagra) should NOT be taken more than once per day. Viagra offers no protection against sexually transmitted diseases. Viagra has no effect in the absence of sexual stimulation.

21. A postmenopausal woman is worried about pain in the upper outer quadrant of her left breast. The nurse's first course of action is to: 1. Do a breast examination and report the results to the physician. 2. Explain that pain is caused by hormonal fluctuations. 3. Reassure the client that pain is not a symptom of breast cancer. 4. Teach the client the correct procedure for breast self-examination (BSE).

21. 1. This information warrants the nurse's performing an examination and reporting the results to the physician. Hormone fluctuations do cause breast discomfort, but an examination must be done at this time to assess the breast. Although pain is not common with breast cancer, it can be a symptom. Teaching the client to perform a breast exam is important, but it is not the priority action in this case.

22. The nurse teaches a female client that the best time in the menstrual cycle to examine the breasts is during the: 1. Week that ovulation occurs. 2. Week that menstruation occurs. 3. First week after menstruation. 4. Week before menstruation occurs.

22. 3 . It is generally recommended that the breasts be examined during the first week after menstruation. During this time, the breasts are least likely to be tender or swollen because estrogen is at its lowest level. Therefore, the examination will be more comfortable for the client. The examination may also be more accurate because the client is more likely to notice an actual change in her breast that is not simply related to hormonal changes.

23. A female with bilateral breast implants asks if she still needs to do breast examinations because she does not know what to feel for. Which of the following is the nurse's best response? 1. "Have your partner assess your breasts on a regular basis." 2. "I will show you the correct technique as I do the breast examination." 3. "A breast examination is very difficult when you have had implant surgery." 4. "You need to have a mammogram instead."

23. 2. The client needs to become more confident and knowledgeable about the normal feel of the implants and her breast tissue. The best technique is for the nurse to demonstrate breast self-examination (BSE) to the client as the nurse conducts the clinical breast examination. Implant surgery does not exclude the need for monthly BSE. A mammogram is not a substitute for monthly BSE.

24. The client states that she has noticed that her bra fits more snugly at certain times of the month. She asks the nurse if this is a sign of breast disease. The nurse should base the reply to this client on the knowledge that: 1. Benign cysts tend to cause the breasts to vary in size. 2. It is normal for the breasts to increase in size before menstruation begins. 3. A change in breast size warrants further investigation. 4. Differences in breast size are related to nor mal growth and development.

24. 2. The breasts may vary in size before menstruation because of breast engorgement caused by hormonal changes. A woman may then note that her bra fits more tightly than usual. Benign cysts do not cause variation in breast size. A change in breast size that does not follow hormonal changes could warrant further assessment. The breasts normally are about the same size, although some women have one breast slightly larger than the other.

What is the average percent of women who perform breast self-exams?

25%-not good because breast cancer can be successfully treated if caught early

25. A 76-year-old client tells the nurse that she has lived long and does not need mammograms. Which is the nurse's best response? 1. "Having a mammogram when you are older is less painful." 2. "The incidence of breast cancer increases with age." 3. "We need to consider your family history of breast cancer first." 4. "It will be sufficient if you perform breast examinations monthly."

25. 2. Advancing age in postmenopausal women has been identified as a risk factor for breast cancer. A 76-year-old client needs monthly breast self- examination and a yearly clinical breast examination and mammogram to comply with the screening schedule. While mammograms are less painful as breast tissue becomes softer, the nurse should advise the woman to have the mammogram. Family history is important, but only about 5% of breast cancers are genetic.

26. After the surgeon met with a client to obtain the client's informed consent for a modified radical mastectomy, the client asks the nurse many questions about breast reconstruction that the nurse cannot to answer. The nurse should: 1. Inform the surgeon that the client has questions about reconstruction before she signs the consent. 2. Inform the client that she should concentrate on recovering from the mastectomy first. 3. Inform the client that she can have a consultation with the plastic surgeon in a few weeks. 4. Inform the client she can ask the surgeon these questions later when the surgeon makes rounds.

26. 1. If a client has questions the nurse cannot answer, it is best to delay the signing of the consent until the questions are clarified for the client. The surgeon should be notified, and the appropriate information or collaboration should be provided for the client before she signs the surgical consent. Telling her she should concentrate on recovery first ignores the client's questions and concerns. Frequently the plastic surgeon needs to be consulted at the beginning of the treatment because various surgical decisions depend on the future plans for breast reconstruction.

39. When emptying the client's bladder during a urinary catheterization, the nurse should allow the urine to drain from the bladder slowly to prevent: 1. Renal failure. 2. Abdominal cramping. 3. Possible shock. 4. Atrophy of bladder musculature.

3. Possible shock. 3. Rapid emptying of an overdistended bladder may cause hypotension and shock due to the sudden change of pressure within the abdominal viscera. The nurse should empty the bladder slowly. Removal of urine from the bladder does not cause renal failure. The client may experience cramping, but the primary concern is the potential for shock. Bladder muscles will not atrophy because of a catheterization.

27. Prior to surgery for a modified radical mastectomy, the client is extremely anxious and asks many questions. Which of the following approaches would offer the best guide for the nurse to answer these questions. 1. Tell the client as much as she wants to know and is able to understand. 2. Delay discussing the client's questions with her until she is convalescing. 3. Delay discussing the client's questions with her until her apprehension subsides. 4. Explain to the client that she should discuss her questions first with the physician.

27. 1. An important nursing responsibility is preoperative teaching, and the most frequently recommended guide for teaching is to tell the client as much as she wants to know and is able to understand. Delaying discussion of issues about which the client has concerns is likely to aggravate the situation and cause the client to feel distrust. As a general guide, the client would not ask the question if she were not ready to discuss her situation. The nurse is available to answer the client's questions and concerns and should not delay discussing these with the client.

28. Atropine sulfate is included in the preoperative prescriptions for a client undergoing a modified radical mastectomy. The expected outcome is to: 1. promote general muscular relaxation. 2. decrease pulse and respiratory rates. 3. decrease nausea. 4. inhibit oral and respiratory secretions.

28. 4. Atropine sulfate, a cholinergic blocking agent, is given preoperatively to reduce secretions in the mouth and respiratory tract, which assists in maintaining the integrity of the respiratory system during general anesthesia. Atropine is not used to promote muscle relaxation, decrease nausea and vomiting, or decrease pulse and respiratory rates. It causes the pulse to increase.

29. During the postoperative period after a modified radical mastectomy, the client confides in the nurse that she thinks she got breast cancer because she had an abortion and she did not tell her husband. The best response by the nurse is which of the following? 1. "Cancer is not a punishment; it is a disease." 2. "You might feel better if you confided in your husband." 3. "Tell me more about your feelings about this." 4. "I can have the social worker talk to you if you would like."

29. 3. The nurse should respond with an open- ended statement that elicits further exploration of the client's feelings. Women with cancer may feel guilt or shame. Previous life decisions, sexuality, and religious beliefs may influence a client's adjustment to a diagnosis of cancer. The nurse should not contradict the client's feelings of punishment or offer advice such as confiding in the husband. A social worker referral may be beneficial in the future, but is not the first response needed to elicit exploration of the client's feelings.

According to the American Cancer Society, a woman in her 20s or 30s with no symptoms or family history of breast cancer should have a clinical breast examination once every ____ years.

3

47. A client with benign prostatic hypertrophy (BPH) is being treated with terazosin (Hytrin) 2 mg at bedtime. The nurse should monitor the client's: 1. Urine nitrites. 2. White blood cell count. 3. Blood pressure. 4. Pulse.

3. Blood pressure. Terazosin is an antihypertensive drug that is also used in the treatment of BPH. Blood pressure must be monitored to ensure that the client does not develop hypotension, syncope, or orthostatic hypotension. The client should be instructed to change positions slowly. Urine nitrates, white blood cell count, and pulse rate are not affected by terazosin.

45. The nurse is reviewing the medication history of a client with benign prostatic hypertrophy (BPH). Which medication will likely aggravate BPH? 1. Metformin (Glucophage). 2. Buspirone (BuSpar). 3. Inhaled ipratropium (Atrovent). 4. Ophthalmic timolol (Timoptic).

3. Ipratropium is a bronchodilator, and its anticholinergic effects can aggravate urine retention. Metformin and buspirone do not affect the urinary system; timolol does not have a systemic effect.

54. A client with benign prostatic hypertrophy (BPH) has an elevated prostate-specifc antigen (PSA) level. The nurse should: 1. Instruct the client to request having a colonoscopy before coming to conclusions about the PSA results. 2. Instruct the client that a urologist will monitor the PSA level biannually when elevated. 3. Determine if the prostatic palpation was done before or after the blood sample was drawn. 4. Ask the client if he emptied his bladder before the blood sample was obtained.

3. Rectal and prostate examinations can increase serum PSA levels; therefore, instruct the client that a manual rectal examination is usually part of the test regimen to determine prostate changes. The prostatic palpation should be done after the blood sample is drawn. The PSA level must be monitored more often than biannually when it is elevated. Having a colonoscopy is not related to the findings of the PSA test. It is not necessary to void prior to having PSA blood levels tested.

48. A client, who had a transurethral resection of the prostate (TURP), has a three-way indwelling urinary catheter with continuous bladder irrigation. In which of the following circumstances should the nurse increase the flow rate of the continuous bladder irrigation? 1. When drainage is continuous but slow. 2. When drainage appears cloudy and dark yellow. 3. When drainage becomes bright red. 4. When there is no drainage of urine and irrigating solution.

3. The decision by the surgeon to insert a catheter after TURP or prostatectomy depends on the amount of bleeding that is expected after the procedure. During continuous bladder irrigation after a TURP or prostatectomy, the rate at which the solution enters the bladder should be increased when the drainage becomes brighter red. The color indicates the presence of blood. Increasing the fow of irrigating solution helps flush the catheter well so that clots do not plug it. There would be no reason to increase the flow rate when the return is continuous or when the return appears cloudy and dark yellow. Increasing the flow would be contraindi-cated when there is no return of urine and irrigating solution.

50. A nursing assistant tells the nurse, "I think the client is confused. He keeps telling me he has to void, but that isn't possible because he has a catheter in place that is draining well." Which of the following responses would be most appropriate for the nurse to make? 1. "His catheter is probably plugged. I'll irrigate it." 2. "That's a common problem after prostate surgery. The client only imagines the urge to void." 3. "The urge to void is usually created by the large catheter, and he may be having some bladder spasms." 4. "I think he may be somewhat confused."

3. The indwelling urinary catheter creates the urge to void and can also cause bladder spasms. The nurse should ensure adequate bladder emptying by monitoring urine output and characteristics. Urine output should be at least 50 mL/h. A plugged catheter, imagining the urge to void, and confusion are less likely reasons for the client's problem.

30. Following a modified radical mastectomy, a client has an incisional drainage tube attached to Hemovac suction. The nurse determines the suction is effective when: 1. The intrathoracic pressure is decreased, and the client breathes easier. 2. There is an increased collateral lymphatic flow toward the operative area. 3. Accumulated serum and blood in the operative area are removed. 4. No adhesions are formed between the skin and chest wall in the operative area.

30. 3. A drainage tube is placed in the wound after a modified radical mastectomy to help remove accumulated blood and fluid in the area. Removal of the drainage fluids assists in wound healing and is intended to decrease the incidence of hematoma, abscess formation, and infection. Drainage tubes placed in a wound do not decrease intrathoracic pressure, increase collateral lymphatic flow, or prevent adhesion formation.

31. Which of the following positions would be best for a client's right arm when she returns to her room after a right modified radical mastectomy with multiple lymph node excisions? 1. Across her chest wall. 2. At her side at the same level as her body. 3. In the position that affords her the greatest comfort without placing pressure on the incision. 4. On pillows, with her hand higher than her elbow and her elbow higher than her shoulder.

31. 4. Lymph nodes can be removed from the axillary area when a modified radical mastectomy is done, and each of the nodes is biopsied. To facilitate drainage from the arm on the affected side, the client's arm should be elevated on pillows with her hand higher than her elbow and her elbow higher than her shoulder. A sentinel node biopsy procedure is associated with a decreased risk of lymphedema because fewer nodes are excised.

32. A client develops lymphedema after a left mastectomy with lymph node dissection. Which of the following should be included in the discharge teaching plan? Select all that apply. 1. Do not allow blood pressures or blood draws in the affected arm. 2. Avoid application of sunscreen on the left arm. 3. Use an electric razor for shaving. 4. Immobilize the left arm. 5. Elevate the left arm. 6. Perform hand pump exercises.

32. 1, 3, 5, 6. Blood pressures or blood draws in the affected arm, sun exposure, trauma with a sharp razor, and immobilization increase the risk of lymphedema. Elevation of the arm and hand pump exercises promote lymph flow and reduce edema.

33. The client with breast cancer is prescribed tamoxifen (Nolvadex) 20 mg daily. The client states she does not like taking medicine and asks the nurse if the tamoxifen is really worth taking. The nurse's best response is which of the following? 1. "This drug is part of your chemotherapy program." 2. "This drug has been found to decrease metastatic breast cancer." 3. "This drug will act as an estrogen in your breast tissue." 4. "This drug will prevent hot flashes since you cannot take hormone replacement."

33. 2. Tamoxifen is an antiestrogen drug that has been found to be effective against metastatic breast cancer and to improve the survival rate. The drug causes hot flashes as an adverse effect.

34. A client undergoing chemotherapy after a modified radical mastectomy asks the nurse questions about breast prosthesis and wigs. After answering the questions directly, the nurse should also: 1. Provide a list of resources, including the local breast cancer support group. 2. Offer a referral to the social worker. 3. Call the home health care agency. 4. Contact the plastic surgeon.

34. 1. Giving the client a list of community resources that could provide support and guidance assists the client to maintain her self-image and independence. The support group will include other women who have undergone similar therapies and can offer suggestions for breast products and wigs. Because the client is asking about specific resources, she does not need a referral to a social worker, home health agency, or plastic surgeon.

35. A client is to have radiation therapy after a modified radical mastectomy. The nurse should teach the client to care for the skin at the site of therapy by: 1. Washing the area with water. 2. Exposing the area to dry heat. 3. Applying an ointment to the area. 4. Using talcum powder on the area.

35. 1. A client receiving radiation therapy should avoid lotions, ointments, and anything that may cause irritation to the skin, such as exposure to sunlight, heat, or talcum powder. The area may safely be washed with water if it is done gently and if care is taken not to injure the skin.

36. The nurse should teach a client that a normal local tissue response to radiation following surgery for breast cancer is: 1. Atrophy of the skin. 2. Scattered pustule formation. 3. Redness of the surface tissue. 4. Sloughing of two layers of skin.

36. 3. The most common reaction of the skin to radiation therapy is redness of the surface tissues. Dryness, tanning, and capillary dilation are also common. Atrophy of the skin, pustules, and sloughing of two layers would not be expected and should be reported to the radiologist.

37. The nurse refers a client who had a mastectomy to "Reach to Recovery." The primary purpose of the American Cancer Society's Reach to Recovery program is to: 1. Foster rehabilitation in women who have had mastectomies. 2. Raise funds to support early breast cancer detection programs. 3. Provide free dressings for women who have had radical mastectomies. 4. Collect statistics for research from women who have had mastectomies.

37. 1. The American Cancer Society's Reach to Recovery is a rehabilitation program for women who have had breast surgery. It is designed to meet their physical, psychological, and emotional needs. The Reach to Recovery program is implemented by women who have had breast cancer themselves. Many women benefit from this peer information and support.

Benign Prostatic Hypertrophy 38. An adult male client has been unable to void for the past 12 hours. The best method for the nurse to use when assessing for bladder distention in a male client is to check for: 1. A rounded swelling above the pubis. 2. Dullness in the lower left quadrant. 3. Rebound tenderness below the symphysis. 4. Urine discharge from the urethral meatus.

38. 1. The best way to assess for a distended bladder in either a male or female client is to check for a rounded swelling above the pubis. This swelling represents the distended bladder rising above the pubis into the abdominal cavity. Dullness does not indicate a distended bladder. The client might experience tenderness or pressure above the symphysis. No urine discharge is expected; the urine flow is blocked by the enlarged prostate.

39. When emptying the client's bladder during a urinary catheterization, the nurse should allow the urine to drain from the bladder slowly to prevent: 1. Renal failure. 2. Abdominal cramping. 3. Possible shock. 4. Atrophy of bladder musculature.

39. 3. Rapid emptying of an overdistended bladder may cause hypotension and shock due to the sudden change of pressure within the abdominal viscera. The nurse should empty the bladder slowly. Removal of urine from the bladder does not cause renal failure. The client may experience cramping, but the primary concern is the potential for shock. Bladder muscles will not atrophy because of a catherization.

While performing a clinical breast examination on a client, the nurse inspects the breasts with the client in various positions. Place the following positions in their proper sequence. 1 Arms resting at the sides 2 Arms raised over the head 3 Hands placed on the hips 4 Hands on hips while standing

3: Hands placed on the hips 1: Arms resting at the sides 4: Hands on hips while standing 2: Arms raised over the head During the clinical breast examination, the nurse inspects the breasts first with the client sitting at the edge of the examination table with her arms resting at her sides. Next, the client places her hands on her hips, and then she raises her arms over her head. Lastly, the client stands and places her hands on her hips and leans forward

Atypical hyperplasia increases a women's risk for breast cancer about how many times compared with that of the general population? a) 4 b) 6 c) 8 d) 2

4 Correct Explanation: Atypical hyperplasia increases a women's risk for breast cancer about four to five times compared with that of the general population.

Within what timeframe is collateral circulation (new blood vessels) usually developed in a woman after a mastectomy?

4 weeks. collateral circulation also decreases lymphadema

49. A client is to receive belladonna and opium suppositories, as needed, postoperatively after transurethral resection of the prostate (TURP). The nurse should give the client these drugs when he demonstrates signs of: 1. A urinary tract infection. 2. Urine retention. 3. Frequent urination. 4. Pain from bladder spasms.

4. Pain from bladder spasms. Belladonna and opium suppositories are prescribed and administered to reduce bladder spasms that cause pain after TURP. Bladder spasms frequently accompany urologic procedures. Antispasmodics offer relief by eliminating or reducing spasms. Antimicrobial drugs are used to treat an infection. Belladonna and opium do not relieve urine retention or urinary frequency.

46. A client is scheduled to undergo transurethral resection of the prostate. The procedure is to be done under spinal anesthesia. Postoperatively, the nurse should assess the client for: 1. Seizures. 2. Cardiac arrest. 3. Renal shutdown. 4. Respiratory paralysis.

4. Respiratory paralysis. If paralysis of vasomotor nerves in the upper spinal cord occurs when spinal anesthesia is used, the client is likely to develop respiratory paralysis. Artificial ventilation is required until the effects of the anesthesia subside. Seizures, cardiac arrest, and renal shutdown are not likely results of spinal anesthesia.

44. The nurse should specifically assess a client with prostatic hypertrophy for which of the following? 1. Voiding at less frequent intervals. 2. Difficulty starting the flow of urine. 3. Painful urination. 4. Increased force of the urine stream.

44. 2. Signs and symptoms of prostatic hypertrophy include difficulty starting the flow of urine, urinary frequency and hesitancy, decreased force of the urine stream, interruptions in the urine stream when voiding, and nocturia. The prostate gland surrounds the urethra, and these symptoms are all attributed to obstruction of the urethra resulting from prostatic hypertrophy. Nocturia from incomplete emptying of the bladder is common. Straining and urine retention are usually the symptoms that prompt the client to seek care. Painful urination is generally not a symptom of prostatic hypertrophy.

40. The primary reason for lubricating the urinary catheter generously before inserting the catheter into a male client is that this technique helps reduce: 1. Spasms at the orifice of the bladder. 2. Friction along the urethra when the catheter is being inserted. 3. The number of organisms gaining entrance to the bladder. 4. The formation of encrustations that may occur at the end of the catheter.

40. 2. Liberal lubrication of the catheter before catheterization of a male reduces friction along the urethra and irritation and trauma to urethral tissues. Because the male urethra is tortuous, a liberal amount of lubrication is advised to ease catheter passage. The female urethra is not tortuous, and, although the catheter should be lubricated before insertion, less lubricant is necessary. Lubrication of the catheter will not decrease spasms. The nurse should use sterile technique to prevent introducing organisms. Crusts will not form immediately. Irrigating the catheter as needed will prevent clot and crust formation.

41. The primary reason for taping an indwelling catheter laterally to the thigh of a male client is to: 1. Eliminate pressure at the penoscrotal angle. 2. Prevent the catheter from kinking in the urethra. 3. Prevent accidental catheter removal. 4. Allow the client to turn without kinking the catheter.

41. 1. The primary reason for taping an indwelling catheter to a male client so that the penis is held in a lateral position is to prevent pressure at the penoscrotal angle. Prolonged pressure at the penoscrotal angle can cause a ureterocutaneous fistula.

42. Many older men with prostatic hypertrophy do not seek medical attention until urinary obstruction is almost complete. One reason for this delay in seeking attention is that these men may: 1. Feel too self-conscious to seek help when reproductive organs are involved. 2. Expect that it is normal to have to live with some urinary problems as they grow older. 3. Fear that sexual indiscretions in earlier life may be the cause of their problem. 4. Have little discomfort in relation to the amount of pathology because responses to pain stimuli fade with age.

42. 2. Some older men tend to believe it is normal to live with urinary problems. As a result, these men often overlook symptoms and simply attribute them to aging. As part of preventive care for men older than age 40, the yearly physical examination should include palpation of the prostate via rectal examination. Prostate-specific antigen screening also is done annually to determine elevations or increasing trends in elevations. The nurse should teach male clients the value of early detection and adequate follow-up for the prostate.

43. When caring for a client with a history of benign prostatic hypertrophy (BPH), the nurse should do which of the following? Select all that apply. 1. Provide privacy and time for the client to void. 2. Monitor intake and output. 3. Catheterize the client for postvoid residual urine. 4. Ask the client if he has urinary retention. 5. Test the urine for hematuria.

43. 1, 2, 4, 5. Because of the history of BPH, the nurse should provide privacy and time for the client to void. The nurse should also monitor intake and output, assess the client for urinary retention, and test the urine for hematuria. It is not necessary to catheterize the client.

45. The nurse is reviewing the medication history of a client with benign prostatic hypertrophy (BPH). Which medication will likely aggravate BPH? 1. Metformin (Glucophage). 2. Buspirone (BuSpar). 3. Inhaled ipratropium (Atrovent). 4. Ophthalmic timolol (Timoptic).

45. 3. Ipratropium is a bronchodilator, and its anticholinergic effects can aggravate urine retention. Metformin and buspirone do not affect the urinary system; timolol does not have a systemic effect.

46. A client is scheduled to undergo transurethral resection of the prostate. The procedure is to be done under spinal anesthesia. Postoperatively, the nurse should assess the client for: 1. Seizures. 2. Cardiac arrest. 3. Renal shutdown. 4. Respiratory paralysis.

46. 4. If paralysis of vasomotor nerves in the upper spinal cord occurs when spinal anesthesia is used, the client is likely to develop respiratory paralysis. Artificial ventilation is required until the effects of the anesthesia subside. Seizures, cardiac arrest, and renal shutdown are not likely results of spinal anesthesia.

47. A client with benign prostatic hypertrophy (BPH) is being treated with terazosin (Hytrin) 2 mg at bedtime. The nurse should monitor the client's: 1. Urine nitrites. 2. White blood cell count. 3. Blood pressure. 4. Pulse.

47. 3. Terazosin is an antihypertensive drug that is also used in the treatment of BPH. Blood pressure must be monitored to ensure that the client does not develop hypotension, syncope, or orthostatic hypotension. The client should be instructed to change positions slowly. Urine nitrates, white blood cell count, and pulse rate are not affected by terazosin.

48. A client, who had a transurethral resection of the prostate (TURP), has a three-way indwelling urinary catheter with continuous bladder irrigation. In which of the following circumstances should the nurse increase the flow rate of the continuous bladder irrigation? 1. When drainage is continuous but slow. 2. When drainage appears cloudy and dark yellow. 3. When drainage becomes bright red. 4. When there is no drainage of urine and irrigating solution.

48. 3. The decision by the surgeon to insert a catheter after TURP or prostatectomy depends on the amount of bleeding that is expected after the procedure. During continuous bladder irrigation after a TURP or prostatectomy, the rate at which the solution enters the bladder should be increased when the drainage becomes brighter red. The color indicates the presence of blood. Increasing the flow of irrigating solution helps flush the catheter well so that clots do not plug it. There would be no reason to increase the flow rate when the return is continuous or when the return appears cloudy and dark yellow. Increasing the flow would be contraindicated when there is no return of urine and irrigating solution.

49. A client is to receive belladonna and opium suppositories, as needed, postoperatively after transurethral resection of the prostate (TURP). The nurse should give the client these drugs when he demonstrates signs of: 1. A urinary tract infection. 2. Urine retention. 3. Frequent urination. 4. Pain from bladder spasms.

49. 4. Belladonna and opium suppositories are prescribed and administered to reduce bladder spasms that cause pain after TURP. Bladder spasms frequently accompany urologic procedures. Antispasmodics offer relief by eliminating or reducing spasms. Antimicrobial drugs are used to treat an infection. Belladonna and opium do not relieve urine retention or urinary frequency.

50. A nursing assistant tells the nurse, "I think the client is confused. He keeps telling me he has to void, but that isn't possible because he has a catheter in place that is draining well." Which of the following responses would be most appropriate for the nurse to make? 1. "His catheter is probably plugged. I'll irrigate it." 2. "That's a common problem after prostate surgery. The client only imagines the urge to void." 3. "The urge to void is usually created by the large catheter, and he may be having some bladder spasms." 4. "I think he may be somewhat confused."

50. 3. The indwelling urinary catheter creates the urge to void and can also cause bladder spasms. The nurse should ensure adequate bladder emptying by monitoring urine output and characteristics. Urine output should be at least 50 mL/h. A plugged catheter, imagining the urge to void, and confusion are less likely reasons for the client's problem.

51. A physician has prescribed amoxicillin (Ampicillin) 100 PO b.i.d. The nurse should teach the client to do which of the following? Select all that apply. 1. Drink 300 to 500 mL of fluids daily. 2. Void frequently, at least every 2 to 3 hours. 3. Take time to empty the bladder completely. 4. Take the last dose of the antibiotic for the day at bedtime. 5. Take the antibiotic with or without food.

51. 2, 3, 4, 5. Ampicillin may be given with or without food, but the nurse should instruct the client to obtain an adequate fluid intake (2,500 to 3,000 mL) to promote urinary output and to flush out bacteria from the urinary tract. The nurse should also encourage the client to void frequently (every 2 to 3 hours) and empty the bladder completely. Taking the antibiotic at bedtime, after emptying the bladder, helps to ensure an adequate concentration of the drug during the overnight period.

Which man is at greatest risk of developing prostate cancer?

52-year-old African American whose diet includes red meat

52. In discussing home care with a client after transurethral resection of the prostate (TURP), the nurse should teach the male client that dribbling of urine: 1. Can be a chronic problem. 2. Can persist for several months. 3. Is an abnormal sign that requires intervention. 4. Is a sign of healing within the prostate.

52. 2. Dribbling of urine can occur for several months after TURP. The client should be informed that this is expected and is not an abnormal sign. The nurse should teach the client perineal exercises to strengthen sphincter tone. The client may need to use pads for temporary incontinence. The client should be reassured that continence will return in a few months and will not be a chronic problem. Dribbling is not a sign of healing, but is related to the trauma of surgery.

53. The nurse should instruct the client who is being discharged to home 3 days after transurethral resection of the prostate (TURP) to do which of the following? Select all that apply. 1. Drink at least 3,000 mL water per day. 2. Increase calorie intake by eating six small meals a day. 3. Report bright red bleeding to the health care provider. 4. Take deep breaths and cough every 2 hours. 5. Report a temperature over 99°F.

53. 1, 3, 5. The nurse should instruct the client to drink a large amount of fluids (about 3,000 mL/day) to keep the urine clear. The urine should be almost without color. About 2 weeks after TURP, when desiccated tissue is sloughed out, a secondary hemorrhage could occur. The client should be instructed to call the surgeon or go to the emergency department if at any time the urine turns bright red. The nurse should also instruct the client to report signs of infection such as a temperature over 99°F. The client is not specifically at risk for nutritional problems after TURP and can resume a diet as tolerated. The client is not specifically at risk for airway problems because the procedure is done under spinal anesthesia and the client does not need to take deep breaths and cough.

54. A client with benign prostatic hypertrophy (BPH) has an elevated prostate-specific antigen (PSA) level. The nurse should: 1. Instruct the client to request having a colonoscopy before coming to conclusions about the PSA results. 2. Instruct the client that a urologist will monitor the PSA level biannually when elevated. 3. Determine if the prostatic palpation was done before or after the blood sample was drawn. 4. Ask the client if he emptied his bladder before the blood sample was obtained.

54. 3. Rectal and prostate examinations can increase serum PSA levels; therefore, instruct the client that a manual rectal examination is usually part of the test regimen to determine prostate changes. The prostatic palpation should be done after the blood sample is drawn. The PSA level must be monitored more often than biannually when it is elevated. Having a colonoscopy is not related to the findings of the PSA test. It is not necessary to void prior to having PSA blood levels tested.

17. The nurse is providing discharge instructions to a patient who has had a modified radical mastectomy. The nurse teaches the patient to do which of the following? A. Protect the arm from trauma. B. Avoid softening agents on the arm. C. Avoid any type of breast prosthesis for 12 weeks. D. Keep the arm positioned so that it is in a straight and dependent alignment.

A. Protect the arm from trauma.

A 71-year-old patient with a diagnosis of benign prostatic hyperplasia (BPH) has been scheduled for a contact laser technique. What is the primary goal of this intervention? A. Resumption of normal urinary drainage B. Maintenance of normal sexual functioning C. Prevention of acute or chronic renal failure D. Prevention of fluid and electrolyte imbalances

A. Resumption of normal urinary drainage

During a breast examination, which finding most strongly suggests that a client has breast cancer? a) Slight asymmetry of the breasts b) A fixed nodular mass with dimpling of the overlying skin c) Multiple firm, round, freely movable masses that change with the menstrual cycle d) Bloody discharge from the nipple

A fixed nodular mass with dimpling of the overlying skin Correct Explanation: A fixed nodular mass with dimpling of the overlying skin is common during late stages of breast cancer. Many women have slightly asymmetrical breasts. Bloody nipple discharge is a sign of intraductal papilloma, a benign condition. Multiple firm, round, freely movable masses that change with the menstrual cycle indicate fibrocystic breasts, a benign condition.

The nurse is providing shift report related to a client newly received back to the unit from the post anesthesia care unit (PACU). The nurse is stating that the client had breast tissue removed with 7 of 14 lymph nodes, the lining of the chest muscles and pectoralis minor muscle removed. The oncoming nurse documents which procedure completed? a) A modified radical mastectomy b) A radical mastectomy c) A total mastectomy d) A segmental mastectomy

A modified radical mastectomy Explanation: A modified radical mastectomy is a procedure in which the breast, some lymph nodes, the lining over the chest muscles, and the pectoralis minor muscle is removed. A segmental mastectomy is where the tumor and some breast tissue and lymph nodes are removed. A total mastectomy includes only breast tissue. A radical mastectomy includes the breast axillary lymph nodes, and pectoralis major and minor muscles are removed. Sternal lymph nodes may also be removed with this procedure.

A 54-year-old woman presents to her healthcare provider's office where you practice nursing. She is very concerned that she might have breast cancer, especially after caring for her sister that recently died from the disease. Included in your discussion is the primary and most common sign of breast cancer. Which of the following would meet this criterion? a) A painless mass in the breast, most often in the upper outer quadrant b) A painful mass in the breast, most often in the lower quadrant near the nipple c) A painless mass in the breast, most often in the lower quadrant near the nipple d) A painful mass in the breast, most often in the upper outer quadrant

A painless mass in the breast, most often in the upper outer quadrant Correct Explanation: The primary sign of breast cancer is a painless mass in the breast, most often in the upper outer quadrant.

11. The patient in a gynecology clinic asks the nurse, "What are the greatest risk factors for developing cervical cancer?" Which statement would be the nurse's best response? A. "The earlier the age of sexual activity and the more partners, the greater the risk." B. "Having no children and use of condoms puts you at the highest risk for cervical cancer." C. "Having routinely scheduled Pap smears will protect you from developing cervical cancer." D. "Eating fast foods that are high in fat and taking birth control pills are the greatest risk factors."

A. "The earlier the age of sexual activity and the more partners, the greater the risk."

After a vasectomy, what teaching should be included in the discharge teaching? A. "You will want to use an alternative form of contraception for 6 weeks." B. "You may lose some secondary sexual characteristics after this surgery." C. "You may have erectile dysfunction for several months after this surgery." D. "You will be uncomfortable, but you may safely have sexual intercourse today."

A. "You will want to use an alternative form of contraception for 6 weeks."

Patient will be receiving testosterone therapy for male hypogonadism and has a new prescription for transdermal testosterone (Testoderm). The nurse needs to include which teaching about the use of this medication? A. Apply the patch only to the scrotum. B. Apply the patch to the chest, back, shoulders. C. If the adverse effects become bothersome, the patient should stop using the patch. D. The patch should be applied to a different area of the upper body each day.

A. Apply the patch only to the scrotum.

A male patient wants know if there are any drugs that can be used for baldness. The nurse noticed that which drug in low dosages, is used for androgenetic alopecia in men? A. Finasteride (Propecia) B. Vardenafil (Levitra) C. Danazol (Danocrine) D. Oxandrolone (Oxandrin)

A. Finasteride (Propecia)

A patient has a new prescription for finasteride (Proscar). The nurse will instruct patient about which potential adverse effects? (SATA) A. Loss of erection B. Gynecomastia C. Headaches. D. Increased libido. E. Ejaculatory dysfunction.

A. Loss of erection B. Gynecomastia E. Ejaculatory dysfunction.

14. The nurse working on an in-patient gynecologic disorders unit knows which of the following are risk factors for the development of endometrial cancer? SELECT ALL THAT APPLY. A. Obesity. B. Multiparity. C. Increasing age. D. Ulcerative colitis. E. Diabetes mellitus.

A. Obesity. C. Increasing age. E. Diabetes mellitus.

12. When instructing participants during a health fair, the nurse should emphasize the importance of which of these routine diagnostic tests in detecting cervical cell abnormalities? A. Pap testing. B. Colposcopy. C. MRI of abdomen. D. Screening for CA-125 marker.

A. Pap testing.

During a counseling session for a group of teenage athletes, the use of androgenic steroids is discussed. The nurse will explain that which problem is a rare but devastating effect of androgenic steroid use? A. Peliosis of the liver B. Bradycardia C. Kidney failure D. Tachydysrhythmias

A. Peliosis of the liver

A nurse is providing education to a group of athletes on the topic of anabolic steroids. What side effects would the nurse include in the discussion with the group of athletes? (Select all that apply.) A. Sterility B. Liver cancer C. Impotence D. Stroke E. Cardiovascular disease

A. Sterility B. Liver cancer E. Cardiovascular disease Anabolic steroids have a great potential for misuse by athletes, especially bodybuilders and weight lifters, because of their muscle-building properties. Improper use of these substances can have many serious consequences, such as sterility, cardiovascular diseases, and even liver cancer. For this reason, anabolic steroids are currently classified as Schedule III controlled substances by the U.S. Drug Enforcement Administration. This classification implies that misuse of these drugs can lead to psychological or physical dependence or both

A 33-year-old patient noticed a painless lump in his scrotum on self-examination of his testicles and a feeling of heaviness. The nurse should first teach him about what diagnostic test? A. Ultrasound B. Cremasteric reflex C. Doppler ultrasound D. Transillumination with a flashlight

A. Ultrasound

8. The spouse of an elderly patient with early-stage prostate cancer asks the nurse, "Why isn't the physician treating my husband's cancer?" The nurse's best response should be based on which of the following? A. Watchful waiting is often appropriate for men over age 80 years. B. The patient must really have Stage IV cancer, which is not curable. C. All prostate cancer should be treated, and this patient should get another opinion. D. The patient is being treated with hormonal manipulation, which isn't perceived as "being treated" by patients.

A. Watchful waiting is often appropriate for men over age 80 years.

2. The nurse is reviewing laboratory data for a male patient being evaluated for testicular cancer. Which of the following are known to be tumor markers for testicular cancer? A. hCG and AFP. B. PSA and LDH. C. CA-125 and HPV. D. BRCA 1 and BRCA 2.

A. hCG and AFP.

Patients prescribed sildenafil (Viagra) should be instructed regarding the potential fatal drug interaction with which medication(s)? A. nitroglycerin B. aspirin C. acetaminophen D. Anticoagulants

A. nitroglycerin When taken in conjunction with nitroglycerin, sildenafil can cause severe hypotension unresponsive to treatment.

Post transurethral resection of the prostate, a client has a three-way catheter with a continuous bladder irrigation. Over the last 12 hours, there has been 1400 mL of irrigation solution infused and 2000 mL measured in output from the drainage bag. What is the recording of the urinary output for the 12-hour period? (Record your answer using a whole number.) ____ mL

ANS: 600 mL 2000 mL from the drainage bag (including both the irrigation fluid and urine) minus the 1400 mL of irrigation fluid equals 600 mL of urine: 2000 mL - 1400 mL = 600 mL.

A 34-year-old client comes to the clinic with concerns about an enlarged left testicle and heaviness in his lower abdomen. Which diagnostic test would the nurse expect to be ordered to confirm testicular cancer? a. Alpha-fetoprotein (AFP) b. Prostate-specific antigen (PSA) c. Prostate acid phosphatase (PAP) d. C-reactive protein (CRP)

ANS: A AFP is a glycoprotein that is elevated in testicular cancer. PSA and PAP testing is used in the screening of prostate cancer. CRP is diagnostic for inflammatory conditions.

The nurse is conducting a history on a male client to determine the severity of symptoms associated with prostate enlargement. Which finding is cause for prompt action by the nurse? A. Cloudy Urine B. Urinary hesitancy C. Post-void dribbling D. Weak Urinary Stream

ANS: A Cloudy urine could indicate infection due to possible urine retention and should be a priority action. Common symptoms of benign prostatic hyperplasia are urinary hesitancy, post-void dribbling, and a weak urinary stream due to the enlarged prostate causing bladder outlet obstruction.

A client is diagnosed with metastatic prostate cancer. The client asks the nurse the purpose of his treatment with the luteinizing hormone-releasing hormone (LH-RH) agonist leuprolide (Lupron) and the bisphosphonate pamidronate (Aredia). Which statement by the nurse is most appropriate? a. "The treatment reduces testosterone and prevents bone fractures." b. "The medications prevent erectile dysfunction and increase libido." c. "There is less gynecomastia and osteoporosis with this drug regimen." d. "These medications both inhibit tumor progression by blocking androgens."

ANS: A Lupron, an LH-RH agonist, stimulates the pituitary gland to release luteinizing hormone (LH) to the point that the gland is depleted of LH and testosterone production is lessened. This may decrease the prostate cancer since it is hormone dependent. Lupron can cause osteoporosis, which results in the need for Aredia to prevent bone loss. Erectile dysfunction, decreased libido, and gynecomastia are side effects of the LH-RH medications. Antiandrogen drugs inhibit tumor progression by blocking androgens at the site of the prostate.

The nurse is teaching an uncircumcised 65-year-old client about self-management of a urinary catheter in preparation for discharge to his home. What statement indicates a lack of understanding by the client? a. "I only have to wash the outside of the catheter once a week." b. "I should take extra time to clean the catheter site by pushing the foreskin back." c. "The drainage bag needs to be changed at least once a week and as needed." d. "I should pour a solution of vinegar and water through the tubing and bag."

ANS: A The first few inches of the catheter must be washed daily starting at the penis and washing outward with soap and water. The other options are correct for self-management of a urinary catheter in the home setting.

The nurse is teaching a client with benign prostatic hyperplasia (BPH). What statement indicates a lack of understanding by the client? a. "There should be no problem with a glass of wine with dinner each night." b. "I am so glad that I weaned myself off of coffee about a year ago." c. "I need to inform my allergist that I cannot take my normal decongestant." d. "My normal routine of drinking a quart of water during exercise needs to change."

ANS: A This client did not associate wine with the avoidance of alcohol, and requires additional teaching. The nurse must teach a client with BPH to avoid alcohol, caffeine, and large quantities of fluid in a short amount of time to prevent overdistention of the bladder. Decongestants also need to be avoided to lower the chance for urinary retention.

A 22-year-old man tells the nurse at the health clinic that he has recently had some problems with erectile dysfunction. When assessing for possible etiologic factors, which question should the nurse ask first? a. "Are you using any recreational drugs or drinking a lot of alcohol?" b. "Have you been experiencing an unusual amount of anxiety or stress?" c. "Do you have any history of an erection that lasted for 6 hours or more?" d. "Do you have any chronic cardiovascular or peripheral vascular disease?"

ANS: A A common etiologic factor for erectile dysfunction (ED) in younger men is use of recreational drugs or alcohol. Stress, priapism, and cardiovascular illness also contribute to ED, but they are not common etiologic factors in younger men.

A 53-year-old man tells the nurse he has been having increasing problems with erectile dysfunction (ED) for several years but is now interested in using Viagra (sildenafil). Which action should the nurse take first? a. Ask the patient about any prescription drugs he is taking. b. Tell the patient that Viagra does not always work for ED. c. Discuss the common adverse effects of erectogenic drugs. d. Assure the patient that ED is commonly associated with aging.

ANS: A Because some medications can cause ED and patients using nitrates should not take Viagra, the nurse should first assess for prescription drug use. The nurse may want to teach the patient about realistic expectations and adverse effects of Viagra therapy, but this should not be the first action. Although ED does increase with aging, it may be secondary to medication use or cardiovascular disease in a 53-year-old.

Which statement indicates that a client understands the most appropriate time of day to take an alpha blocker drug for treatment of benign prostatic hyperplasia (BPH)? a. "I'll take my medication at bedtime." b. "As soon as I get up, I will take my medication." c. "I will take my medication with food or milk." d. "I'll take my medication on an empty stomach."

ANS: A Bedtime dosing should decrease the risk of hypotension with an alpha blocker drug. Giving the medication during the day will increase the client's risk of experiencing weakness, lightheadedness, and dizziness.

A client with a catheter in place after a suprapubic prostatectomy has decreased urinary output. Which action by the nurse is most appropriate? a. Assess the client for bladder spasms. b. Encourage the client to drink more water. c. Administer pain medication. d. Have the client try to void around the catheter.

ANS: A Bladder spasms and decreased urinary output can indicate obstruction. The nurse should assess whether the client is experiencing these spasms and should treat the client with an antispasmodic if needed. Encouraging increased intake will not be helpful if the problem is obstruction. Pain medication will not be helpful, although an antispasmodic can be beneficial. Trying to void around the catheter is not recommended.

15. A client is placed on a medical regimen of doxorubicin (Adriamycin), cyclophosphamide (Cytoxan), and fluorouracil (5-FU) for breast cancer. Which side effect seen in the client should the nurse report to the provider immediately? a. Shortness of breath b. Nausea and vomiting c. Hair loss d. Mucositis

ANS: A Doxorubicin (Adriamycin) can cause cardiac problems with symptoms of extreme fatigue, shortness of breath, chronic cough, and edema. These need to be reported as soon as possible to the provider. Nausea, vomiting, hair loss, and mucositis are common problems associated with chemotherapy regimens. DIF: Applying/Application REF: 1477

When caring for a patient who has been diagnosed with orchitis, the nurse will plan to provide teaching about a. pain management. b. emergency surgical repair. c. aspiration of fluid from the scrotal sac. d. application of warm packs to the scrotum.

ANS: A Orchitis is very painful and effective pain management will be needed. The other therapies will not be used to treat orchitis.

A client had a seminoma removed via an open procedure. The following day, the client's abdomen is soft and nontender, but no bowel sounds are present. Which action by the nurse is most appropriate? a. Encourage the client to ambulate several times a day. b. Reassess the client's abdomen in 4 hours. c. Document the finding and call the surgeon immediately. d. Give the client a laxative and encourage high-fiber food.

ANS: A Paralytic ileus is a complication of open seminoma removal. However, on the day after a major, lengthy operation, it is common for bowel sounds to still be absent. Narcotic analgesics can diminish bowel activity. The client can improve peristalsis with increased activity. It is not necessary to re-examine the client in 4 hours, nor is it necessary to call the surgeon right away. A laxative probably is not needed yet. The client should eat high-fiber foods when they can be tolerated.

A client with BPH asks why his enlarged prostate is causing difficulty with urination. Which is the nurse's most accurate response? a. "It compresses the urethra, blocking the flow of urine." b. "It presses on the kidneys, decreasing urine formation." c. "It secretes acids that weaken the bladder, causing dribbling." d. "It destroys nerves, decreasing awareness of a full bladder."

ANS: A The prostate gland encircles the urethra and bladder neck like a doughnut. Enlargement of the gland constricts the urethra and obstructs the outflow of urine by encroaching on the bladder opening. The other responses are inaccurate.

17. A woman diagnosed with breast cancer had these laboratory tests performed at an office visit: Alkaline phosphatase 125 U/L Total calcium 12 mg/dL Hematocrit 39% Hemoglobin 14 g/dL Which test results indicate to the nurse that some further diagnostics are needed? a. Elevated alkaline phosphatase and calcium suggests bone involvement. b. Only alkaline phosphatase is decreased, suggesting liver metastasis. c. Hematocrit and hemoglobin are decreased, indicating anemia. d. The elevated hematocrit and hemoglobin indicate dehydration.

ANS: A The alkaline phosphatase (normal value 30 to 120 U/L) and total calcium (normal value 9 to 10.5 mg/dL) levels are both elevated, suggesting bone metastasis. Both the hematocrit and hemoglobin are within normal limits for females. DIF: Applying/Application REF: 1470

11. A client is discharged to home after a modified radical mastectomy with two drainage tubes. Which statement by the client would indicate that further teaching is needed? a. I am glad that these tubes will fall out at home when I finally shower. b. I should measure the drainage each day to make sure it is less than an ounce. c. I should be careful how I lie in bed so that I will not kink the tubing. d. If there is a foul odor from the drainage, I should contact my docto

ANS: A The drainage tubes (such as a Jackson-Pratt drain) lie just under the skin but need to be removed by the health care professional in about 1 to 3 weeks at an office visit. Drainage should be less than 25 mL in a days time. The client should be aware of her positioning to prevent kinking of the tubing. A foul odor from the drainage may indicate an infection; the doctor should be contacted immediately. DIF: Applying/Application REF: 1474

A 32-year-old man who is being admitted for a unilateral orchiectomy for testicular cancer does not talk to his wife and speaks to the nurse only to answer the admission questions. Which action is best for the nurse to take? a. Ask the patient if he has any questions or concerns about the diagnosis and treatment. b. Document the patient's lack of communication on the chart and continue preoperative care. c. Assure the patient's wife that concerns about sexual function are common with this diagnosis. d. Teach the patient and the wife that impotence is rarely a problem after unilateral orchiectomy.

ANS: A The initial action by the nurse should be assessment for any anxiety or questions about the surgery or postoperative care. The nurse should address the patient, not the spouse, when discussing the diagnosis and any possible concerns. Without further assessment of patient concerns, the nurse should not offer education about complications after orchiectomy. Documentation of the patient's lack of interaction is not an adequate nursing action in this situation.

A patient with symptomatic benign prostatic hyperplasia (BPH) is scheduled for photovaporization of the prostate (PVP) at an outpatient surgical center. The nurse will plan to teach the patient a. how to care for an indwelling urinary catheter. b. that the urine will appear bloody for several days. c. about complications associated with urethral stenting. d. that symptom improvement will occur in 2 to 3 weeks.

ANS: A The patient will have an indwelling catheter for 24 to 48 hours and will need teaching about catheter care. There is minimal bleeding with this procedure. Symptom improvement is almost immediate after PVP. Stent placement is not included in the procedure.

A 46-year-old man who has erectile dysfunction (ED) tells the nurse that he decided to seek treatment because his wife "is losing patience with the situation." The most appropriate nursing diagnosis for the patient is a. ineffective role performance related to effects of ED. b. anxiety related to inability to have sexual intercourse. c. situational low self-esteem related to decrease in sexual activity. d. ineffective sexuality patterns related to frequency of intercourse.

ANS: A The patient's statement indicates that the relationship with his wife is his primary concern. Although anxiety, low self-esteem, and ineffective sexuality patterns also may be concerns, the patient information suggests that addressing the role performance problem will lead to the best outcome for this patient.

Which information will the nurse plan to include when teaching a 19-year-old to perform testicular self-examination? a. Testicular self-examination should be done in a warm area. b. The only structure normally felt in the scrotal sac is the testis. c. Testicular self-examination should be done at least every week. d. Call the health care provider if one testis is larger than the other.

ANS: A The testes will hang lower in the scrotum when the temperature is warm (e.g., during a shower), and it will be easier to palpate. The epididymis is also normally palpable in the scrotum. One testis is normally larger. The patient should perform testicular self-examination monthly.

4. A woman has been using acupuncture to treat the nausea and vomiting caused by the side effects of chemotherapy for breast cancer. Which conditions would cause the nurse to recommend against further use of acupuncture? (Select all that apply.) a. Lymphedema b. Bleeding tendencies c. Low white blood cell count d. Elevated serum calcium e. High platelet count

ANS: A, B, C Acupuncture could be unsafe for the client if there is poor drainage of the extremity with lymphedema or if there was a bleeding tendency and low white blood cell count. Coagulation would be compromised with a bleeding disorder, and the risk of infection would be high with the use of needles. An elevated serum calcium and high platelet count would not have any contraindication for acupuncture. DIF: Remembering/Knowledge REF: 1472

A client came to the clinic with erectile dysfunction. What are some possible causes of this condition that the nurse could discuss with the client during history taking? (Select all that apply.) a. Recent prostatectomy b. Long-term hypertension c. Diabetes mellitus d. Hour-long exercise sessions e. Consumption of beer each night

ANS: A, B, C, E Organic erectile dysfunction can be caused by surgical procedures, hypertension and its treatment, diabetes mellitus, and alcohol consumption. There is no evidence that exercise is related to this problem.

Which symptoms are expected in orchitis? (Select all that apply.) a. Scrotal pain b. Dysuria c. Scrotal edema d. Priapism e. Penile discharge f. Inability to ejaculate

ANS: A, B, C, E Manifestations of orchitis include scrotal pain, edema, reports of heavy feelings in the involved testicle(s), dysuria, pain on ejaculation, blood in the semen, and discharge from the penis. Ejaculation may be painful. Priapism is not a manifestation.

The nurse is administering finasteride (Proscar) and doxazosin (Cardura) to a 67-year-old client with benign prostatic hyperplasia. What precautions are related to the side effects of these medications? (Select all that apply.) a. Assessing for blood pressure changes when lying, sitting, and arising from the bed b. Immediately reporting any change in the alanine aminotransferase laboratory test c. Teaching the client about the possibility of increased libido with these medications d. Taking the client's pulse rate for a minute in anticipation of bradycardia e. Asking the client to report any weakness, light-headedness, or dizziness

ANS: A, B, E Both the 5-alpha-reductase inhibitor (5-ARI) and the alpha1-selective blocking agents can cause orthostatic (postural) hypotension and liver dysfunction. The 5-ARI agent (Proscar) can cause a decreased libido rather than an increased sexual drive. The alpha-blocking drug (Cardura) can cause tachycardia rather than bradycardia.

1. The nurse is taking a history of a 68-year-old woman. What assessment findings would indicate a high risk for the development of breast cancer? (Select all that apply.) a. Age greater than 65 years b. Increased breast density c. Osteoporosis d. Multiparity e. Genetic factors

ANS: A, B, E The high risk factors for breast cancer are age greater than 65 with the risk increasing until age 80; an increase in breast density because of more glandular and connective tissue; and inherited mutations of BRCA1 and/or BRCA2 genes. Osteoporosis and multiparity are not risk factors for breast cancer. A high postmenopausal bone density and nulliparity are moderate and low increased risk factors, respectively.

3. After a breast examination, the nurse is documenting assessment findings that indicate possible breast cancer. Which abnormal findings need to be included as part of the clients electronic medical record? (Select all that apply.) a. Peau dorange b. Dense breast tissue c. Nipple retraction d. Mobile mass at two oclock e. Nontender axillary nodes

ANS: A, C, D In the documentation of a breast mass, skin changes such as dimpling (peau dorange), nipple retraction, and whether the mass is fixed or movable are charted. The location of the mass should be stated by the face of a clock. Dense breast tissue and nontender axillary nodes are not abnormal assessment findings that may indicate breast cancer. DIF: Remembering/Knowledge REF: 1469

2. The nurse is formulating a teaching plan according to evidence-based breast cancer screening guidelines for a 50-year-old woman with low risk factors. Which diagnostic methods should be included in the plan? (Select all that apply.) a. Annual mammogram b. Magnetic resonance imaging (MRI) c. Breast ultrasound d. Breast self-awareness e. Clinical breast examination

ANS: A, D, E Guidelines recommend a screening annual mammogram for women ages 40 years and older, breast self-awareness, and a clinical breast examination. An MRI is recommended if there are known high risk factors. A breast ultrasound is used if there are problems discovered with the initial screening or dense breast tissue. DIF: Applying/Application REF: 1467

A client is interested in learning about the risk factors for prostate cancer. Which factors does the nurse include in the teaching? (Select all that apply.) a. Family history of prostate cancer b. Smoking c. Obesity d. Advanced age e. Eating too much red meat f. Race

ANS: A, D, E, F Advanced family history of prostate cancer, age, a diet high in animal fat, and race are all risk factors for prostate cancer. Smoking and obesity are not known risk factors.

The nurse is administering sulfamethoxazole-trimethoprim (Bactrim) to a client diagnosed with bacterial prostatitis. Which finding causes the nurse to question this medication for this client? a. Urinary tract infection b. Allergy to sulfa medications c. Hematuria d. Elevated serum white blood cells

ANS: B Before administering sulfamethoxazole-trimethoprim, the nurse must assess if the client is allergic to sulfa drugs. Urinary tract infection, hematuria, and elevated serum white blood cells are common problems associated with bacterial prostatitis that require long-term antibiotic therapy.

A client has returned from a transurethral resection of the prostate with a continuous bladder irrigation. Which action by the nurse is a priority if bright red urinary drainage and clots are noted 5 hours after the surgery? a. Review the hemoglobin and hematocrit as ordered. b. Take vital signs and notify the surgeon immediately. c. Release the traction on the three-way catheter. d. Remind the client not to pull on the catheter.

ANS: B Bright red urinary drainage with clots may indicate arterial bleeding. Vital signs should be taken and the surgeon notified. The traction on the three-way catheter should not be released since it places pressure at the surgical site to avoid bleeding. The nurse's review of hemoglobin and hematocrit and reminding the client not to pull on the catheter are good choices, but not the priority at this time.

A client who was treated 1 year ago for testicular cancer now has an elevated serum alpha-fetoprotein level. Which topic is most important for the nurse to teach this client about? a. Sperm banking options b. Effects of chemotherapy c. Hospice philosophy d. Importance of yearly monitoring

ANS: B Alpha-fetoprotein is a tumor marker that is not produced in significant amounts by normal adult tissues. An increase in the level of alpha-fetoprotein after treatment most commonly indicates recurrence or metastasis. Chemotherapy is used to treat recurrent or metastatic disease. The client should already have been taught about sperm banking. Hospice is not indicated at this time. Because a rise in these levels indicates recurrence or metastasis, yearly monitoring is not appropriate.

A 70-year-old client returned from a transurethral resection of the prostate 8 hours ago with a continuous bladder irrigation. The nurse reviews his laboratory results as follows: Sodium 128 mEq/L Hemoglobin 14 g/dL Hematocrit 42% Red blood cell count 4.5 What action by the nurse is the most appropriate? a. Consider starting a blood transfusion. b. Slow down the bladder irrigation if the urine is pink. c. Report the findings to the surgeon immediately. d. Take the vital signs every 15 minutes.

ANS: B The serum sodium is decreased due to large-volume bladder irrigation (normal is 136 to 145 mEq/L). By slowing the irrigation, there will be less fluid overload and sodium dilution. The hemoglobin and hematocrit values are a low normal, with a slight decrease in the red blood cell count. Therefore, a blood transfusion or frequent vital signs should not be necessary. Immediate report to the surgeon is not necessary.

The health care provider prescribes finasteride (Proscar) for a 56-year-old patient who has benign prostatic hyperplasia (BPH). When teaching the patient about the drug, the nurse informs him that a. he should change position from lying to standing slowly to avoid dizziness. b. his interest in sexual activity may decrease while he is taking the medication. c. improvement in the obstructive symptoms should occur within about 2 weeks. d. he will need to monitor his blood pressure frequently to assess for hypertension.

ANS: B A decrease in libido is a side effect of finasteride because of the androgen suppression that occurs with the drug. Although orthostatic hypotension may occur if the patient also is taking a medication for erectile dysfunction (ED), it should not occur with finasteride alone. Improvement in symptoms of obstruction takes about 6 months. The medication does not cause hypertension.

A client's prostate-specific antigen (PSA) level is 2.0 ng/mL. Which action by the nurse is most appropriate? a. Inform the client that the results are normal and no cancer has been detected. b. Inform the client that results are normal and schedule a digital rectal examination. c. Let the client know that the results are elevated and he is at risk for prostate cancer. d. Tell the client that cancer is indicated and that the health care provider recommends watchful waiting.

ANS: B A normal PSA in men younger than age 50 is less than 2.5 ng/mL. Although the finding is within normal limits for a PSA value, a client could have prostate cancer and not present with an elevated PSA. Also, laboratory findings should not be used as the sole screening tool. Without a digital rectal examination (DRE), the health care provider cannot know whether the client is in the early stages of prostate cancer. The client should be informed that although the level is within the normal range, he still needs a DRE.

When performing discharge teaching for a patient who has undergone a vasectomy in the health care provider's office, the nurse instructs the patient that a. he may have temporary erectile dysfunction (ED) because of postoperative swelling. b. he should continue to use other methods of birth control for 6 weeks. c. he should not have sexual intercourse until his 6-week follow-up visit. d. he will notice a decrease in the appearance and volume of his ejaculate.

ANS: B Because it takes about 6 weeks to evacuate sperm that are distal to the vasectomy site, the patient should use contraception for 6 weeks. ED that occurs after vasectomy is psychologic in origin and not related to postoperative swelling. The patient does not need to abstain from intercourse. The appearance and volume of the ejaculate are not changed because sperm are a minor component of the ejaculate.

A patient with benign prostatic hyperplasia (BPH) with mild obstruction tells the nurse, "My symptoms have gotten a lot worse this week." Which response by the nurse is most appropriate? a. "I will talk to the doctor about ordering a prostate specific antigen (PSA) test." b. "Have you been taking any over-the-counter (OTC) medications recently?" c. "Have you talked to the doctor about surgical procedures such as transurethral resection of the prostate (TURP)?" d. "The prostate gland changes slightly in size from day to day, and this may be making your symptoms worse."

ANS: B Because the patient's increase in symptoms has occurred abruptly, the nurse should ask about OTC medications that might cause contraction of the smooth muscle in the prostate and worsen obstruction. The prostate gland does not vary in size from day to day. A TURP may be needed, but more assessment about possible reasons for the sudden symptom change is a more appropriate first response by the nurse. PSA testing is done to differentiate BPH from prostatic cancer.

A client's laboratory findings reveal an elevated serum acid phosphatase level and a high-normal prostate-specific antigen level. How does the nurse interpret this information? a. The client shows evidence of renal disease and should be evaluated further. b. These results may indicate prostate cancer. He should be further evaluated. c. These results are not abnormal. He does not need to be evaluated further. d. These results may indicate an infection. He should be evaluated further.

ANS: B Both serum acid phosphatase and prostate-specific antigen levels will be elevated when the client has prostate cancer. The results are not indicative of renal disease or infection, but they are abnormal, may indicate prostate cancer, and should be further evaluated.

A client is receiving brachytherapy for prostate cancer. Which intervention is most important for the nurse to include in the client's care plan? a. Reassure the client that erectile dysfunction does not occur with brachytherapy. b. Help the client plan activities interspersed with rest periods during the day. c. Remind the client that while hospitalized, he cannot have any visitors. d. Discuss hospice philosophy and availability with the client and family.

ANS: B Fatigue is common throughout treatment and may continue for several months after treatment has concluded. The client will need to learn to manage the fatigue; this can include spacing activities and planning for rest periods throughout the day. Erectile dysfunction can occur as a side effect of brachytherapy. This procedure is usually done on an outpatient basis, and the client does not pose a danger to others. Brachytherapy is often a first-line treatment choice, so discussion of hospice is not appropriate at this time.

Leuprolide (Lupron) is prescribed for a patient with cancer of the prostate. In teaching the patient about this drug, the nurse informs the patient that side effects may include a. dizziness. b. hot flashes. c. urinary incontinence. d. increased infection risk.

ANS: B Hot flashes may occur with decreased testosterone production. Dizziness may occur with the -blockers used for benign prostatic hyperplasia (BPH). Urinary incontinence may occur after prostate surgery, but it is not an expected medication side effect. Risk for infection is increased in patients receiving chemotherapy.

16. A client is concerned about the risk of lymphedema after a mastectomy. Which response by the nurse is best? a. You do not need to worry about lymphedema since you did not have radiation therapy. b. A risk factor for lymphedema is infection, so wear gloves when gardening outside. c. Numbness, tingling, and swelling are common sensations after a mastectomy. d. The risk for lymphedema is a real threat and can be very self-limiting.

ANS: B Infection can create lymphedema; therefore, the client needs to be cautious with activities using the affected arm, such as gardening. Radiation therapy is just one of the factors that could cause lymphedema. Other risk factors include obesity and the presence of axillary disease. The symptoms of lymphedema are heaviness, aching, fatigue, numbness, tingling, and swelling, and are not common after the surgery. Women with lymphedema live fulfilling lives. DIF: Applying/Application REF: 1478

Which client is most likely to have organic erectile dysfunction? a. Middle-aged man who first had sexual intercourse at age 15 b. Middle-aged man who has had diabetes mellitus for 25 years c. Young man who had a myocardial infarction 2 years ago d. Young man who has a job that causes him high stress levels

ANS: B Organic erectile dysfunction occurs as a gradual reduction in sexual functioning. Diabetes mellitus causes microvascular and macrovascular complications that decrease the sensation and autonomic nerve activity required for achievement of an erection. The other factors will not increase the client's risk for development of organic erectile dysfunction.

Following a radical retropubic prostatectomy for prostate cancer, the patient is incontinent of urine. The nurse will plan to teach the patient a. to restrict oral fluid intake. b. pelvic floor muscle exercises. c. the use of belladonna and opium suppositories. d. how to perform intermittent self-catheterization.

ANS: B Pelvic floor muscle training (Kegel) exercises are recommended to strengthen the pelvic floor muscles and improve urinary control. Belladonna and opium suppositories are used to reduce bladder spasms after surgery. Intermittent self-catheterization may be taught before surgery if the patient has urinary retention, but it will not be useful in reducing incontinence after surgery. The patient should have a daily oral intake of 2 to 3 L.

A couple is seen at the infertility clinic because they have not been able to conceive. When performing a focused examination to determine any possible causes for infertility, the nurse will check the man for the presence of a. hydrocele. b. varicocele. c. epididymitis. d. paraphimosis.

ANS: B Persistent varicoceles are commonly associated with infertility. Hydrocele, epididymitis, and paraphimosis are not risk factors for infertility

The nurse working in a health clinic receives calls from all these patients. Which patient should be seen by the health care provider first? a. A 44-year-old man who has perineal pain and a temperature of 100.4° F b. A 66-year-old man who has a painful erection that has lasted over 7 hours c. A 62-year-old man who has light pink urine after having a transurethral resection of the prostate (TURP) 3 days ago d. A 23-year-old man who states he had difficulty maintaining an erection last night

ANS: B Priapism can cause complications such as necrosis or hydronephrosis, and this patient should be treated immediately. The other patients do not require immediate action to prevent serious complications.

Following discharge teaching for a patient who has had a transurethral resection of the prostate (TURP) for benign prostatic hyperplasia (BPH), the nurse determines that additional instruction is needed when the patient says, a. "I will avoid driving until I get approval from my doctor." b. "I should call the doctor if I have any incontinence at home." c. "I will increase fiber and fluids in my diet to prevent constipation." d. "I should continue to schedule yearly appointments for prostate exams."

ANS: B Since incontinence is common for several weeks after a TURP, the patient does not need to call the health care provider if this occurs. The other patient statements indicate that the patient has a good understanding of post-TURP instructions.

A client has been taking finasteride (Proscar) for 4 weeks and reports that he has not yet seen a reduction in symptoms. Which response by the nurse is most appropriate? a. "Have you been taking the medication as ordered?" b. "It may take several months to see results." c. "It may not be the right drug for you." d. "We can try dutasteride (Avodart) next."

ANS: B So that he does not become discouraged, the nurse should first reassure the client that this class of medications may take up to 6 months to be effective. The nurse then can assess for compliance, but asking that question first may put the client on the defensive. The client needs to try the medication for several more months before the health care team changes it. Avodart is in the same class of medications, and its use for up to 6 months of therapy may be required before results are seen.

A patient with benign prostatic hyperplasia (BPH) is admitted to the hospital with urinary retention and new onset elevations in the blood urea nitrogen (BUN) and creatinine. Which of these prescribed therapies should the nurse implement first? a. Schedule an abdominal computed tomography (CT) scan. b. Insert a urinary retention catheter. c. Draw blood for a complete blood count. d. Infuse normal saline at 50 mL/hr.

ANS: B The patient data indicate that the patient may have acute renal failure caused by the BPH. The initial therapy will be to insert a catheter. The other actions also are appropriate, but they can be implemented after the acute urinary retention is resolved.

A client had a transurethral prostatectomy and has incontinence. Which statement by the client indicates a need for clarification about managing this condition? a. "I will practice stopping the urine stream to strengthen my sphincter control." b. "I will limit my fluid intake every day to prevent incontinence." c. "I will avoid vigorous activity for the first 3 weeks after surgery." d. "I will avoid caffeinated beverages and spicy foods."

ANS: B Unless fluid restriction is needed because of another medical problem, clients with incontinence should drink plenty of water and other fluids. Client statements regarding Kegel exercises, activity restrictions post-surgery, and avoiding bladder irritants are all indicative of understanding.

An older client with benign prostatic hyperplasia (BPH) and hypertension is being treated with doxazosin (Cardura) while staying in the hospital. Which activity does the nurse delegate to the unlicensed assistive personnel (UAP) as a priority? a. Helping the client choose low-sodium meal items b. Assisting the client whenever he gets out of bed c. Encouraging the client to use the spirometer hourly d. Frequently re-orienting the client to his surroundings

ANS: B When treating a client in an inpatient setting with alpha blockers such as doxazosin (Cardura) or terazosin (Hytrin), the nurse must provide for the client's safety because this medication can cause orthostatic hypotension or syncope. The nurse should instruct the UAP to help the client whenever he gets out of bed, to prevent injury. Because this medication is being used for BPH and not for hypertension, a low-sodium diet is not necessary. Using the spirometer is always a good intervention, but it use is not related to safety and to this medication. The client, although older, may not be confused and may not need frequent reorientation.

A client has returned to the nursing unit after a prostatectomy. Which activities does the nurse delegate to the unlicensed assistive personnel (UAP)? (Select all that apply.) a. Demonstrating how to use the incentive spirometer b. Measuring and recording output from the in-dwelling catheter c. Encouraging the client to get out of bed and into the chair d. Irrigating the catheter with normal saline for blood clots e. Re-taping the catheter tape if the client reports pain

ANS: B, C The UAP can assess and document intake and output and can encourage the client to get out of bed. Use of the incentive spirometer is taught by the nurse or respiratory therapist. The catheter is irrigated by the nurse. The catheter should be taped so that slight traction is left on it to help with bleeding; this may cause discomfort and would need to be explained to the client. The catheter should not be repositioned and then re-taped.

A 55-year-old male client is admitted to the emergency department with symptoms of a myocardial infarction. Which question by the nurse is the most appropriate before administering nitroglycerin? a. "On a scale from 0 to 10, what is the rating of your chest pain?" b. "Are you allergic to any food or medications?" c. "Have you taken any drugs like Viagra recently?" d. "Are you light-headed or dizzy right now?"

ANS: C Phosphodiesterase-5 inhibitors such as sildenafil (Viagra) relax smooth muscles to increase blood flow to the penis for treatment of erectile dysfunction. In combination with nitroglycerin, there can be extreme hypotension with reduction of blood flow to vital organs. The other questions are appropriate but not the highest priority before administering nitroglycerin.

A 51-year-old man is scheduled for an annual physical exam at the outpatient clinic. The nurse will plan to teach the patient about the purpose of a. urinalysis collection. b. uroflowmetry studies. c. prostate specific antigen (PSA) testing. d. transrectal ultrasound scanning (TRUS).

ANS: C An annual digital rectal exam (DRE) and PSA are recommended starting at age 50 for men who have an average risk for prostate cancer. Urinalysis and uroflowmetry studies are done if patients have symptoms of urinary tract infection or changes in the urinary stream. TRUS may be ordered if the DRE or PSA are abnormal.

A 55-year-old African-American client is having a visit with his health care provider. What test should the nurse discuss with the client as an option to screen for prostate cancer, even though screening is not routinely recommended? a. Complete blood count b. Culture and sensitivity c. Prostate-specific antigen d. Cystoscopy

ANS: C The prostate-specific antigen test should be discussed as an option for prostate cancer screening. A complete blood count and culture and sensitivity laboratory test will be ordered if infection is suspected. A cystoscopy would be performed to assess the effect of a bladder neck obstruction.

The nurse is assessing a client who has undergone a transurethral resection of the prostate (TURP). Which assessment finding requires immediate action by the nurse? a. Passing small blood clots after catheter removal b. Experiencing urinary frequency after catheter removal c. Having bright red drainage with multiple blood clots d. Having the urge to void continuously while the catheter is inserted

ANS: C A client who undergoes a TURP is at risk for bleeding during the first 24 hours after surgery. Passage of small blood clots and tissue debris, urinary frequency and leakage, and the urge to void continuously while the client still has the catheter inserted are all considered to be expected complications of the procedure. They will resolve as the client continues to recover and the catheter is removed. However, the presence of bright red blood with clots indicates arterial bleeding and should be reported to the provider.

A client had a spermatocele removed in an outpatient surgical center. Which statement by the client indicates good understanding of discharge instructions? a. "The heavy drainage will go away within a few days." b. "I need to buy dressing supplies at the drugstore." c. "I should report any redness or drainage from the incision." d. "Because of all the narcotics I'll be taking, I will need laxatives."

ANS: C A spermatocele is removed via a small scrotal incision. Heavy drainage should not occur, nor should extensive dressing supplies be needed. The small incision should not require the use of large doses of narcotics.

A client is scheduled for a penectomy for penile cancer. Which action by the nurse is most important? a. Teaching the client to sit when he urinates b. Demonstrating dressing changes and wound cleaning c. Assessing the client's psychosocial status and support d. Explaining the purpose of the in-dwelling catheter

ANS: C Clients may have strong emotional responses to penectomy, even when they seem to be accepting of the surgery, and the risk for suicide is present. It is critical to assess the client's emotional status and support systems before the operation is performed (and afterward). The other actions are appropriate too but do not take priority over ensuring the client's safety.

A client diagnosed with early prostate cancer is confused that surgery has not been planned. Which is the nurse's best response? a. "The disease is slow-growing. The risks of surgery at your age are not justified by the outcome." b. "Your disease is so advanced that surgery at this point would not increase your chances of cure." c. "Your disease is in a very early stage and is slow-growing. Your doctor will monitor you." d. "This stage indicates that you do not really have cancer, so surgery is not necessary."

ANS: C Early prostate cancer may have no clinical manifestations and may be found on a routine physical. It is slow-growing and may never become a problem for the client. Close follow-up (or watchful waiting) is the common prescription for this stage unless the client experiences symptoms. Telling the client that surgery is not justified, or that the cancer is too advanced at this point, and stating that he does not have cancer are inaccurate statements.

Which information will the nurse include when teaching a patient who has a diagnosis of chronic prostatitis? a. Ibuprofen (Motrin) should provide good pain control. b. Prescribed antibiotics should be taken for 7 to 10 days. c. Sexual intercourse and masturbation will help relieve symptoms. d. Cold packs should be used every 4 hours to reduce inflammation.

ANS: C Ejaculation helps drain the prostate and relieve pain. Warm baths are recommended to reduce pain. Nonsteroidal antiinflammatory drugs (NSAIDs) are frequently prescribed but usually do not offer adequate pain relief. Antibiotics for chronic prostatitis are taken for 4 to 12 weeks.

10. A client has just returned from a right radical mastectomy. Which action by the unlicensed assistive personnel (UAP) would the nurse consider unsafe? a. Checking the amount of urine in the urine catheter collection bag b. Elevating the right arm on a pillow c. Taking the blood pressure on the right arm d. Encouraging the client to squeeze a rolled washcloth

ANS: C Health care professionals need to avoid the arm on the side of the surgery for blood pressure measurement, injections, or blood draws. Since lymph nodes are removed, lymph drainage would be compromised. The pressure from the blood pressure cuff could promote swelling. Infection could occur with injections and blood draws. Checking urine output, elevation of the affected arm on a pillow, and encouraging beginning exercises are all safe postoperative interventions. DIF: Applying/Application REF: 1473

7. With a history of breast cancer in the family, a 48-year-old female client is interested in learning about the modifiable risk factors for breast cancer. After the nurse explains this information, which statement made by the client indicates that more teaching is needed? a. I am fortunate that I breast-fed each of my three children for 12 months. b. It looks as though I need to start working out at the gym more often. c. I am glad that we can still have wine with every evening meal. d. When I have menopausal symptoms, I must avoid hormone replacement therapy.

ANS: C Modifiable risk factors can help prevent breast cancer. The client should lessen alcohol intake and not have wine 7 days a week. Breast-feeding, regular exercise, and avoiding hormone replacement are also strategies for breast cancer prevention. DIF: Applying/Application REF: 1465

A client with prostate cancer reports pain in his lower back and legs. Which action by the nurse is most appropriate? a. Discuss medications for arthritis. b. Perform a bladder scan. c. Facilitate imaging studies. d. Encourage weight-bearing exercises.

ANS: C The primary site of metastasis for prostate cancer is the bone of the spine and the legs. The nurse should suspect metastasis and inform the health care provider. The client will need imaging studies to look for metastasis, and the nurse should facilitate them. The other interventions are not appropriate.

12. What comfort measure can only be performed by a nurse, as opposed to an unlicensed assistive personnel (UAP), for a client who returned from a left modified radical mastectomy 4 hours ago? a. Placing the head of bed at 30 degrees b. Elevating the left arm on a pillow c. Administering morphine for pain at a 4 on a 0-to-10 scale d. Supporting the left arm while initially ambulating the client

ANS: C Only the nurse is authorized to administer medications, but the UAP could inform the nurse about the rating of pain by the client. The UAP could position the bed to 30 degrees and elevate the clients arm on a pillow to facilitate lymphatic fluid drainage return. The clients arm should be supported while walking at first but then allowed to hang straight by the side. The UAP could support the arm while walking the client. DIF: Applying/Application REF: 1474

A 25-year-old client has recently been diagnosed with testicular cancer and is scheduled for radiation therapy. Which intervention by the nurse is best? a. Ask the client about his support system of friends and relatives. b. Encourage the client to verbalize his fears about sexual performance. c. Explore with the client the possibility of sperm collection. d. Provide privacy to allow time for reflection about the treatment.

ANS: C Sperm collection is a viable option for a client diagnosed with testicular cancer and should be completed before radiation therapy, chemotherapy, or radical lymph node dissection. The other options would promote psychosocial support but are not the priority intervention.

9. A 35-year-old woman is diagnosed with stage III breast cancer. She seems to be extremely anxious. What action by the nurse is best? a. Encourage the client to search the Internet for information tonight. b. Ask the client if sexuality has been a problem with her partner. c. Explore the idea of a referral to a breast cancer support group. d. Assess whether there has been any mental illness in her past.

ANS: C Support for the diagnosis would be best with a referral to a breast cancer support group. The Internet may be a good source of information, but the day of diagnosis would be too soon. The nurse could assess the frequency and satisfaction of sexual relations but should not assume that there is a problem in that area. Assessment of mental illness is not an appropriate action. DIF: Applying/Application REF: 1469

14. A client is starting hormonal therapy with tamoxifen (Nolvadex) to lower the risk for breast cancer. What information needs to be explained by the nurse regarding the action of this drug? a. It blocks the release of luteinizing hormone. b. It interferes with cancer cell division. c. It selectively blocks estrogen in the breast. d. It inhibits DNA synthesis in rapidly dividing cells.

ANS: C Tamoxifen (Nolvadex) reduces the estrogen available to breast tumors to stop or prevent growth. This drug does not block the release of luteinizing hormone to prevent the ovaries from producing estrogen; leuprolide (Lupron) does this. Chemotherapy agents such as ixabepilone (Ixempra) interfere with cancer cell division, and doxorubicin (Adriamycin) inhibits DNA synthesis in susceptible cells. DIF: Remembering/Knowledge REF: 1477

Which assessment information about a 62-year-old man is most important for the nurse to report to the health care provider when the patient is asking for a prescription for testosterone replacement therapy? a. The patient's symptoms have increased steadily over the last few years. b. The patient has been using sildenafil (Viagra) several times every week. c. The patient has had a gradual decrease in the force of his urinary stream. d. The patient states that he has noticed a decrease in energy level for a few years.

ANS: C The decrease in urinary stream may indicate benign prostatic hyperplasia (BPH) or prostate cancer, which are contraindications to the use of testosterone replacement therapy (TRT). The other patient data indicate that TRT may be a helpful therapy for the patient.

When reviewing patient laboratory results, the nurse in the clinic notes elevated prostate specific antigen (PSA) levels in the following four patients. Which patient's PSA result is most important to report to the health care provider? a. A 75-year-old who uses saw palmetto to treat benign prostatic hyperplasia (BPH) b. A 38-year-old who is being treated for acute prostatitis c. A 48-year-old whose father died of metastatic prostate cancer d. A 52-year-old who goes on long bicycle rides every weekend

ANS: C The family history and elevation of PSA in the 48-year-old indicate that further evaluation of the patient for prostate cancer is needed. The elevations in PSA for the other patients are not unusual.

A 64-year-old has a perineal radical prostatectomy for prostatic cancer. In the immediate postoperative period, the nurse establishes the nursing diagnosis of risk for infection related to a. urinary stasis. b. urinary incontinence. c. possible fecal contamination of the surgical wound. d. placement of a suprapubic catheter into the bladder.

ANS: C The perineal approach increases the risk for infection because the incision is located close to the anus and contamination with feces is possible. Urinary stasis and incontinence do not occur because the patient has a retention catheter in place for 1 to 2 weeks. A urethral catheter is used after the surgery.

When teaching a patient who is scheduled for a transurethral resection of the prostate (TURP) about continuous bladder irrigation, which information will the nurse include? a. Bladder irrigation decreases the risk of postoperative bleeding. b. Hydration and urine output are maintained by bladder irrigation. c. Bladder irrigation prevents obstruction of the catheter after surgery. d. Antibiotics are infused on a continuous basis with bladder irrigation.

ANS: C The purpose of bladder irrigation is to remove clots from the bladder and to prevent obstruction of the catheter by clots. The irrigation does not decrease bleeding or improve hydration. Antibiotics are given by the IV route, not through the bladder irrigation.

The nurse is caring for a young adult who just got married and has been diagnosed with testicular cancer. To which community resource does the nurse refer him? a. American Cancer Society b. Red Cross c. Sperm bank d. Public Health Department

ANS: C The young man with testicular cancer should be referred to a sperm bank, so that he will have the option to have children in the future if he so desires. The other resources listed will not provide assistance in this area. The American Cancer Society does offer several resources for clients with cancer and their families, but referral to a sperm bank would be the priority owing to the man's age and his newly married status.

A client is taking goserelin (Zoladex). What periodic assessment does the nurse plan for this client? a. Weight and abdominal girth b. Pulmonary function tests c. Bone density testing d. Abdominal ultrasound

ANS: C Zoladex is a luteinizing hormone-releasing hormone (LH-RH) agonist. Side effects include hot flashes, erectile dysfunction, decreased libido, gynecomastia, and osteoporosis. A periodic bone density screening test should be done to assess for osteoporosis. The other assessments would not be needed to assess for side effects of this drug.

A client is diagnosed with benign prostatic hyperplasia and seems sad and irritable. After assessing the client's behavior, which statement by the nurse would be the most appropriate? A. "The urine incontinence should not prevent you from socializing." B. "You seem depressed and should seek more pleasant things to do." C. "It is common for men at your age to have changes in mood." D. "Nocturia could cause interruption of your sleep and cause changes in mood."

ANS: D Frequent visits to the bathroom during the night could cause sleep interruptions and affect the client's mood and mental status. Incontinence could cause the client to feel embarrassment and cause him to limit his activities outside the home. The social isolation could lead to clinical depression and should be treated professionally. The nurse should not give advice before exploring the client's response to his change in behavior. The statement about age has no validity.

Which client statement indicates understanding about a transrectal ultrasound? a. "This will determine if the outlet of my bladder is obstructed." b. "This will determine the amount of residual urine present." c. "This is performed to view the interior of the bladder and urethra." d. "This is performed to view the prostate and do a tissue biopsy."

ANS: D A transrectal ultrasound is performed to view the prostate and surrounding structures and possibly also to do a tissue biopsy. A urodynamic pressure flow study will determine if the outlet of the client's bladder is obstructed. A bladder scan will determine the amount of residual urine that is present. A cytoscopy will allow the interior of the bladder and urethra to be visualized.

Which finding by the nurse will be most helpful in determining whether a 67-year-old patient with benign prostatic hyperplasia has an upper urinary tract infection (UTI)? a. Bladder distention b. Foul-smelling urine c. Suprapubic discomfort d. Costovertebral tenderness

ANS: D Costovertebral tenderness is characteristic of pyelonephritis. Bladder distention, foul-smelling urine, and suprapubic discomfort are characteristic of lower UTI and are likely to be present if the patient also has an upper UTI.

When obtaining a focused health history for a patient with possible testicular cancer, the nurse will ask the patient about any history of a. sexually transmitted disease (STD) infection. b. testicular trauma. c. testicular torsion. d. undescended testicles.

ANS: D Cryptorchidism is a risk factor for testicular cancer if it is not corrected before puberty. STD infection, testicular torsion, and testicular trauma are risk factors for other testicular conditions but not for testicular cancer.

A patient who has been recently diagnosed with benign prostatic hyperplasia (BPH) tells the nurse that he does not want to have a transurethral resection of the prostate (TURP) because he is afraid it might affect his ability to have intercourse. Which action should the nurse take? a. Offer reassurance that sperm production is not affected by TURP. b. Discuss alternative methods of sexual expression besides intercourse. c. Provide education about the use of medications for erectile dysfunction (ED) occurring after TURP. d. Teach that ED is not a common complication following a TURP.

ANS: D ED is not a concern with TURP, although retrograde ejaculation is likely and the nurse should discuss this with the patient. Erectile function is not usually affected by a TURP, so the patient will not need information about penile implants or reassurance that other forms of sexual expression may be used. Because the patient has not asked about fertility, reassurance about sperm production does not address his concerns.

A client is going home after outpatient surgery for a hydrocele. Which information does the nurse emphasize in teaching this client? a. "Report to the doctor immediately any drainage from your drain." b. "Use a condom during intercourse to prevent incisional infection." c. "Sit when you urinate until all swelling is gone and drainage has stopped." d. "Wear the scrotal support device for at least 3 weeks after surgery."

ANS: D Edema from residual inflammation can remain for several weeks. This problem is increased if the scrotum is not supported and can cause the client considerable discomfort. The client needs to wear a supportive garment such as a jockstrap during this time. If the client goes home with a drain in place, serosanguineous drainage can be expected for up to 2 days.

A patient has an enlarged prostate detected by digital rectal examination (DRE) and an elevated prostate specific antigen (PSA) level. The nurse will anticipate that the patient will need teaching about a. cystourethroscopy. b. uroflowmetry studies. c. magnetic resonance imaging (MRI). d. transrectal ultrasonography (TRUS).

ANS: D In a patient with an abnormal DRE and elevated PSA, transrectal ultrasound (TRUS) is used to visualize the prostate for biopsy. Uroflowmetry studies help determine the extent of urine blockage and treatment, but there is no indication that this is a problem for this patient. Cystoscopy may be used before prostatectomy but will not be done until after the TRUS and biopsy. MRI is used to determine whether prostatic cancer has metastasized but would not be ordered at this stage of the diagnostic process.

The nurse is performing a psychosocial assessment of a young man diagnosed with testicular cancer. Which does the nurse include as a priority in the assessment? a. Encouraging the client to verbalize his thoughts and feelings to his health care provider b. Assisting the client in locating a support group for men with testicular cancer c. Asking the client to rate his fears of sexual deficiency on a scale of 1 to 10 d. Identifying all components of his support system, including his partner

ANS: D Part of conducting a psychosocial assessment is determining who makes up the client's support system. It would be ineffective merely to refer the client to a support group, ask him to rate his fears of sexual inadequacy, or encourage him to discuss thoughts and feelings not with the nurse, but with his provider.

The client with sickle cell anemia has had an erection for longer than 4 hours. How does the nurse intervene? a. Administer a diuretic to increase urine output. b. Attempt to relieve pressure by catheterizing the client. c. Document the finding and reassess in 4 hours. d. Notify the health care provider and prepare to give meperidine (Demerol).

ANS: D Prolonged penile erection—priapism—is common during sickle cell crisis. It is considered a urologic emergency because circulation to the penis may be compromised, and the client may not be able to void. Therefore, the provider must be notified promptly. Demerol is often given to induce hypotension. A diuretic will not help the client. Catheterization should be reserved for the man who cannot void. Waiting another 4 hours to intervene may lead to ischemia.

A client has decided to treat his enlarged prostate with saw palmetto. Which is the nurse's best response? a. "You'll need to get permission from your health care provider to make that decision." b. "Saw palmetto is a well-respected alternative therapy for benign prostatic hyperplasia." c. "Have you discussed this decision with your family?" d. "What has your health care provider told you about this choice of therapy?"

ANS: D Saw palmetto is an alternative therapy for benign prostatic hyperplasia (BPH) that has not yet been proven to be therapeutic. A client's decision to use this as the primary form of treatment should be discussed with his provider. Some herbs and natural products interfere with the actions of medications taken for other conditions. The other statements do not give the client accurate information to help him make this decision.

To determine the severity of the symptoms for a patient with benign prostatic hyperplasia (BPH) the nurse will ask the patient about a. blood in the urine. b. lower back or hip pain. c. erectile dysfunction (ED). d. strength of the urinary stream.

ANS: D The American Urological Association (AUA) Symptom Index for a patient with BPH asks questions about the force and frequency of urination, nocturia, etc. Blood in the urine, ED, and back or hip pain are not typical symptoms of BPH.

A nurse and an unlicensed assistive personnel (UAP) are caring for a client with an open radical prostatectomy. Which comfort measure could the nurse delegate to the UAP? a. Administering an antispasmodic for bladder spasms b. Managing pain through patient-controlled analgesia c. Applying ice to a swollen scrotum and penis d. Helping the client transfer from the bed to the chair

ANS: D The UAP could aid the client in transferring from the bed to the chair and with ambulation. The nurse would be responsible for medication administration, assessment of swelling, and the application of ice if needed.

After a transurethral resection of the prostate (TURP), a patient with continuous bladder irrigation complains of painful bladder spasms. The nurse observes a decrease in urine output and clots in the urine. Which action should the nurse take first? a. Increase the flow rate of the bladder irrigation. b. Administer the prescribed IV morphine sulfate. c. Give the patient the prescribed belladonna and opium suppository. d. Manually instill and then withdraw 50 mL of saline into the catheter.

ANS: D The assessment suggests that obstruction by a clot is causing the bladder spasms, and the nurse's first action should be to irrigate the catheter manually and to try to remove the clots. IV morphine will not decrease the spasm, although pain may be reduced. Increasing the flow rate of the irrigation will further distend the bladder and may increase spasms. The belladonna and opium suppository will decrease bladder spasms but will not remove the obstructing blood clot.

A client is scheduled for a prostatectomy for benign prostatic hyperplasia (BPH). On the morning of surgery, the laboratory report on the client's urine indicates the presence of red blood cells, white blood cells, and bacteria. Which is the nurse's highest priority action? a. Document the report in the client's chart. b. Insert a new Foley catheter before surgery. c. Strain the client's urine. d. Assess the client's vital signs and notify the health care provider.

ANS: D The client may have a urinary tract infection. The nurse should obtain a set of vital signs and notify the provider of the laboratory results. Any surgery may need to be delayed if the client has infection. Documentation is needed after other actions have been taken. Inserting a catheter and straining the client's urine will not be beneficial.

8. A 37-year-old Nigerian woman is at high risk for breast cancer and is considering a prophylactic mastectomy and oophorectomy. What action by the nurse is most appropriate? a. Discourage this surgery since the woman is still of childbearing age. b. Reassure the client that reconstructive surgery is as easy as breast augmentation. c. Inform the client that this surgery removes all mammary tissue and cancer risk. d. Include support people, such as the male partner, in the decision making.

ANS: D The cultural aspects of decision making need to be considered. In the Nigerian culture, the man often makes the decisions for care of the female. Women with a high risk for breast cancer can consider prophylactic surgery. If reconstructive surgery is considered, the procedure is more complex and will have more complications compared to a breast augmentation. There is a small risk that breast cancer can still develop in the remaining mammary tissue. DIF: Applying/Application REF: 1468

Which intervention helps the client with chronic prostatitis prevent spread of infection to other areas of the urinary tract? a. "Wear a condom during intercourse." b. "Avoid alcohol and caffeinated beverages." c. "Be sure to empty your bladder completely at each voiding." d. "Sexual intercourse or masturbation can help drain the prostate."

ANS: D The prostate is not easily penetrated by antibiotics and can serve as a reservoir for microorganisms, which can infect other areas of the genitourinary tract. Draining the prostate regularly through intercourse or masturbation decreases the number of microorganisms present and reduces the risk for further infection. The other interventions listed will be ineffective with prostatitis.

Which client diagnosed with prostate cancer is not a candidate for watchful waiting? a. Client with very early cancer of the prostate b. Client who is asymptomatic c. Client who wants to avoid urinary incontinence as a result of treatment d. Client who refuses frequent digital rectal examinations (DREs)

ANS: D To participate in watchful waiting, the client must be monitored on a regular basis with a DRE and prostate-specific antigen (PSA) testing. Clients who are asymptomatic, who have early cancer, and who wish to avoid urinary incontinence from treatment would all be excellent candidates for watchful waiting.

Which client statement indicates understanding about post-orchiectomy care for testicular cancer? a. "I will avoid contact sports to prevent injury and development of cancer in my remaining testis." b. "I will always use a condom because I am at increased risk for acquiring a sexually transmitted disease." c. "I will wear an athletic supporter and cup to prevent testicular cancer in my remaining testicle." d. "I will continue to perform testicular self-examination (TSE) monthly on my remaining testicle."

ANS: D Treatment (e.g., surgery, radiation, chemotherapy) for testicular cancer does not protect the person from development of testicular cancer in the remaining testicle. A monthly TSE should be performed to monitor for changes in size, shape, or consistency of the testis. The other statements are inaccurate. Testicular cancer is not caused by trauma, cannot be prevented by an athletic cup, and does not cause increased risk for sexually transmitted diseases.

The RN working in the hospital emergency department is assigned to care for these four clients. Which client does the nurse attend to first?

Adolescent with an erection for "10 or 11 hours" who is reporting severe pain

With which male client does the nurse conduct prostate screening and education?

Adult who is older than 50 years

A 67-year-old client underwent a lumpectomy for a breast lesion that was determined to be malignant. Which of the following are factors in the client's history that may have increased her risk of breast cancer? a) Not giving birth b) Obesity c) Increased age d) All options are correct.

All options are correct. The risk for breast cancer in women increases with age. Certain factors appear to increase the risk of breast cancer. Being female, being older than 50 years of age, and having a family history of breast cancer are the most common risk factors. Additional factors include obesity, and having no children or having children after 30 years of age. The risk for breast cancer in women increases with age. Certain factors appear to increase the risk of breast cancer including obesity and having no children or having children after 30 years of age.

Kara Carpenter is a 54-year-old woman who just had a left radical mastectomy. The nurse caring for her is providing information on complications that may arise due to removing the axillary lymph nodes. Which of the following would not be included? a) Reduced range of motion b) Infection c) Tissue necrosis d) All would be included in the discussion.

All would be included in the discussion. Impaired lymphatic circulation predisposes to disfigurement, reduced range of motion, heaviness of the limb, skin changes, infection, and, in severe cases, tissue necrosis that may require amputation of the limb. Reduced range of motion is a potential consequence from removing the axillary lymph nodes. Tissue necrosis that may require amputation of the limb is a potential consequence from removing the axillary lymph nodes. Infection is a potential consequence from removing the axillary lymph nodes.

A client is undergoing a diagnostic workup for suspected testicular cancer. Which serum tumor marker is indicative of testicular cancer?

Alpha-fetoprotein

A client underwent a transurethral resection of the prostate (TURP) yesterday for benign prostatic hyperplasia (BPH). What intervention does the nurse perform for the client during the first postoperative day?

Assess for excessive bleeding.

A client who had an open orchiectomy with a radical retroperitoneal lymph node dissection yesterday reports abdominal distention and discomfort today. What is the nurse's priority action?

Assess the client's bowel sounds.

A client returns to the recovery room following a mastectomy. An initial postoperative assessment is performed by the nurse. What is the nurse's priority assessment? a) Checking level of pain first upon the clients return from the operating room. b) Assessing the vital signs and oxygen saturation levels. c) Checking for urinary retention and the need to void. d) Checking the dressing, drain, and amount of drainage.

Assessing the vital signs and oxygen saturation levels. The nurse prioritizes vital signs and breathing based on principles of ABCs.

A female client is diagnosed with breast abscess. She would like to continue to breast-feed her newborn. Which of the following would be most appropriate in this situation? a) Reduce the frequency of removing and reapplying the dressings. b) Assist the client to pump the breasts to remove breast milk. c) Instruct the client to wear a tight-fitting bra. d) Encourage the client to include protein content in the diet.

Assist the client to pump the breasts to remove breast milk. The nurse should help the client pump the breasts and remove breast milk to prevent engorgement. Because the client has decided to continue breastfeeding, the client should wear a loose-fitting bra. Including protein content in the diet would be unrelated to the client's current situation. Frequency of dressing changes does not play a role in the intervention

9. The nurse understands which of the following is a possible complication of cryotherapy used to treat cancer of the prostate? A. Bone marrow depression. B. Urethral damage causing urinary incontinence. C. Massive blood loss due to severing of blood vessels. D. Chronic vomiting and diarrhea causing electrolyte imbalance.

B. Urethral damage causing urinary incontinence.

7. The nurse caring for a patient diagnosed with benign prostatic hyperplasia, who is being treated with the 5 alpha reductase inhibitor, finasteride (Proscar), knows teaching has been understood when the patient makes which of the following comments? A. "I should begin to see results in about two weeks." B. "I cannot allow my pregnant daughter to touch the pills." C. "My wife and I should not have sexual relations as long as I am on this drug." D. "I will need an increase in the usual dose because I am also taking the herbal saw palmetto."

B. "I cannot allow my pregnant daughter to touch the pills."

A male patient complains of fever, dysuria, and cloudy urine. What additional information may indicate that these manifestations may be something other than a urinary tract infection (UTI)? A. E. coli bacteria in his urine B. A very tender prostate gland C. Complaints of chills and rectal pain D. Complaints of urgency and frequency

B. A very tender prostate gland

A 73-year-old male patient admitted for total knee replacement states during the health history interview that he has no problems with urinary elimination except that the "stream is less than it used to be." The nurse should give the patient anticipatory guidance that what condition may be developing? A. A tumor of the prostate B. Benign prostatic hyperplasia C. Bladder atony because of age D. Age-related altered innervation of the bladder

B. Benign prostatic hyperplasia

20. Which intervention would be most beneficial in resolving the nursing diagnosis of Disturbed Body Image for the patient recovering from a mastectomy? A. Refer the patient to a physical therapist. B. Encourage the patient to attend a breast cancer support group. C. Have the social worker visit the patient and the patient's family. D. Force the patient to look at the incision with the first dressing change.

B. Encourage the patient to attend a breast cancer support group.

In which situations would androgens be prescribed for women? (SATA) A. Development of secondary sex characteristics. B. Fibrocystic breast disease. C. Ovarian cancer D. Treatment of endometriosis E. Postmenopausal osteoporosis prevention F. Inoperable breast cancer.

B. Fibrocystic breast disease. D. Treatment of endometriosis F. Inoperable breast cancer.

13. The nurse is teaching a community course on female gynecologic disorders. The nurse teaches information on vaccines available to protect against the most common types of human papillomavirus (HPV) placing females at increased risk for cervical cancer. Which of the following represents correct information related to these vaccines? A. Both Gardasil and Cervarix protect against four types of HPV. B. Gardasil vaccination is given in three IM injections over a 6-month period. C. Vaccination is recommended for all females between the ages of 10 and 50. D. Both Gardasil and Cervarix are effective in treating recurrent HPV infection.

B. Gardasil vaccination is given in three IM injections over a 6-month period.

Which is a pharmacodynamic effect of exogenous androgens? A. Decreased erythropoiesis B. Increased protein synthesis C. Increased nitrogen excretion D. Decreased sperm production

B. Increased protein synthesis Androgens retard the breakdown of amino acids, contributing to an increased synthesis of body proteins, which aids in the formation and maintenance of muscle tissue. Additionally, they stimulate the production of erythropoietin by the kidney, resulting in enhanced erythropoiesis (red blood cell synthesis); increase the retention of nitrogen; and may suppress sperm production when given in large doses as a result of the feedback inhibition of pituitary follicle-stimulating hormone (FSH).

19. A patient has just returned to the surgical unit following a modified radical mastectomy. During this early postoperative period the nurse plans to teach the patient which of the following? A. Maintain high-Fowler's position. B. Keep the arm in an elevated position. C. Observe the incision site for redness and bleeding. D. Perform range-of-motion exercises including flexion, extension, and abduction of the affected arm.

B. Keep the arm in an elevated position.

A 19-year-old college football player asked the nurse about taking steroids to help him "beef up" his muscles. Which statement is true? A. There should be no problem as long as he does not exceed the recommended dosage. B. Long-term use may cause a life-threatening liver condition. C. He would need to be careful to watch for excessive weight loss. D. These drugs also tend to increase the male's sperm count

B. Long-term use may cause a life-threatening liver condition.

A patient has been taking finasteride (Proscar) for almost a year. The nurse knows that which is most important to evaluate at this time? A. CBC B. PSA levels C. BP D. Fluid retention

B. PSA levels

When the Testoderm form of testosterone is ordered to treat hypogonadism in a teenage boy, which instruction by the nurse are correct? (SATA) A. Place the patch on clean, dry skin on the back, upper arms, abdomen or thighs. B. Place the patch on clean, dry scrotal skin that has been shaved. C. Place the patch on clean, dry scrotal skin, but do not shave the skin first. D. Place the patch on any clean, dry, nonhairy area of the body. E. Remove the old patch before applying a new patch.

B. Place the patch on clean, dry scrotal skin that has been shaved. E. Remove the old patch before applying a new patch.

The nurse is educating a patient about medications used to treat erectile dysfunction. Which erectile dysfunction medications have the longest therapeutic effect when taken orally? (Select all that apply.) A. sildenafil (Viagra) B. vardenafil (Levitra) C. tadalafil (Cialis) D. alprostadil (Caverject)

B. vardenafil (Levitra) C. tadalafil (Cialis) Phosphodiesterase inhibitors are used in the treatment of erectile dysfunction. Sildenafil (Viagra) was the first oral drug approved for the treatment of erectile dysfunction. Two drugs that are similar but have a longer duration of action are vardenafil (Levitra) and tadalafil (Cialis). Collectively, these drugs are referred to as erectile dysfunction drugs. ; A second type of drug used to treat erectile dysfunction is the prostaglandin alprostadil (Caverject). This drug must be given by injecting it directly into the erectile tissue of the penis or pushing a suppository form of the drug into the urethra

When is the best time for the nurse to begin discharge planning and a community-based plan of care for a client with prostate cancer?

Before surgery

A 65-year-old client with benign prostatic hyperplasia (BPH) is prescribed finasteride (Proscar) and doxazosin (Cardura). What is the priority nursing assessment to perform following the first dose?

Blood pressure in different positions

Which of the following is considered a diagnostic tool for breast cancer? a) Clinical breast exam b) Ultrasonography c) Mammography d) Breast biopsy

Breast biopsy Correct Explanation: Breast biopsy is a tool used to diagnose breast cancer. Ultrasonography, mammography, and clinical breast exam are all tools used to screen for breast cancer.

After teaching a group of students about the signs and symptoms of breast cancer, the instructor determines that additional teaching is needed when the group identifies which of the following? a) Painless mass b) Peau d'orange skin c) Nipple retraction d) Breast symmetry

Breast symmetry The primary sign of breast cancer is a painless mass in the breast. Other signs of breast cancer include a bloody discharge from the nipple, a dimpling of the skin over the lesion, retraction of the nipple, peau d'orange (orange peel) appearance of the skin, and a difference in size between the breasts.

After teaching a group of students about the signs and symptoms of breast cancer, the instructor determines that additional teaching is needed when the group identifies which of the following? a) Breast symmetry b) Peau d'orange skin c) Painless mass d) Nipple retraction

Breast symmetry Correct Explanation: The primary sign of breast cancer is a painless mass in the breast. Other signs of breast cancer include a bloody discharge from the nipple, a dimpling of the skin over the lesion, retraction of the nipple, peau d'orange (orange peel) appearance of the skin, and a difference in size between the breasts.

A patient is asking about the use of saw palmetto for prostate health. The nurse tells him that drugs that interact with saw palmetto include: A. acetaminophen (Tylenol) B. nitrates C. nonsteroidal anti-inflammatory drugs D. antihypertensive drugs

C. nonsteroidal anti-inflammatory drugs

5. The nurse is caring for a male patient diagnosed with benign prostatic hyperplasia (BPH). Which statement made by the patient indicates the need for further teaching? A. "The enlarged prostate gland may produce blood in my urine." B. "I can get urinary tract infections because of the enlarged prostate gland." C. "I should cut down on the fluids I drink so I won't have to urinate so often." D. "The enlarged prostate gland causes me to get up three times every night to urinate."

C. "I should cut down on the fluids I drink so I won't have to urinate so often."

15. The nurse is aware which patient comment best correlates with a diagnosis of endometrial cancer? A. "My periods are scant." B. "My periods are heavy and I have cramps." C. "I started bleeding again after being in menopause for two years." D. "I am going through the change (perimenopause) and my periods are irregular."

C. "I started bleeding again after being in menopause for two years."

To accurately monitor progression of a symptom of decreased urinary stream, the nurse should encourage the patient to have which primary screening measure done on a regular basis? A. Uroflowmetry B. Transrectal ultrasound C. Digital rectal examination (DRE) D. Prostate-specific antigen (PSA) monitoring

C. Digital rectal examination (DRE)

What common side effect would the nurse include in the discharge teaching for a patient receiving finasteride (Proscar)? A. Hair loss B. Increased libido C. Ejaculatory dysfunction D. Muscle weakness

C. Ejaculatory dysfunction Common side effects of finasteride include impotence, decreased libido, and decreased volume of ejaculate. It is also used to treat male pattern baldness and thus would cause hair growth, not hair loss.

The nurse would question the administration of testosterone to a patient with which condition? A. Hypocalcemia B. Hyponatremia C. Hyperkalemia D. Hypovolemia

C. Hyperkalemia Testosterone can cause increased serum potassium levels

The nurse is providing education to a patient and his caregiver—his pregnant daughter—about dutasteride (Avodart), which he will be taking for benign prostatic hyperplasia (BPH). What important teaching would the nurse provide to the patient and his daughter about the administration of this medication? A. It should be taken with food containing dairy. B. It may be placed in daily medication container with other medications. C. It must not be touched or handled by his daughter due to teratogenic effects. D. For best results, it needs to be taken at hour of sleep on empty stomach.

C. It must not be touched or handled by his daughter due to teratogenic effects Finasteride (Proscar) and dutasteride (Avodart) are indicated for BPH. Both drugs are contraindicated in patients who have shown hypersensitivity and in pregnant women and children. It is considered potentially dangerous for a pregnant woman even to handle crushed or broken tablets. Both drugs are classified as pregnancy category X.

The nurse is performing an assessment of a patient who is asking for a prescription for sildenafil (Viagra). Which finding would be a contraindication to its use? A. Age of 65 years B. History of thyroid disease C. Medication list that includes nitrates D. Medication list that includes saw palmetto

C. Medication list that includes nitrates

The patient has had cardiovascular disease for some time and has now developed erectile dysfunction. He is frustrated because he cannot take erectogenic medications because he takes nitrates for his cardiac disease. What should the nurse do first to help this patient? A. Give the patient choices for penile implant surgery. B. Recommend counseling for the patient and his partner. C. Obtain a thorough sexual, health, and psychosocial history. D. Assess levels of testosterone, prolactin, LH, and thyroid hormones.

C. Obtain a thorough sexual, health, and psychosocial history.

When teaching a patient regarding proper application of a Testoderm transdermal patch, the nurse will instruct the patient to perform which action? A. Replace the patch every 72 hours. B. Replace the patch every 36 hours. C. Place the patch on clean, dry, shaved scrotal skin. D. Place the patch on the back, abdomen, upper arms, or thigh

C. Place the patch on clean, dry, shaved scrotal skin Testoderm transdermal patches should be placed on clean, dry scrotal skin that has been shaved for optimal skin contact. A patch should be replaced every 24 hours.

The patient has a low-grade carcinoma on the left lateral aspect of the prostate gland and has been on "watchful waiting" status for 5 years. Six months ago his last prostate-specific antigen (PSA) level was 5 ng/mL. Which manifestations now indicate that the prostate cancer may be growing and he needs a change in his care (select all that apply)? A. Casts in his urine B. Presence of α-fetoprotein C. Serum PSA level 10 ng/mL D. Onset of erectile dysfunction E. Nodularity of the prostate gland

C. Serum PSA level 10 ng/mL E. Nodularity of the prostate gland

A patient is one day postoperative following a transurethral resection of the prostate (TURP). Which event is not an expected normal finding in the care of this patient? A. The patient requires two tablets of Tylenol #3 during the night. B. The patient complains of fatigue and claims to have minimal appetite. C. The patient has continuous bladder irrigation (CBI) infusing, but output has decreased. D. The patient has expressed anxiety about his planned discharge home the following day.

C. The patient has continuous bladder irrigation (CBI) infusing, but output has decreased

The patient is taking finasteride (Proscar) for the treatment of benign prostatic hyperplasia. His wife, which is 3 months pregnant, is worried about the adverse effects that may occur with this drug. Which statement by the nurse is most important at this time? A. Gastric upset may be reduced if he takes this drug on an empty stomach. B. He should notice therapeutic effects of increased libido an erection within one month. C. This medication should not even be handled by pregnant women because it may harm fetus. D. He may experience transient hair loss while taking this medication.

C. This medication should not even be handled by pregnant women because it may harm the fetus.

A 33-year-old female patient with three children has had a follow-up mammogram following an abnormal BSE. Mammogram findings reveal an incidental microscopic abnormal tissue growth in the left breast lobules. The physician orders tamoxifen (Soltamox) for the patient. The nurse understands that the physician is implementing which of the following primary prevention modalities to treat this patient? a) Chemoprevention b) Radiation therapy c) Prophylactic mastectomy d) Long-term surveillance

Chemoprevention Correct Explanation: Chemoprevention is a primary prevention modality that aims at preventing the disease before it starts.

Which of the following is a primary prevention modality that aims at preventing the disease before it starts? a) Prophylactic mastectomy b) Chemoprevention

Chemoprevention Explanation: Chemoprevention is a primary prevention modality that aims at preventing the disease before it starts

The nurse is caring for a client with chronic prostatitis. What instructions does the nurse provide?

Chronic prostatitis is not contagious.

The nurse has identified the following nursing diagnosis on the plan of care for a client who has undergone breast cancer surgery: Disturbed sensory perception related to nerve irritation in the affected breast area. Which of the following indicates that the outcome has been achieved? a) Client identifies appropriate measures to reduce the risk of lymphedema. b) Client demonstrates appropriate use of prescribed analgesic. c) Client reports a sensation of pulling in the breast area. d) Client states that feeling in her breast area will gradually subside with time.

Client states that feeling in her breast area will gradually subside with time. Correct Explanation: After breast surgery, the nerves in the skin and axilla are often cut or injured, leading the client to experience various sensations, including tenderness, soreness, numbness, tightness, pulling, and twinges. These sensations are normal and usually persist for several months and then subside. Therefore, acknowledging that the feeling in her breast area will gradually subside indicates that she understands why the sensation is occurring and that it is normal. The report of the sensation of pulling indicates that the client is feeling the sensation, but it does not indicate whether the nursing diagnosis was addressed. Analgesics usually would not be required for these sensations. These sensations are not associated with lymphedema.

A group of students is reviewing information about breast cancer and metastasis in preparation for an examination. The students demonstrate the need for additional study when they identify which of the following as a common site for metastasis? a) colon b) liver

Colon Explanation: Breast cancer typically does not metastasize to the colon. The four major sites of metastasis include the skeletal and pulmonary systems, brain, and liver.

A 43-year-old client is a single parent and has been admitted for a left mastectomy after confirmation of cancer from a node biopsy. She has a daughter who is 12 years old. What are primary issues for the nurse to discuss with this client? a) Effect of surgery on the family's coping abilities. b) History of breast cancer in the family. c) How body image changes will affect her sexual relationships. d) Concerns regarding the cancer and how the surgery will affect her.

Concerns regarding the cancer and how the surgery will affect her. The two primary concerns are the confirmation of cancer and the impending mastectomy. The other issues are important, but not as high a priority at this time.

The nurse working on a woman's cancer treatment floor performs nursing assessments on her assigned patients. It is most important for the nurse to report which of the following assessment findings? a) Immediate postoperative reports of throat tenderness b) Coolness and mottling of a newly constructed breast site c) Temperature of 99.2°F, pulse 72, respirations 18, blood pressure 130/80 d) Small amount of bloody drainage on surgical dressing 12 hours postoperatively

Coolness and mottling of a newly constructed breast site Correct Explanation: Mottling or an obvious decrease in skin temperature may signify flap loss and needs to be reported to the surgeon immediately. Throat discomfort immediately following surgery is an expected effect of airway management during surgery. A small amount of bloody drainage is an expected finding 12 hours postoperatively. Vital signs are within acceptable range for a postoperative patient.

A patient scheduled for a transurethral resection of the prostate (TURP) for BPH tells the nurse that he has delayed having surgery because he is afraid it will affect his sexual function. When responding to his concern, the nurse explains that a. with this type of surgery, erectile problems are rare, but retrograde ejaculation may occur. b. information about penile implants used for ED is available if he is interested. c. there are many methods of sexual expression that can be alternatives to sexual intercourse. d. sterility will not be a problem after surgery because sperm production will not be affected.

Correct Answer: A Rationale: Erectile problems are rare, but retrograde ejaculation may occur after TURP. Erectile function is not usually affected by a TURP, so the patient will not need information about penile implants or reassurance that other forms of sexual expression may be used. Because the patient has not asked about fertility, reassurance about sperm production does not address his concerns.

A 32-year-old man scheduled for a unilateral orchiectomy for testicular cancer is admitted to the hospital the morning of surgery. He is accompanied by his wife but does not talk to her and does not initiate interaction with the nurse. The most appropriate action by the nurse is to a. ask the patient if he has any questions or concerns about the diagnosis and treatment. b. tell the patient's wife that concerns about sexual function are common with this diagnosis. c. teach the patient that impotence is rarely a problem after unilateral orchiectomy. d. document the patient's lack of communication on the chart and continue preoperative care.

Correct Answer: A Rationale: The initial action by the nurse should be assessment for any anxiety or questions about the surgery or postoperative care. The nurse should address the patient, not the spouse, when discussing the diagnosis and any possible concerns. Without further assessment of patient concerns, it is inappropriate for the nurse to initiate teaching. It would be inappropriate for the nurse to provide patient teaching without further assessment of the patient's teaching needs and concerns. Documentation of the patient's lack of interaction is not an adequate nursing action in this situation.

A patient with symptomatic BPH is scheduled for visual laser ablation of the prostate (VLAP) at an outpatient surgical center. The nurse will plan to teach the patient a. how to care for an indwelling urinary catheter. b. that the urine will appear bloody for several days. c. to expect an immediate improvement in urinary force. d. that an intraprostatic urethral stent will be placed.

Correct Answer: A Rationale: The patient will have indwelling catheter for up to a week and will need to be instructed on catheter care to avoid problems such as infection. There is minimal bleeding with this procedure. It will take several weeks before the full benefits of the procedure take effect. Stent placement is not included in the procedure.

Leuprolide (Lupron), an LH-RH Agonist, and bicalutamide (Casodex), an androgen receptor blocker, are prescribed for a patient with cancer of the prostate. In teaching the patient about these drugs, the nurse informs the patient that side effects may include a. low blood pressure. b. decreased sexual drive. c. urinary incontinence. d. frequent infections.

Correct Answer: B Rationale: Hormonal therapy blocks the effects of testosterone and decreases libido. Hypotension is associated with the -blockers used for BPH. Urinary incontinence may occur after prostate surgery, but it is not an expected medication side effect. Risk for infection is increased in patients receiving chemotherapy.

Following discharge teaching for a patient who has had a transurethral prostatectomy for benign prostatic hyperplasia (BPH), the nurse determines that additional instruction is needed when the patient says, a. "I will increase fiber and fluids in my diet to prevent constipation." b. "I should call the doctor if I have any incontinence at home." c. "I will avoid heavy lifting or driving until I get approval from my health care provider." d. "I should continue to schedule yearly appointments for prostate exams."

Correct Answer: B Rationale: Incontinence is common for several weeks after a TURP. The other patient statements indicate that the patient has a good understanding of post-TURP instructions.

The wife of a patient who has undergone a TURP and has continuous bladder irrigation asks the nurse about the purpose of the continuous bladder irrigation. Which response by the nurse is appropriate? a. "The bladder irrigation is needed to stop the postoperative bleeding in the bladder." b. "The irrigation is needed to keep the catheter from being occluded by blood clots." c. "Normal production of urine is maintained with the irrigations until healing occurs." d. "Antibiotics are being administered into the bladder with the irrigation solution."

Correct Answer: B Rationale: The purpose of bladder irrigation is to remove clots from the bladder and to prevent obstruction of the catheter by clots. The irrigation does not decrease bleeding or maintain urine production. Antibiotics are given by the IV route, not through the bladder irrigation.

A 41-year-old man asks the nurse what he can do to decrease the risk of BPH. The nurse explains that a. riding a bicycle raises prostate specific antigen levels and may increase BPH risk. b. prevention is not possible because prostatic enlargement occurs with normal aging. c. decreasing butter and margarine and increasing fruits in the diet may help. d. taking a daily vitamin E supplement has reduced prostate size in some men.

Correct Answer: C Rationale: A diet high in saturated fats, found in foods like butter, is associated with an increased risk for BPH. Individuals who eat more fruits and vegetables may be at lower risk. Riding a bicycle does increase prostate-specific antigen (PSA) levels, but this is not associated with development of BPH. Dietary changes and increased exercise do appear to help prevent BPH. Vitamin E supplements do not decrease prostate size.

When taking a nursing history from a patient with BPH, the nurse would expect the patient to report a. nocturia, dysuria, and bladder spasms. b. urinary frequency, hematuria, and perineal pain. c. urinary hesitancy, postvoid dribbling, and weak urinary stream. d. urinary urgency with a forceful urinary stream and cloudy urine.

Correct Answer: C Rationale: Classic symptoms of uncomplicated BPH are those associated with urinary obstruction and include diminished caliber and force of the urinary stream, hesitancy, difficulty initiating voiding, intermittent urination, dribbling at the end of urination, and a feeling of incomplete bladder emptying because of urinary retention. Irritative symptoms, including nocturia, frequency, dysuria, urgency, or hematuria, occur if infection results from urinary retention.

To determine the severity of the symptoms for a patient with benign prostatic hyperplasia (BPH), the nurse will ask the patient about a. the presence of blood in the urine. b. any erectile dysfunction (ED). c. occurrence of a weak urinary stream. d. lower back and hip pain.

Correct Answer: C Rationale: The American Urological Association (AUA) Symptom Index for a patient with BPH asks questions about the force and frequency of urination, nocturia, etc. Blood in the urine, ED, and back or hip pain are not typical symptoms with BPH.

A 64-year-old man undergoes a perineal radical prostatectomy for stage C prostatic cancer. Postoperatively, the nurse establishes the nursing diagnosis of risk for infection related to a. urinary stasis. b. urinary incontinence. c. possible fecal contamination of the surgical wound. d. placement of a suprapubic catheter into the bladder.

Correct Answer: C Rationale: The perineal approach increases the risk for infection because the incision is located close to the anus and contamination with feces is possible. Urinary stasis and incontinence do not occur because the patient has a retention catheter in place for 1 to 2 weeks. A urethral catheter is used after the surgery.

On admission to the ambulatory surgical center, a patient with BPH informs the nurse that he is going to have a laser treatment of his enlarged prostate. The nurse plans patient teaching with the knowledge that the patient will need a. monitoring for postoperative urinary retention. b. teaching about the effects of general anesthesia. c. to be informed of the possibility of short-term incontinence. d. instruction about home management of an indwelling catheter.

Correct Answer: D Rationale: Because of edema, urinary retention, and delayed sloughing of tissue that occurs with a laser prostatectomy, the patient will have postprocedure catheterization for up to 7 days. The procedure is done under local anesthetic, and incontinence is not usually a problem.

The health care provider orders a blood test for prostate-specific antigen (PSA) when an enlarged prostate is palpated during a routine examination of a 56-year-old man. When the patient asks the nurse the purpose of the test, the nurse's response is based on the knowledge that a. elevated levels of PSA are indicative of metastatic cancer of the prostate. b. PSA testing is the "gold standard" for making a diagnosis of prostate cancer. c. baseline PSA levels are necessary to determine whether treatment is effective. d. PSA levels are usually elevated in patients with cancer of the prostate.

Correct Answer: D Rationale: PSA levels are usually elevated above the normal in patients with prostate cancer. PSA testing does not determine whether metastasis has occurred. A biopsy of the prostate is needed for a definitive diagnosis of prostate cancer. Success of treatment is determined by a fall in PSA to an undetectable level; the patient's baseline PSA is not needed to determine the success of treatment.

A patient undergoing a TURP returns from surgery with a three-way urinary catheter with continuous bladder irrigation in place. The nurse observes that the urine output has decreased and the urine is clear red with multiple clots. The patient is complaining of painful bladder spasms. The most appropriate action by the nurse is to a. administer the ordered IV morphine sulfate, 4 mg. b. increase the flow rate of the continuous bladder irrigation. c. give the ordered the belladonna and opium suppository. d. manually instill 50 ml of saline and try to remove the clots.

Correct Answer: D Rationale: The assessment suggests that obstruction by a clot is causing the bladder spasms, and the nurse's first action should be to irrigate the catheter manually and to try to remove the clots. IV morphine will not decrease the spasm, although pain may be reduced. Increasing the flow rate of the irrigation will further distend the bladder and may increase spasms. The belladonna and opium suppository will decrease bladder spasms but will not remove the obstructing blood clot.

A 32-year-old client is concerned with the lumps that have developed in her breasts and is fearful of cancer. She reports variability in the size of the lumps. What could be causing her condition? a) Cyclical hormonal changes b) Progesterone c) Caffeine d) Nicotine

Cyclical hormonal changes Correct Explanation: Fibrocystic disease results from hormonal changes during the menstrual cycle. The likely cause is fibrocystic disease, which results from hormonal changes during the menstrual cycle.

18. The nurse is performing a breast examination on a female patient. Which finding should the nurse report as suspicious of breast cancer to the primary healthcare provider? A. A single soft lump that is well-defined and tender. B. A soft, mobile, singular, lobular nodule that is non-tender. C. Multiple, bilateral, round, lumpy tissues that are tender to palpation. D. An irregularly shaped, hard lump that is non-tender and non-mobile.

D. An irregularly shaped, hard lump that is non-tender and non-mobile.

16. The nurse practitioner (NP) is evaluating a patient for ovarian cancer risk even though the pelvic exam does not reveal any abnormalities. The NP determines the patient is at high risk for ovarian cancer. Which tests should the NP order to aid in the diagnosis of this cancer? A. CBC and OVA1. B. BRCA1 and HPV. C. Pap smear and CEA. D. CA-125 and abdominal ultrasound.

D. CA-125 and abdominal ultrasound.

4. The nurse is taking a health history of a male patient who has a history of benign prostatic hyperplasia (BPH). To determine whether the patient currently experiences difficulty, for which manifestation would the nurse ask the patient? A. Polyuria. B. Hematuria. C. Erectile dysfunction. D. Decreased force in the stream of urine.

D. Decreased force in the stream of urine.

10. Postoperatively, a patient who had a transurethral resection of the prostate has continuous bladder irrigation with a three-way Foley catheter with a 30 mL balloon and traction applied reports the urge to void even with the catheter in place. The nurse should implement which of the following? A. Vigorously hand-irrigate the catheter to ensure it is patent. B. Deflate the catheter balloon to 10 mL to decrease bulk in the bladder. C. Encourage the patient to have a bowel movement to relieve colon pressure. D. Explain this is a normal feeling and he should not try to urinate around the catheter.

D. Explain this is a normal feeling and he should not try to urinate around the catheter.

The patient has asked for a new prescription of sildenafil (Viagra), and erectile dysfunction drug. As the nurse reviews his current medications, which drug, if taken by the patient, would cause a significant interaction? A. Warfarin (Coumadin), an oral anticoagulant B. Amoxicillin (Amoxil), and antibiotic C. Esomeprazole (Nexium), a proton pump inhibitor. D. Isosorbide dinitrate (Isordil), a nitrate

D. Isosorbide dinitrate (Isordil), a nitrate

Which task can the nurse delegate to an unlicensed assistive personnel (UAP) in the care of a patient who has recently undergone prostatectomy? A. Assessing the patient's incision B. Irrigating the patient's Foley catheter C. Assessing the patient's pain and selecting analgesia D. Performing cleansing of the meatus and perineal region

D. Performing cleansing of the meatus and perineal region

Before a patient begins therapy with finasteride (Proscar), the nurse should make sure that which laboratory test has been performed? A. Blood glucose level. B. Complete blood count C. Urinalysis D. Prostate-specific antigen level (PSA)

D. Prostate specific antigen (PSA) level.

3. The nurse would provide which instructions to a male patient when teaching him how to perform a testicular self-examination? A. Perform the exam immediately after awakening. B. Gently feel each testicle with one finger for abnormal growths. C. A testicular exam should be performed once every six (6) months. D. The best time to examine the testicles is during or immediately after a warm shower.

D. The best time to examine the testicles is during or immediately after a warm shower.

The nurse is assessing a patient with suspected testicular cancer. Which assessment data would support a diagnosis of testicular cancer? A. A red, raised rash on the scrotum. B. The patient reports pain when urinating. C. A chancre sore on the shaft of the penis. D. The patient reports heaviness in the scrotum.

D. The patient reports heaviness in the scrotum.

6. The nurse observes red urine and several clots in the urinary catheter tubing of a patient receiving a normal saline continuous irrigation who has just begun ambulating one day postoperative following a transurethral resection of the prostate for benign prostatic hyperplasia. Which intervention should the nurse implement? A. Administer vitamin K slowly. B. Remove the indwelling catheter. C. Administer protamine sulfate IV push. D. Titrate the 0.9% saline (normal saline) irrigation to run faster.

D. Titrate the 0.9% saline (normal saline) irrigation to run faster.

The nurse is teaching a patient about the possible adverse effect of priapism, which may occur when taking erectile dysfunction drugs. The nurse emphasizes that, if this occurs the most important action is to..... A. stay in bed till the erection ceases B. apply an ice pack for 30 min C. turn toward his left side and rest D. seek medical attention immediately

D. seek medical attention immediately

A 37-year-old client has been diagnosed with breast cancer and is awaiting the cytology results of her biopsy. During your client education session, you discuss the possible types of breast malignancies. Which is the most common type? a) Medullary b) Infiltrating lobular c) Inflammatory d) Ductal

Ductal Explanation: The most common malignancy is ductal carcinoma (80%); followed by infiltrating lobular carcinoma (10%); medullary carcinoma, mucinous carcinoma, and tubular ductal carcinoma; and inflammatory breast cancer, the rarest but most aggressive form of breast cancer. The most common malignancy is ductal carcinoma (80%); inflammatory breast cancer (1% to 3%) is the rarest but most aggressive form of breast cancer. The most common malignancy is ductal carcinoma (80%). The most common malignancy is ductal carcinoma (80%) followed by infiltrating lobular carcinoma (10%).

A young adult with testicular cancer is admitted for unilateral orchiectomy and retroperitoneal lymph node dissection. Which nursing action is best for the nurse to delegate to unlicensed assistive personnel (UAP)?

Encourage the client to cough and deep-breathe after surgery.

The nurse is caring for a client after an open radical prostatectomy. What action does the nurse take?

Encourage the client to use patient-controlled analgesia (PCA).

A client has undergone a transverse rectus abdominis myocutaneous (TRAM) flap procedure for breast reconstruction immediately following a mastectomy. Which of the following would be most appropriate to include in the client's postoperative plan of care? a) Inspecting the breast site for expected mottling b) Maintaining the client in the supine position c) Monitoring the single incisional site at the breast d) Encouraging coughing and deep breathing exercises

Encouraging coughing and deep breathing exercises Explanation: A client who has undergone a TRAM flap procedure has incisions at both the mastectomy and abdominal donor sites. In addition, the breathing and leg exercises are essential because the client is more limited in her activity and is at greater risk for respiratory complications and deep vein thrombosis. Mottling at the newly created breast site must be reported to the surgeon immediately. Elevating the head of the bed 45 degrees and flexing the client's knees help to reduce tension on the abdominal incision during the first postoperative week.

Which of the following is a characteristic of a breast cancer mass? a) Firm, hard, embedded in surrounding tissue b) Occurs as disseminated masses c) Symmetrical mass d) Tender upon palpation

Firm, hard, embedded in surrounding tissue Correct Explanation: A characteristic of a breast cancer mass is a firm, hard, embedded lesion in surrounding tissue. It has an irregular shape, usually not tender, and occurs as a single mass in one breast.

A nurse is reviewing a client's history for possible risk factors associated with breast cancer. Which of the following would the nurse identify as increasing the client's risk? a) Menopause after age 50 b) Menarche at age 14 c) First full-term pregnancy at age 34.

First full-term pregnancy at age 34 years Correct Explanation: Risk factors associated with breast cancer include menarche before age 12 years, menopause after age 55 years, nulliparity, and late age at first full-term pregnancy.

A 30-year-old client whose mother died of breast cancer at age 44 and whose sister has ovarian cancer, is concerned about developing cancer. As a member of the oncology multidisciplinary team, the nurse should suggest that the client ask the physician about which topic? a) Mammogram b) Genetic counseling c) Pap testing d) Contacting the American Cancer Society

Genetic counseling Explanation: The nurse should suggest that the client ask the physician about genetic counseling. Genetic counseling is indicated for those at high risk because of family or personal cancer history. Genetic counseling involves obtaining a detailed medical and three-generational family history; calculating a personalized risk assessment; providing options for prevention, surveillance, and genetic testing; coordinating and interpreting genetic testing; and developing a management plan based on the test results. Mammography will assist with early detection of most breast cancers, but it won't establish a risk assessment and provide options for prevention, surveillance, and genetic testing. Pap testing every 6 months assists in early detection of most cervical cancers, but it won't establish a risk assessment. Contacting the American Cancer Society will provide the client with information about cancer but the organization won't help assess the client's risk for developing cancer.

What is the usual presenting symptom for testicular cancer?

Hard, painless mass

What term is used for a cystic mass that forms around the testis and is usually filled with straw-colored fluid?

Hydrocele

The nurse is caring for a client who is beginning doxorubicin (Adriamycin) therapy for breast cancer. When preparing the client for probable side effects, which would the nurse include? a) Information regarding wigs from the American Cancer Society b) Information regarding depression from a mental health association c) Information regarding high caloric meals from a dietician d) Information about blood donation from the American Red Cross

Information regarding wigs from the American Cancer Society Alopecia is a common side effect from the use of doxorubicin (Adriamycin). It is best for the client to be proactive in planning for hair loss so that the client has a suitable plan. Also, the American Cancer Society in some areas offers financial support and guidance in obtaining a wig or head covering. Blood donation is not completed during chemotherapy because anemia is common. Due to recent antiemetic medications, nausea is less common not as prolonged. Client's rarely need a high-calorie diet. Emphasis is placed on nutrient dense, not calorie dense. Depression may be a concern, and a support group is an excellent resource. A mental health association may be more than what is needed.

When the female patient demonstrates a wartlike growth near the nipple, causing bloody nipple discharge, the patient is exhibiting signs of which disease process? a) Intraductal papilloma b) Fibroadenoma c) Paget's disease d) Acute mastitis

Intraductal papilloma Correct Explanation: Intraductal papilloma is a wartlike growth that often involves the large milk ducts near the nipple, causing bloody nipple discharge. Surgery usually involves removal of the papilloma and a segment of the duct where the papilloma is found. Paget's disease is a malignancy of mammary ducts with early signs of erythema of nipple and areola. Acute mastitis is demonstrated by nipple cracks or abrasions along with reddened and warm breast skin and tenderness. Fibroadenoma is characterized as the occurrence of a single, nontender mass that is firm, mobile, and not fixed to breast tissue or chest wall.

A client had a transurethral resection of the prostate (TURP) yesterday. The staff nurse notes that the urinary output from the catheter has decreased and the client is reporting severe bladder spasms. What is the nurse's priority action?

Irrigate the catheter with normal saline.

A client is having a radical prostatectomy. Which preoperative teaching specific to this surgery does the nurse emphasize?

Kegel exercises

The nurse is providing care to a client who has had surgery as treatment for breast cancer. The nurse would be alert for the development of which of the following? a) Breast abscess b) Fibroadenoma c) Lymphedema d) Fibrocystic breast disease

Lymphedema Correct Explanation: Lymphedema occurs in some women after breast cancer surgery. It causes disfigurement and increases the lifetime potential for infection and poor healing. Fibrocystic breast disease and fibroadenoma are two benign breast conditions that occur usually in premenopausal woman. Breast abscess is the infectious and inflammatory breast condition that is common among breast-feeding mothers.

Sentinel lymph node mapping is done to validate the lack of lymph node metastasis. Which of the following complications does this technique help avoid? a) Lymphedema b) Fibroadenoma c) Breast cancer d) Mastalgia

Lymphedema Correct Explanation: Validating the lack of lymph node metastasis allows the surgeon to preserve more breast and axillary tissue and chest muscle. Leaving more normal lymph nodes intact reduces the potential for complications, such as lymphedema caused by the extensive disruption of lymphatic circulation.

What is used to better define breast cancer by defining an area slice by slice?

MRI

A client is considering breast augmentation. Which of the following would the nurse recommend to the client to ensure that there are no malignancies? a) Breast biopsy b) Mammogram c) Mastopexy d) Ultrasound

Mammogram Correct Explanation: When caring for a client considering breast augmentation, the nurse should provide her with a general guideline to have a mammogram to verify that there are no malignancies. Mastopexy involves a breast life for drooping breasts. Ultrasound or breast biopsy would not be necessary unless there was evidence of a problem.

Which of the following terms refers to breast pain? a) Mastalgia b) Mammoplasty c) Mastitis d) Gynecomastia

Mastalgia Mastalgia refers to breast pain. Mastitis is an inflammation or infection of the breast. Gynecomastia refers to overdeveloped breast tissue typically seen in adolescent boys. Mammoplasty refers to a surgical procedure to reconstruct or change the size or shape of the breast.

A patient has been newly diagnosed with breast cancer. During her preoperative instructions the physician indicated that removal of the breast tissue and axillary lymph node dissection leaving the muscular structure intact was indicated as surgical treatment for her breast cancer. To which of the following procedures is the physician asking the patient to consent? a) Total mastectomy b) Radical mastectomy c) Segmental mastectomy d) Modified radical mastectomy

Modified radical mastectomy Correct Explanation: A modified radical mastectomy leaves the pectoralis major and minor muscles intact. In a segmental mastectomy, varying amounts of breast tissue are removed, including the malignant tissue and some surrounding tissue to ensure clear margins. In a total mastectomy, breast tissue only is removed. Radical mastectomy includes removal of the pectoralis major and minor muscles in addition to breast tissue and axillary lymph node dissection.

Which of the following terms is used to describe removal of the breast tissue and an axillary lymph node dissection leaving muscular structure intact as surgical treatment of breast cancer? a) Radical mastectomy b) Total mastectomy c) Segmental mastectomy d) Modified radical mastectomy

Modified radical mastectomy Correct Explanation: A modified radical mastectomy leaves the pectoralis major and minor muscles intact. In a segmental mastectomy, varying amounts of breast tissue are removed, including the malignant tissue and some surrounding tissue to ensure clear margins. In a total mastectomy, breast tissue only is removed. Radical mastectomy includes removal of the pectoralis major and minor muscles in addition to breast tissue and axillary lymph node dissection.

The nurse recognizes which of the following statements as accurately reflecting a risk factor for breast cancer? a) Onset of menses before 14 years of age b) Multiparity c) Mother affected by cancer before 60 years of age d) No alcohol consumption

Mother affected by cancer before 60 years of age Correct Explanation: Risk for breast cancer increases twofold if first-degree female relatives (sister, mother, or daughter) have had breast cancer. Increased risk is associated with early menarche (i.e. menses beginning before 12 years of age). Nulliparity and later maternal age for first birth are associated with increased risk for breast cancer. Alcohol use remains controversial; however, a slightly increased risk is found in women who consume even one drink daily and doubles among women drinking three drinks daily.

Which of the following would be inconsistent as a risk factor for breast cancer? a) Family history of breast cancer b) Increased age c) Late menopause d) Multiparity

Multiparity Correct Explanation: Nulliparity is a risk factor for breast cancer, along with late menopause, increased age, and family history of breast cancer.

An early sign of Paget's disease includes which of the following? a) Increased pulse b) Thickening of areola c) Fever d) Nipple erythema

Nipple erythema Explanation: Early signs of Paget's disease include nipple and areola erythema. Late signs include thickening, scaling, and erosion of the nipple and areola.

The nurse is caring for a group of breast cancer survivors post mastectomy. When developing a list of instructions of points to avoid, which point is highlighted? a) Applying cream to breast b) Arm exercises on affected side c) Wearing loose fitting shirts d) No lifting greater than 15 lb

No lifting greater than 15 lb Correct Explanation: Of the list citing things to avoid, the correct option to avoid is lifting more than 15 lb. This fact is important and needs to be highlighted. Tight-fitting or constrictive clothing is to be avoided. The client is encouraged to place cream on any area of the breast which is dry. Arm exercises are routinely completed on the affected arm.

Which statement about the classification of testicular cancers is correct?

Nonseminomas are testicular germ cell tumors.

In the United States, what type of cancer accounts for one-third of cancer diagnoses and is the most common cancer in women? a) Breast cancer b) Uterine cancer c) Lung cancer d) Ovarian cancer

a: According to the American Cancer Society (ACS), in the United States, breast cancer is the most common cancer in women and accounts for 1 in 3 cancer diagnoses.

The nurse teaches the female patient who is premenopausal to perform breast self-examination (BSE) at which time frame? a) With the onset of menstruation b) On day 5 to day 7, counting the first day of menses as day 1 c) On day 2 to day 4, counting the first day of menses as day 1 d) Any time during the month

On day 5 to day 7, counting the first day of menses as day 1 BSE is best performed after menses, when less fluid is retained. Because most women notice increased tenderness, lumpiness, and fluid retention before their menstrual period, BSE is not recommended with the onset of menses. Because the tenderness, lumpiness, and fluid retention problems noticed by women in relation to onset of menses and generally continues through menses, BSE is not recommended during that time. Because most women notice increased tenderness, lumpiness, and fluid retention before their menstrual period, BSE is best performed when the time for menses is taken into account.

A client with breast cancer is receiving chemotherapy as part of the treatment plan. The client develops neutropenia and is scheduled to receive pegfilgrastim (Neulasta). The nurse would expect this drug to be given at which time? a) Once, approximately 24 hours after chemotherapy b) Weekly during chemotherapy administration c) Every 2 to 3 weeks after chemotherapy is given d) 7 to 10 days after chemotherapy administration

Once, approximately 24 hours after chemotherapy Explanation: Pegfilgrastim (Neulasta) is a long-acting granulocyte colony-stimulating factor(G-CSF) that is given via injection once, 24 hours after chemotherapy. Filgrastim (Neupogen), a short-acting G-CSF, is injected subcutaneously from 7 to 10 days after chemotherapy administration. Epoetin alfa, an erythropoietin growth factor, is administered weekly; darbepoetin alfa, a long-acting erythorpoietin growth factor, is administered every 2 to 3 weeks.

The nurse is caring for a client who is being treated with a 5-alpha reductase inhibitor (5-ARI) as the first-line drug therapy for benign prostatic hyperplasia (BPH). What instruction does the nurse give to the client about this therapy?

Orthostatic hypotension is a side effect of this treatment.

The nurse is completing the admission history for a client who is admitted for a reduction mammoplasty. Which of the following client statements is uncommon when explaining the rationale for the procedure? a) Back pain b) Skin irritation c) Low self-esteem d) Others disapprove

Others disapprove Correct Explanation: The rationale for a reduction mammoplasty most often comes from the client herself as she is experiencing a complication due to the size of the breast. Back pain, low self-esteem and a self-consciousness, and skin irritation are common rationales.

Which assessment finding causes the nurse to suspect that a client may have testicular cancer?

Painless testicular lump

The nurse is caring for a client with erectile dysfunction (ED) who has not had success with other treatment modalities. The nurse anticipates that the health care provider will recommend which treatment for this client?

Penile implants

The nurse is assessing a client suspected of having prostate cancer. What risk factor is associated with the development of prostate cancer?

Presence of BRCA2 gene mutations

What are the risk factors for prostate cancer? Select all that apply.

Presence of BRCA2 gene mutations Mutation in the glutathione S-transferase (GST P1) gene Exposure to arsenic

The nurse is reviewing the results of a client's prostate specific antigen (PSA) test and notices an elevation in the findings. What may this finding represent?

Prostate cancer

Why is prostate cancer screening often emphasized to the African-American population in the United States?

Prostate cancer occurs at an earlier age.

The potential problem of grief is most relevant to a client after which procedure?

Radical prostatectomy

On a follow-up visit, the patient is informed that her breast cancer has recurred. The nurse knows which of the following situations places the patient at risk for developing bone metastases? a) Recurrence within 2 years of the original diagnosis b) No maternal family history of breast cancer c) Aggressive treatment, including surgery, radiation, and hormonal therapy d) Previous therapeutic response to chemotherapy

Recurrence within 2 years of the original diagnosis Local recurrence may be an indicator that systemic disease will develop in the future, particularly if it occurs within 2 years of the original diagnosis. Local recurrence in the absence of systemic disease is treated aggressively with surgery, radiation, and hormonal therapy. Overall prognosis and optimal treatment are determined by a variety of factors such as the time to recurrence from the original diagnosis and history of prior treatments.

A nurse is caring for a client who has just had a modified radical mastectomy with immediate reconstruction. She's in her 30s and has two young children. Although she's worried about her future, she seems to be adjusting well to her diagnosis. What should the nurse do to support her coping? a) Encourage the client to proceed with the next phase of treatment. b) Tell the client's spouse or partner to be supportive while she recovers. c) Recommend that the client remain cheerful for the sake of her children. d) Refer the client to the American Cancer Society's Reach for Recovery program or another support program.

Refer the client to the American Cancer Society's Reach for Recovery program or another support program. The client isn't withdrawn and doesn't show other signs of anxiety or depression. Therefore, the nurse can probably safely approach her about talking with others who have had similar experiences, either through Reach for Recovery or another formal support group. The nurse may educate the client's spouse or partner and listen to his concerns, but the nurse shouldn't tell the client's spouse what to do. The client must consult with her physician and make her own decisions about further treatment. The client needs to express her sadness, frustration, and fear. She can't be expected to be cheerful at all times.

Which method is a common complementary and alternative therapy for benign prostatic hyperplasia (BPH)?

Serenoa repens

A client with prostate cancer is being evaluated for metastasis. Which laboratory test indicates possible bone metastasis?

Serum alkaline phosphatase

The issue that is often foremost in the minds of men who have been diagnosed with prostate cancer and must be addressed by the nurse is the alteration of which factor?

Sexual function after treatment

A client is scheduled for a transurethral resection of the prostate (TURP) for benign prostatic hyperplasia (BPH). What preoperative information must the client understand?

Sexual function is not usually affected with TURP.

The nurse is obtaining a health history from a client suspected of benign prostatic hyperplasia (BPH). What are the symptoms of BPH? Select all that apply.

Sexual function is not usually affected with TURP. Sensation of incomplete bladder emptying Difficulty in starting and continuing urination Decreased force and intensity of urinary stream Frequent waking during the night to void

A client with testicular cancer is worried about sterility and the ability to conceive children later. Which resource does the nurse refer the client to before surgery takes place?

Sperm bank

Which of the following is one of the most important prognostic factor in breast cancer? a) Age of patient b) Obesity c) Family history d) Status of lymph nodes

Status of lymph nodes Explanation: The two most important prognostic factor in breast cancer is the status of the lymph nodes and tumor size. Obesity, age of the patient, and family history are not the most important prognostic factors in breast cancer.

Which type of biopsy is used for nonpalpable lesions found on mammography? a) Tru-Cut core b) Incisional c) Stereotactic d) Excisional

Stereotactic Correct Explanation: Stereotactic biopsy utilizes computer location of the suspicious area found on biopsy, followed by core needle insertion and sampling of tissue for pathologic examination. An excisional biopsy is the usual procedure for any palpable breast mass. Incisional biopsy is performed on a palpable mass when tissue sampling alone is required. Tru-Cut core biopsy is used when a tumor is relatively large and close to the skin surface.

Hormone treatment for prostate cancer works by which action?

Suppresses growth of the tumor

Students are reviewing information about the use of adjuvant hormonal therapy for breast cancer. They demonstrate understanding of this information when they identify which of the following as an example of a selective estrogen receptor modulator (SERM)? a) Tamoxifen b) Exemestane c) Letrozole d) Anastrozole

Tamoxifen Tamoxifen is an example of a SERM. It acts by competing for estrogen-receptor binding sites. Anastrozole, letrozole, and exemestane are examples of aromatase inhibitors, drugs that block estrogen production.

The nurse is preparing a client for a digital rectal examination (DRE) of the prostate. What does the nurse teach the client about the examination?

The client must void before the physical examination.

A client comes to the clinic for a routine evaluation. During the physical examination, the nurse palpates the client's breast and finds a small lump. Which of the following would lead the nurse to suspect possible breast cancer? a) The lump is soft. b) The client reports tenderness during the palpation. c) The lump is irregularly shaped. d) The lump is mobile.

The lump is irregularly shaped. Correct Explanation: Generally, breast cancer lesions are nontender, fixed rather than mobile, and hard with irregular borders. Diffuse breast pain and tenderness with menstruation are usually associated with benign breast disease.

The client arrives at a public health clinic worried that she has breast cancer since finding a lump in her breast. When assessing the breast, which assessment finding is characteristic of fibrocystic disease? a) One breast is larger than the other. b) Nipple retractions are noted. c) The lump is round and movable. d) The lump is firm and nonmovable.

The lump is round and movable. When assessing a breast with fibrocystic disease, the lumps typically are different from cancerous lumps. The characteristic breast mass of fibrocystic disease is soft to firm, circular, movable, and unlikely to cause nipple retraction. A cancerous mass is typically irregular in shape, firm, and nonmovable. Lumps typically do not make one breast larger than the other. Nipple retractions are suggestive of cancerous masses.

The patient with breast cancer has a left mastectomy with axillary node dissection. Ten lymph nodes are resected with three positive for malignant cells. The patient has stage IIB breast cancer. What is the best nursing intervention to use in planning care? a. evaluate left arm lymphatic accumulation b. maintain joint flexibility and left arm function c. teach her about chemotherapy and radiation therapy d. assess the patient's response to the diagnosis of breast cancer

d.

The nurse is caring for a client who is ordered a sentinel lymph node biopsy. The physician explained the procedure and desired outcome. Which statement, made by the client, indicates a need for further instruction? a) The procedure allows for conservation of breast tissue. b) The procedure allows for an understanding of the spread of cancer cells. c) The procedure removes all cancer from the body. d) The procedure includes minimal surrounding tissue damage.

The procedure removes all cancer from the body. Explanation: Sentinel lymph node mapping involves identifying the first (sentinel) lymph nodes through which the breast cancer cells would spread to regional lymph nodes in the axilla. Validating the lack of lymph node metastasis allows the surgeon to preserve more breast tissue, axillary tissue and chest muscle. Further instruction would be needed to explain that the sentinel lymph node biopsy does not remove cancer from the body.

What does the nurse teach a client with urge incontinence about Kegel exercises?

They involve the use of perineal and gluteal muscles.

A client with breast cancer is scheduled to undergo chemotherapy with aromatase inhibitors. Which of the following best reflects the rationale for using this group of drugs? a) They lower the level of estrogen in the body blocking the tumor's ability to use it. b) They stimulate the immune system to attack a protein common in many tumors. c) They block progesterone-dependent tumors from growing. d) They attach to endogenous protein receptors to slow the growth of cancerous cells.

They lower the level of estrogen in the body blocking the tumor's ability to use it. Explanation: Aromatase inhibitors lower the level of estrogen in the body thereby interfering with the ability of hormone-sensitive tumors to use estrogen for growth. Antiprogestin drug, such as mifepristone, blocks progesterone-dependent breast cancers. The monoclonal antibody, trastuzumab attaches to protein receptors to slow the growth of cancer cells. A breast cancer vaccine is under investigation in Italy. This vaccine stimulates the immune system to attack a protein called mammaglobin-A, which is found in 80% of breast cancer tumors.

A client complains of having tender and painful breasts, often feeling multiple lumps within her breast tissue. The nurse would need to gather additional information about which of the following? a) Client's workplace in relation to the surroundings b) Timing of symptoms in relation to the menstrual cycle c) Bathing frequency and living surroundings d) Alcohol and caffeine consumption

Timing of symptoms in relation to the menstrual cycle Considering that the client has tender and painful breasts and that she often feels lumps within her breast tissue, it is most likely that she suffers from fibrocystic breast disease. To confirm these findings, the nurse should ask relevant questions about the characteristics and timing of symptoms in relation to the menstrual cycle. Symptoms of fibrocystic breast disease are noticeable before menstruation and usually abate during menstruation. The size of the cyst becomes larger before menstruation and often changes with the menstrual cycle. The nurse should further ask the client about her habits of smoking and consuming coffee, chocolate, and caffeinated soft drinks, not alcohol, because they aggravate the condition. Workplace surroundings or cleanliness habits do not matter because fibrocystic breast disease is not infectious.

A client complains of having tender and painful breasts, often feeling multiple lumps within her breast tissue. The nurse would need to gather additional information about which of the following? a) Bathing frequency and living surroundings b) Client's workplace in relation to the surroundings c) Alcohol and caffeine consumption d) Timing of symptoms in relation to the menstrual cycle

Timing of symptoms in relation to the menstrual cycle Correct Explanation: Considering that the client has tender and painful breasts and that she often feels lumps within her breast tissue, it is most likely that she suffers from fibrocystic breast disease. To confirm these findings, the nurse should ask relevant questions about the characteristics and timing of symptoms in relation to the menstrual cycle. Symptoms of fibrocystic breast disease are noticeable before menstruation and usually abate during menstruation. The size of the cyst becomes larger before menstruation and often changes with the menstrual cycle. The nurse should further ask the client about her habits of smoking and consuming coffee, chocolate, and caffeinated soft drinks, not alcohol, because they aggravate the condition. Workplace surroundings or cleanliness habits do not matter because fibrocystic breast disease is not infectious.

The nursing is caring for a client who will be having artificial implants for breast reconstruction. The client is arriving at the physician's office for which procedure completed before the surgery can be done?

Tissue expansion Correct Explanation: Before an implant for breast reconstruction can produce an optimum cosmetic appearance, the skin and tissue on the chest wall are expanded to provide a large enough space to fill and approximate the size of the remaining breast. The other options are not correct.

A client with a fibroadenoma is being scheduled for diagnostic testing. Which of the following would the nurse expect as most likely? a) Ultrasound b) Culture of discharge c) Excisional biopsy d) Mammogram

Ultrasound Explanation: Ultrasound can reveal physical characteristics unique to a fibroadenoma versus malignant mass with a higher degree of accuracy than mammography. In the case of very young women—an atypical age for breast cancer—an excisional biopsy is performed only if the mass changes or becomes larger. If the mass is detected in a woman with a higher risk for developing breast cancer, such as one with a family history or of an older age, a biopsy is performed to confirm that the tissue is indeed benign. There is no discharge to culture.

A group of students are reviewing the anatomy and physiology of the breasts. The students demonstrate understanding of breast structure when they identify the tail of Spence as an extension of which quadrant? a) Lower inner b) Upper inner c) Lower outer d) Upper outer

Upper outer The tail of Spence is an area of breast tissue that extends from the upper outer quadrant of the breast into the axilla. The upper and lower inner quadrants are closer to the midline. The lower outer quadrant is below the upper outer quadrant.

The nurse is caring for a client after a transurethral resection of the prostate (TURP). What action does the nurse take?

Use normal saline solution for bladder irrigant.

The nurse is working with a couple having difficulty conceiving a child. What may be a possible cause of infertility in the male?

Varicocele

Which factors contribute to the development of breast cancer? a) Aging b) High breast density c) Family history d) Antibiotic use e) Hormonal factors

a, b, c, e: The following factors are thought to contribute to the development of breast cancer: aging, hormonal factors, delayed childbearing or not having children, high breast density, a family history of breast or ovarian cancer, and late menopause. Antibiotic use has not been identified as a factor contributing to the development of breast cancer.

A woman's breast examination reveals a small mass. Which of the following characteristics would lead the nurse to suspect that the mass is malignant? Select all that apply. a) Skin dimpling b) Hard on palpation c) Painless d) Clearly delineated e) Rubbery feel f) Mobile

a, b, c: Malignant breast masses are typically described as hard on palpation and painless, with skin dimpling. Benign breast masses are described as mobile and clearly delineated, with a rubbery feel

Lumpectomy is a treatment option for clients diagnosed with breast cancer with tumors smaller than 5 cm. For which clients is lumpectomy contraindicated? Select all that apply. a) Client who has had previous radiation to the affected breast b) Client whose surgery will not result in a clean margin of tissue c) Client who has had an early menarche or late onset of menopause d) Client whose connective tissue is reported to be sensitive to radiation e) Client who has failed to breastfeed for up to 1 year after pregnancy

a, b, d: Lumpectomy is contraindicated for women who have previously undergone radiation to the affected breast, those whose connective tissue is reported to be sensitive to radiation, and those whose surgery will not result in a clean margin of tissue. Clients who have had an early menarche or late onset of menopause and clients who have failed to breastfeed for up to 1 year after pregnancy are at risk for developing breast cancer. Lumpectomy is a treatment option for clients with breast cancer

A patient with breast cancer has a lumpectomy with sentinel lymph node dissection that is positive for cancer. You explain that, of the other tests done to determine the risk for cancer recurrence or spread, the results that support the more favorable prognosis are (select all that apply): a. well-differentiated tumor b. estrogen receptor-positive tumor c. overexertion of HER-2 cell marker d. involvement of two to four axillary nodes e. aneuploidy status from cell proliferation studies

a, b.

The nurse is volunteering at a community center to teach women about breast cancer. What should the nurse include when discussing risk factors (select all that apply)? a. nulliparity b. age 30 or over c. early menarche d. late menopause e. personal history of colon cancer

a, c, d, e.

A nurse is screening women for risk factors for breast cancer. Which of the following are considered a risk for this disorder? Select all that apply. a) Cystic breast disorders b) Birth of a child before age 25 c) Having breast-fed children d) Having an extreme fear of cancer e) Previous cancer f) Using female hormone therapy

a, d, e, f: Routine mammography is strongly recommended for women who have any of the following characteristics: previous cancer, cystic breast disorders, no children or birth of first child after age 30 years, no breast-fed children, family history of breast cancer, strong family history of any type of cancer, female hormone (estrogen) therapy, or extreme fear of cancer (need mammography for reassurance

A 33-year-old female client reports yellow nipple discharge and a pain in her breasts a week before menses that dissipates on the onset of menses. Diagnosis reveals that the client is experiencing fibrocystic breast changes. Which instructions should the nurse offer the client to help alleviate the condition? Select all that apply. a) Avoid caffeine b) Practice good hand-washing techniques c) Increase fluid intake steadily d) Take diuretics as recommended e) Maintain a low-fat diet

a, d, e: The nurse should instruct the client with fibrocystic breast changes to avoid caffeine. Caffeine acts as a stimulant that can lead to discomfort. It is important to maintain a lowfat diet rich in fruits, vegetables, and grains to maintain a healthy body weight. Taking diuretics is important to counteract fluid retention and swelling of the breasts. Practicing good handwashing techniques and increasing fluid intake are important for clients with mastitis but may not help clients with fibrocystic breast changes.

Which of the following are modifiable risk factors for breast cancer? Select all that apply. a) Sedentary lifestyle b) Smoking c) Duration of breastfeeding d) Alcohol consumption e) Obesity

a,b,c,d,e: All of the above are modifiable risk factors for breast cancer. Women who are nonsmokers, have a normal BMI, exercise, and drink little alcohol have a lower lifetime risk of breast cancer. Breastfeeding is inversely correlated with breast cancer. The more a woman breastfeeds, the less likely she is to be diagnosed with breast cancer

A 72-year-old patient who had a mastectomy for breast cancer 6 months ago wants to have breast reconstructive surgery. The nurse knows that what is the most likely motivation for this patient seeking this surgery? a. improve the woman's self-image b. be able to experience sexual arousal c. to get a tummyt tuck as well as the breast mound d. restore the pre-mastectomy appearance of the breast

a.

To prevent capsular formation after breast reconstruction with implants, teach the patient to: a. gently massage the area around the implant b. bind the breasts tightly with elastic bandages c. exercise the arm on the affected side to promote drainage d. avoid strenuous exercise until the implant has healed

a.

When discussing risk factors for breast cancer with a group of women, you emphasize that the greatest known risk factor for breast cancer is: a. being a woman over age 60 b. experiencing menstruation for 30 years or more c. using hormone therapy for 5 years for menopausal symptoms d. having a paternal grandmother with postmenopausal breast cancer

a.

14. Which information will the nurse include in patient teaching for a 36-year-old patient who is scheduled for stereotactic core biopsy of the breast? a. A local anesthetic will be given before the biopsy specimen is obtained. b. You will need to lie flat on your back and lie very still during the biopsy. c. A thin needle will be inserted into the lump and aspirated to remove tissue. d. You should not have anything to eat or drink for 6 hours before the procedure.

a. A local anesthetic will be given before the biopsy specimen is obtained.

15. A student nurse prepares a list of teaching topics for a patient with a new diagnosis of breast cancer. Which item should the charge nurse suggest that the student nurse omit from the teaching topic list about breast cancer diagnostic testing? a. CA 15-3 level testing b. HER-2 receptor testing c. Estrogen receptor testing d. Oncotype DX assay testing

a. CA 15-3 level testing

3. A 51-year-old patient with a small immobile breast lump is considering having a fine-needle aspiration (FNA) biopsy. The nurse explains that an advantage to this procedure is that a. FNA is done in the outpatient clinic and results are available in 1 to 2 days. b. only a small incision is needed, resulting in minimal breast pain and scarring. c. if the biopsy results are negative, no further diagnostic testing will be needed. d. FNA is guided by a mammogram, ensuring that cells are taken from the lesion.

a. FNA is done in the outpatient clinic and results are available in 1 to 2 days.

During a counseling session for a group of teenage athletes, the use of androgenic steroids is discussed. The nurse will explain that which problem is a rare but devastating effect of androgenic steroid use? a. Peliosis of the liver b. Bradycardia c. Kidney failure d. Tachydysrhythmias

a. Peliosis of the liver

26. A patient who is scheduled for a lumpectomy and axillary lymph node dissection tells the nurse, I would rather not know much about the surgery. Which response by the nurse is best? a. Tell me what you think is important to know about the surgery. b. It is essential that you know enough to provide informed consent. c. Many patients do better after surgery if they have more information. d. You can wait until after surgery for teaching about pain management.

a. Tell me what you think is important to know about the surgery.

After teaching a woman about breast self-examination, which statement indicates that the teaching was successful? a) "I'll do the check about a week after my period." b) "I will check my breasts every other week." c) "I'll feel my breasts using my thumb and index finger." d) "I don't have to check under my arm if I don't feel any breast lumps."

a: Breast self-examination is best performed a week after menses, when swelling has subsided. Breast self-examination is typically performed every month. Both the breast area and the area between the breast and underarm, including the underarm itself, should be part of breast self-examination. The woman should use the pads of her three middle fingers for palpation

Edema and enlargement in the ducts and glands of the breast that occur as effects of hormones are called a) Fibrocystic breast changes b) Fibroadenomas c) Duct ectasia d) Intraductal papilloma

a: Changes in the breast that result from hormonal influences are fibrocystic breast changes.

A woman has been treated for a tumor of the left breast that has not responded to chemotherapy. The woman has just found out that she has the BRCA mutations and discusses her options with her physician. What treatment would be most difficult for this woman? a) Bilateral mastectomy b) Left mastectomy c) Radiation therapy d) More aggressive chemotherapy

a: Chemotherapy, left mastectomy, and radiation therapy may be difficult for the woman, but the most difficult and controversial treatment is bilateral mastectomy. Right mastectomy is considered a prophylactic mastectomy which is a primary prevention modality. Patients who are considering prophylactic mastectomy are often faced with a very controversial and emotion decision

A patient who came to the clinic after finding a mass in her breast is scheduled for a diagnostic breast biopsy. What is it important for the nurse to do? a) Acknowledge the fear the patient is experiencing b) Set up a consultation with a breast cancer survivor c) Instruct the patient on prosthetic devices d) Refer her to Reach for Recovery

a: In the breast cancer diagnostic phase it is appropriate to acknowledge her feelings of fear, concern, and apprehension. Referral to Reach for Recovery and with a breast cancer survivor is indicated when the diagnosis has been made and surgical intervention is scheduled. Instructing the patient on prosthetic devices is only appropriate when the patient has had or is scheduled for a mastectomy

During a routine physical examination, a firm mass is palpated in the right breast of a 35-year-old woman. Which finding or client history would suggest cancer of the breast as opposed to fibrocystic disease? a) Increased vascularity of the breast b) Mass located in upper, outer quadrant c) Cyclic change in mass size d) History of anovulatory cycles

a: Increase in breast size or vascularity is consistent with breast cancer. Masses associated with fibrocystic disease of the breast are firm, are most commonly located in the upper outer quadrant of the breast, and increase in size before menstruation. They may be bilateral in a mirror image and are typically well demarcated and freely moveable

A small amount of breast milk is obtained for culture and sensitivity testing from a client with mastitis. The nurse would expect the results to identify which organism as the most likely cause? a) Staphylococcus aureus b) Group A streptococcus c) Chlamydia trachomatis d) Escherichia coli

a: The most common causative microorganism associated with mastitis is Staphylococcus aureus. Chlamydia is a sexually transmitted infection. Streptococcus is commonly associated with strep throat. E. coli is a common cause of urinary tract infections

A client diagnosed with fibroadenoma is worried about the chances of developing breast cancer. She also asks the nurse about various breast disorders and their risks. Which benign breast disorder should the nurse include as having the greatest risk for the development of breast cancer? a) Intraductal papilloma b) Fibrodenomas c) Mastitis d) Mammary duct ectasia

a: The nurse should inform the client that intraductal papillomas and fibrocystic breasts, although considered benign, carry a cancer risk with prolific masses and hyperplastic changes within the breasts. Other benign breast disorders such as mastitis, mammary duct ectasia, and fibroadenomas carry little risk.

It is recommended that a 48-year-old female client with breast cancer undergo a sentinel lymph node biopsy before a lumpectomy. The client asks the nurse the reason for removing the sentinel lymph node. Which statement will the nurse make? a) "It will prevent lymphedema, which is a common side effect." b) "It will decrease the amount of treatment you need." c) "It will reveal the hormone receptor status of the cancer." d) "It will lessen the aggressiveness of the subsequent chemotherapy."

a: The nurse should inform the client that removing only the sentinel lymph node prevents side effects such as lymphedema, which is otherwise associated with a traditional axillary lymph node dissection. It does not help reveal the hormonal status of the cancer. Hormone-receptor status can be revealed through normal breast epithelium, which has hormone receptors and responds specifically to the stimulatory effects of estrogen and progesterone. A sentinel lymph node biopsy will determine how powerful a chemotherapy regimen the client will have to undergo, but undergoing a sentinel lymph node biopsy will not lessen the aggressiveness of the chemotherapy. Degree of HER-2/neu oncoprotein will be revealed through the HER-2/neu genetic marker, not through a sentinel lymph node biopsy

A female client with a malignant tumor of the breast has to undergo chemotherapy for a period of 6 months. For which side effect should the nurse monitor when caring for this client? a) Constipation b) Chills c) Vaginal discharge d) Headache

a: The side effects of chemotherapy are constipation, hair loss, weight loss, vomiting, diarrhea, immunosuppression, and, in extreme cases, bone marrow suppression. The nurse should monitor for these side effects when caring for the client undergoing chemotherapy. Vaginal discharge, headache, and chills are not side effects of chemotherapy. Vaginal discharge is one of the side effects of SERMs as a part of hormonal therapy, which is used to prevent cancer from spreading further into the body. Headache is a side effect of aromatase inhibitors under hormonal therapy to counter cancer. Chills are a side effect of immunotherapy

When preparing a teaching plan for a woman with mastitis, which instruction would the nurse include? a) Applying warm compresses to the affected breast b) Ceasing breastfeeding to prevent transmission to the infant c) Limiting fluid intake to minimize swelling d) Avoiding the use of a supportive bra to prevent pressure on the inflamed breast

a: Warm compresses are soothing and help reduce inflammation. Breastfeeding is encouraged with mastitis to ensure continued emptying of the breast. Fluid intake is encouraged to promote milk production and resolution of infection. Wearing a supportive bra 24 hours a day is necessary to support the breasts

A woman is crying because she just recently received the results of her biopsies, and they confirm that she has invasive breast cancer. Which response by the nurse is the most appropriate? a) Listen to the woman talk and remain silent for a while b) "You'll beat this thing, I know it. You are very strong." c) "I'm sure you are going to be fine. You are in great hands." d) "I know a great support group you can join."

a: When a woman first receives the devastating news of the diagnosis of cancer, most often the best response is to allow the woman to express her feelings and concerns before speaking. Giving her false reassurances is not therapeutic and can break reliability and trust in a provider/patient relationship. Attempting to give her information about groups or next steps before she is in a state to take it in is also nontherapeutic

A nurse is caring for a client who has just had her intraductal papilloma removed through a surgical procedure. What instructions should the nurse give this client as part of her care? a) Continue monthly breast self-examinations b) Wear a supportive bra 24 hours per day c) Refrain from consuming salt in diet d) Apply warm compresses to the affected breast

a: When caring for a client who has just undergone surgery for intraductal papilloma, the nurse should instruct the client to continue monthly breast self-examinations along with yearly clinical breast examinations. Applying warm compresses to the affected breast and wearing a supportive bra 24 hours a day are instructions given in cases of mastitis but not for intraductal papilloma. The nurse should instruct clients to refrain from consuming salt in the diet in cases of fibrocystic breast changes but not in cases of intraductal papilloma

A 52-year-old female client with an estrogen receptors positive (ER+) breast cancer is undergoing hormonal therapy. While taking a selective estrogen receptor modulator (SERM), the client begins to experience hot flashes. What should the nurse do next? a) Document the hot flash in the client's chart b) Notify the client's health care provider c) Instruct the client to stop taking the SERM d) Assess the client's blood pressure

a: When caring for a client who is being administered selective estrogen receptor modulator, the nurse should monitor for side effects such as hot flashes, vaginal discharge, bleeding, and cataract formation. Hot flashes are an expected side effect of SERM; therefore the nurse should document the finding in the chart

Mammography is recommended for a client diagnosed with intraductal papilloma. Which factor should the nurse ensure when preparing the client for a mammography? a) Client has not applied deodorant on the day of testing. b) Client is just going to start her menses. c) Client has taken an aspirin before the testing. d) Client has not consumed fluids 1 hour before testing.

a: When preparing a client for mammography, the nurse should ensure the client has not applied deodorant or powder on the day of testing because these products can appear on the x-ray film as calcium spots. It is not necessary for the client to avoid fluid intake 1 hour prior to testing. Mammography has to be scheduled just after the client's menses to reduce chances of breast tenderness, not when the client is going to start her menses. The client can take aspirin or acetaminophen after the completion of the procedure to ease any discomfort, but these medications are not taken before mammography

What are guidelines for early detection of breast cancer?

age groups (early 20's teach importance and benefit of BSE, including men), age appropriate mammograms, initial screening for a baseline, teach to be aware and get checked, BSE-mark on calendar or have some type of reminder, what to report, give patient info on national breast cancer awareness with copies to take home

Where do most breast cancers occur?

at the tail of spence

A 38-year-old female client has to undergo lymph node surgery in conjunction with mastectomy. The client is likely to experience lymphedema due to the surgery. Postsurgery, which factors will make the client more susceptible to lymphedema? Select all that apply. a) Not consuming a diet high in fiber and protein b) Not wearing a well-fitted compression sleeve c) Using the affected arm for drawing blood or measuring blood pressure d) Engaging in activities like gardening without using gloves e) Not consuming foods that are rich in phytochemicals

b, c, d: The client is more susceptible to lymphedema if the affected arm is used for drawing blood or measuring blood pressure, if she engages in activities like gardening without using gloves, or if she's not wearing a well-fitted compression sleeve to promote drainage return. Consuming foods rich in phytochemicals is essential to prevent the incidence of cancer, not lymphedema. Not consuming a diet high in fiber and protein will not make the client susceptible to lymphedema

A nurse is educating a 43-year-old female client about required lifestyle changes to help avoid breast cancer. Which instructions regarding diet and food habits should the nurse include in the teaching plan? Select all that apply. a) Consume at least five servings of proteins daily b) Limit intake of processed foods c) Restrict intake of salted foods d) Consume seven or more portions of complex carbohydrates daily e) Increase liquid intake to 3 L daily

b, c, d: The nurse should instruct the client to restrict intake of salted foods, limit intake of processed foods, and consume seven or more daily portions

How should the nurse instruct a woman to perform a breast self-examination? Select all that apply. a) "Use two different degrees of pressure: light for the skin and hard for the bone." b) "You will need to visually inspect both breasts and palpate them for changes." c) "Use the pads of your first two fingers on the right hand for the right breast." d) "Look for any changes in shape, size, contour, or symmetry between the breasts, as well as any skin discoloration." e) "It doesn't matter which pattern you use to palpate the breast as long as you cover the entire breast."

b, d, e: To palpate the breasts, the woman should use the pads of three fingers from the opposite hand to the opposite breast (left to right). The nurse should instruct the woman to do three different levels of pressure, light (skin and tissue), medium (tissue), and hard (bone).

A 24-year-old patient who has undergone breast augmentation earlier in the day will be discharged home in the early evening. What instructions should the nurse provide in order to minimize the patient's risk of complications in the immediate recovery period? a. avoid wearing a bra until postoperative day 3 b. ask the patient to avoid strenuous exercise during her recovery period c. sleep in a semi-Fowler's position until her scheduled follow-up appointment d. enlist a friend or family member to perform passive range-of-motion exercises

b.

A 50-year-old patient is preparing to begin breast cancer treatment with tamoxifen (Nolvadex). What point should the nurse emphasize when teaching the patient about her new drug regimen? a. "you may find that your medication causes some breast sensitivity." b. "it's important that you let your care provider know about any changes in your vision." c. "you'll find that this drug often alleviates some of the symptoms that accompany menopause." d. "it's imperative that you abstain from drinking alcohol after you begin taking tamoxifen."

b.

The nurse has been asked to participate in a healthy living workshop. While teaching about women's health, which guidelines should the nurse provide to the audience? a. "mammograms are necessary if you have a family history of breast cancer." b. "it's recommended that you get a mammogram each year after you turn 40." c. "if you are not able to perform breast self-examination (BSE), you should go for regular mammograms." d. "you should ensure that your primary care provider performs a breast exam each time you visit."

b.

Which task could the registered nurse delegate to unlicensed assistive personnel (UAP) during the care of a patient who has had recent transverse rectus abdomens musculocutaneous (TRAM) flap surgery? a. document the condition of the patient's incisions b. mobilize the patient in a slightly hunched position c. change the patient's abdominal and chest dressings d. change the parameters of the patient-controlled analgesic (PCA) pump

b.

4. Which assessment finding in a 36-year-old patient is most indicative of a need for further evaluation? a. Bilateral breast nodules that are tender with palpation b. A breast nodule that is 1 cm in size, nontender, and fixed c. A breast lump that increases in size before the menstrual period d. A breast lump that is small, mobile, with a rubbery consistency

b. A breast nodule that is 1 cm in size, nontender, and fixed

17. Which information should the nurse include in teaching a patient who is scheduled for external beam radiation to the breast? a. The radiation therapy will take a week to complete. b. Careful skin care in the radiated area will be necessary. c. Visitors are restricted until the radiation therapy is completed. d. Wigs may be used until the hair regrows after radiation therapy.

b. Careful skin care in the radiated area will be necessary.

10. The nurse provides discharge teaching for a 61-year-old patient who has had a left modified radical mastectomy and lymph node dissection. Which statement by the patient indicates that teaching has been successful? a. I will need to use my right arm and to rest the left one. b. I will avoid reaching over the stove with my left hand. c. I will keep my left arm in a sling until the incision is healed. d. I will stop the left arm exercises if moving the arm is painful.

b. I will avoid reaching over the stove with my left hand.

18. Which patient statement indicates that the nurses teaching about tamoxifen (Nolvadex) has been effective? a. I can expect to have leg cramps. b. I will call if I have any eye problems. c. I should contact you if I have hot flashes. d. I will be taking the medication for 6 to 12 months.

b. I will call if I have any eye problems.

A patient has been taking finasteride (Proscar) for almost 1 year. The nurse knows that which is most important to evaluate at this time? a. Complete blood count b. PSA levels c. Blood pressure d. Fluid retention

b. PSA levels

When the Testoderm form of testosterone is ordered to treat hypogonadism in a teenage boy, which instructions by the nurse are correct? (Select all that apply.) a. Place the patch on clean, dry skin on the back, upper arms, abdomen, or thighs. b. Place the patch on clean, dry scrotal skin that has been shaved. c. Place the patch on clean, dry scrotal skin, but do not shave the skin first. d. Place the patch on any clean, dry, nonhairy area of the body. e. Remove the old patch before applying a new patch.

b. Place the patch on clean, dry scrotal skin that has been shaved. e. Remove the old patch before applying a new patch.

6. A 58-year-old woman tells the nurse, I understand that I have stage II breast cancer and I need to decide on a surgery, but I feel overwhelmed. What do you think I should do? Which response by the nurse is best? a. I would have a lumpectomy, but you need to decide what is best for you. b. Tell me what you understand about the surgical options that are available. c. It would not be appropriate for me to make a decision about your health. d. There is no need to make a decision rapidly; you have time to think about this.

b. Tell me what you understand about the surgical options that are available.

The nurse is reviewing the history and physical exam of a woman who has come to the clinic for a routine physical. Which factor would the nurse identify as increasing the client's risk for breast cancer? a) Asian race b) History of ovarian cancer c) Menarche at age 14 d) 39 years of age

b: A personal history of ovarian cancer is considered a risk factor for breast cancer. Typically, breast cancer is associated with aging (women over 50 years of age). Breast cancer is more common in Caucasian women, but African-American women are more likely to die of it. Early menarche (before 12 years of age) or late onset of menopause (after age 55 years) is associated with an increased risk for breast cancer

A client complains of having tender and painful breasts, often feeling multiple lumps within her breast tissue. The nurse would need to gather additional information about which of the following? a) Client's workplace in relation to the surroundings b) Timing of symptoms in relation to the menstrual cycle c) Bathing frequency and living surroundings d) Alcohol and caffeine consumption

b: Considering that the client has tender and painful breasts and that she often feels lumps within her breast tissue, it is most likely that she suffers from fibrocystic breast disease. To confirm these findings, the nurse should ask relevant questions about the characteristics and timing of symptoms in relation to the menstrual cycle. Symptoms of fibrocystic breast disease are noticeable before menstruation and usually abate during menstruation. The size of the cyst becomes larger before menstruation and often changes with the menstrual cycle. The nurse should further ask the client about her habits of smoking and consuming coffee, chocolate, and caffeinated soft drinks, not alcohol, because they aggravate the condition. Workplace surroundings or cleanliness habits do not matter because fibrocystic breast disease is not infectious

A client with a fibroadenoma is being scheduled for diagnostic testing. Which of the following would the nurse expect as most likely? a) Excisional biopsy b) Ultrasound c) Mammogram d) Culture of discharge

b: Ultrasound can reveal physical characteristics unique to a fibroadenoma versus malignant mass with a higher degree of accuracy than mammography. In the case of very young women—an atypical age for breast cancer—an excisional biopsy is performed only if the mass changes or becomes larger. If the mass is detected in a woman with a higher risk for developing breast cancer, such as one with a family history or of an older age, a biopsy is performed to confirm that the tissue is indeed benign. There is no discharge to culture

A client complains of having tender and painful breasts, often feeling multiple lumps within her breast tissue. The nurse would need to gather additional information about which of the following? a) Alcohol and caffeine consumption b) Timing of symptoms in relation to the menstrual cycle c) Bathing frequency and living surroundings d) Client's workplace in relation to the surroundings

b: Considering that the client has tender and painful breasts and that she often feels lumps within her breast tissue, it is most likely that she suffers from fibrocystic breast disease. To confirm these findings, the nurse should ask relevant questions about the characteristics and timing of symptoms in relation to the menstrual cycle. Symptoms of fibrocystic breast disease are noticeable before menstruation and usually abate during menstruation. The size of the cyst becomes larger before menstruation and often changes with the menstrual cycle. The nurse should further ask the client about her habits of smoking and consuming coffee, chocolate, and caffeinated soft drinks, not alcohol, because they aggravate the condition. Workplace surroundings or cleanliness habits do not matter because fibrocystic breast disease is not infectious.

Which of the following would the nurse include in the plan of care for a client diagnosed with breast cancer who has undergone a mastectomy? a) Instructing client to use arm pumps b) Encouraging the client to perform arm exercises c) Applying ace bandages to the arms d) Counseling the client to apply elastic sleeves to her arms

b: Encouraging the client to perform arm exercises is important to maintain circulation to the arm on the affected side. Use of elastic sleeves, arm pumps, and ace bandaging is recommended for clients who may have complications following radiation or surgery for lymphedema of the arm

A 45-year-old woman comes into the OB/GYN clinic for her yearly check-up. The woman mentions to the nurse that she has a dimpling of the right breast that has occurred in the last 2 weeks. She has not performed a self-breast examination. What assessment would be appropriate for the nurse to make? a) Order an immediate mammogram b) Palpate the area for a breast mass c) Call the physician to schedule a biopsy d) Evaluate the patient's milk production

b: It would be most important for the nurse to palpate the breast to determine the presence of a mass. Edema and pitting of the skin may result from a neoplasm blocking lymphatic drainage, giving the skin an orange-peel appearance (peau d'orange), a classic sign of advanced breast cancer. Evaluation of milk production is required in lactating women. There is no indication of lactation in the scenario. A mammogram is an appropriate diagnostic test, but unless the nurse is a practitioner this would not be within the nurse's realm of practice. A referral could be made to a physician but the nurse would not proceed to schedule a biopsy

When counseling a woman with monthly breast pain, what dietary recommendations should the nurse provide? a) "Caffeine may help with monthly breast pain." b) "Eat plenty of fruits, vegetables, and whole grains, and follow a low-fat diet." c) "Drink milk and eat yogurt." d) "Be sure to consume plenty of electrolytes, including salt."

b: Maintaining a healthy weight will reduce pain from fibrocystic breasts. Caffeine and too much salt can contribute to fibrocystic breast changes. Calcium does not have a conclusive effect on breast pain

All of the following are characteristics of malignant breast disease EXCEPT a) Immobile, fixed masses b) Smooth, firm, rubbery masses c) Skin dimpling and nipple retraction d) Painless and nontender masses

b: Malignant breast disease is generally characterized by immobile, fixed lumps that may cause skin dimpling and nipple retraction. They are generally painless and not tender. They usually are not smooth and have poorly delineated edges

Which of the following pharmacologic treatments would NOT be therapeutic for a woman with fibrocystic breast changes? a) Diuretics b) Antihistamines c) Oral contraceptives d) Anti-inflammatory agents

b: Oral contraceptives will keep hormone levels stable to avoid peaks and troughs, which may cause breast edema and pain. Anti-inflammatory medications will be helpful in reducing pain from fibrocystic breast changes. Diuretics can be helpful in reducing excess fluid leading to edema and subsequent pain in the breast. Antihistamines are used to treat an overactive immune response, not edema from breast changes

You are caring for a 52-year-old woman whose sisters and mother died of breast cancer. The patient states, "My doctor wants me to take tamoxifen to help prevent breast cancer. What do you think?" What would be your best response? a) "Tamoxifen prevents osteoporosis." b) "Tamoxifen reduces the incidence." c) "I would recommend raloxifene." d) "We call this drug a chemotherapy agent."

b: Tamoxifen is referred to as a chemopreventive agent, not a chemotherapeutic agent. It does prevent osteoporosis, but this response does not address breast cancer prevention. Raloxifene (Evista) is another drug that shows promise as a chemopreventive agent. Tamoxifen reduces breast cancer incidence by 49%

Which of the following would the nurse expect to find when assessing the breasts of a client with fibrocystic breast disease? a) Nipple retraction b) Soft mass c) Skin dimpling d) Enlarged lymph nodes

b: The characteristic breast mass of fibrocystic disease is soft to firm, movable, and unlikely to cause nipple retraction. Nipple retraction, enlarged lymph nodes, and skin dimpling are more commonly associated with breast cancer

A female client who has a 2-month-old infant arrives at a health care facility reporting flu-like symptoms with fever and chills. When examining the breast, the nurse observes an increase in warmth. Which instruction should the nurse provide the client to help her cope with the condition? a) Avoid changing positions while nursing b) Increase fluid intake c) Avoid breastfeeding for 1 month d) Apply cold compresses to the affected breast

b: The nurse should instruct the client with mastitis to increase her fluid intake. A client with mastitis is instructed to continue breastfeeding as tolerated and to frequently change positions while nursing. The nurse should also instruct the client to apply warm, not cold, compresses to the affected breast area or to take a warm shower before breastfeeding

A 41-year-old female client arrives at a health care setting reporting dull nipple pain with a burning sensation, accompanied by pruritus around the nipple. The physician suspects mammary duct ectasia. Which order should the nurse question? a) Penicillin orally for 10 days b) Cool compresses to the affected area c) Acetaminophen as needed for discomfort d) Monitor temperature

b: The symptom of mammary duct ectasia include the presence of green, brown, straw-colored, reddish, gray, or cream-colored nipple discharge with a consistency of toothpaste. Treatment includes antibiotic therapy with penicillinase-resistant penicillin or cephalosporin, pain medication, and warm compresses to the inflamed area

A client is considering breast augmentation. Which of the following would the nurse recommend to the client to ensure that there are no malignancies? a) Mastopexy b) Mammogram c) Breast biopsy d) Ultrasound

b: When caring for a client considering breast augmentation, the nurse should provide her with a general guideline to have a mammogram to verify that there are no malignancies. Mastopexy involves a breast life for drooping breasts. Ultrasound or breast biopsy would not be necessary unless there was evidence of a problem

A nurse is educating a client on the technique for performing breast self-examination. Which instruction should the nurse include in the teaching plan with regard to the different degrees of pressure that need to be applied on the breast? a) Light pressure midway into the tissue b) Hard pressure applied down to the ribs c) Medium pressure on the skin throughout d) Medium pressure around the areolar area

b: When performing the breast self-examination, the nurse should instruct the client to apply hard pressure down to the ribs. Light, not medium, pressure should be applied when moving the skin without moving the tissue underneath. Medium, not light, pressure should be applied midway into the tissue. Client need not specifically palpate the areolar area during breast self-examination

Mammography is recommended for a client diagnosed with intraductal papilloma. Which factor should the nurse ensure when preparing the client for a mammography? a) Client has taken an aspirin before the testing. b) Client has not applied deodorant on the day of testing. c) Client is just going to start her menses. d) Client has not consumed fluids 1 hour before testing.

b: When preparing a client for mammography, the nurse should ensure the client has not applied deodorant or powder on the day of testing because these products can appear on the x-ray film as calcium spots. It is not necessary for the client to avoid fluid intake 1 hour prior to testing. Mammography has to be scheduled just after the client's menses to reduce chances of breast tenderness, not when the client is going to start her menses. The client can take aspirin or acetaminophen after the completion of the procedure to ease any discomfort, but these medications are not taken before mammography.

A client has undergone a mastectomy for breast cancer. Which instruction should the nurse include in the postsurgery client-teaching plan? a) Restrict intake of medication b) Elevate the affected arm on a pillow c) Breathe rapidly for an hour d) Avoid moving the affected arm in any way

b: When providing care to the client, the nurse should instruct the client to elevate the affected arm on a pillow. As part of the respiratory care, the nurse should instruct the client to turn, cough, and breathe deeply every 2 hours; rapid breathing is not encouraged. Active range-of-motion and arm exercises are necessary. To counter any pain experienced by the client, analgesics are administered as needed; intake of medication is not restricted

Which of the following is TRUE regarding the treatment of breast cancer? a) Women with hormone-sensitive cancers often have a poor prognosis and require many different types of treatment. b) Often, the first treatment option for breast cancer is surgery. c) Immunotherapy is seldom used as an adjunct therapy. d) Because breast cancer is often a systemic disease, chemotherapy is not effective

b: Women with hormone-sensitive cancers have a good prognosis and often need only hormonal treatment. Immunotherapy is often used as an adjunct to surgery. Chemotherapy is effective in treating systemic cancers

What is very good for the psychosocial effect, but can't be performed until all tissue has healed so that expanders can be put in?

breast reconstruction therapy

What plays a significant role in a woman's sexuality and self identity?

breasts

A nurse is assigned to educate a group of women on cancer awareness. Which risk factors for breast cancer are modifiable? Select all that apply. a) Previous abnormal breast biopsy b) Early menarche or late menopause c) Failing to breastfeed for up to a year after pregnancy d) Postmenopausal use of estrogen and progestins e) Not having children until after age 30

c, d, e: The modifiable risk factors for breast cancer are postmenopausal use of estrogen and progestins, not having children until after the age of 30, and failing to breastfeed for up to a year after pregnancy. Early menarche or late menopause and previous abnormal breast biopsy are the nonmodifiable risk factors for breast cancer

You are a community health nurse planning a program on breast cancer screening guidelines for women in the neighborhood. To best promote the participants' learning and adherence, you would include (select all that apply): a. a short audiotape on the BSE procedure b. a packet of articles from the medical literature c. written guidelines for mammography and CBE d. a discussion of the value of early breast cancer detection e. community resources where they can obtain an ultrasound and MRI

c, d.

A 29-year-old primiparous patient has a 3-week-old infant whom she is breastfeeding. The woman has sought care because of recent breast tenderness, redness, and fever. Which teaching point should the nurse prioritize when following up her care? a. encourage patient to continue breastfeeding her infant b. refer patient for a mammogram as quickly as possible c. ensure patient adheres to her prescribed antibiotic regimen d. teach patient to use warm compresses and educate her about self-limiting nature of illness

c.

A nurse is teaching a health promotion workshop to a group of women in their 40s and 50s. What information about nipple discharge should the nurse teach to participants? a. inappropriate lactation necessitates breast biopsy b. nipple discharge of any type is considered a precursor to cancer c. unexpected nipple discharge of any type warrants medical follow-up d. galactorrhea is a normal age-related change and a frequent perimenopausal symptom

c.

Preoperatively, to meet the psychologic needs of a woman scheduled for a modified radical mastectomy, you would: a. discuss the limitations of breast reconstruction b. include her significant other in all conversations c. promote an environment for expression of feelings d. explain the importance of regular follow-u screening

c.

The nurse is caring for a client with breast cancer who just underwent an axillary lymph node dissection. What intervention should the nurse use to decrease the lymphedema? a. keep the affected arm flat at the patient's side b. apply an elastic bandage on the affected arm c. assess blood pressure only on the unaffected arm d. restrict exercise of the affected arm for 1 week

c.

When doing breast self-examination, the female patient should report which findings to her physician? a. palpable rib margins b. denser breast tissue c. left nipple deviation d. different sized breasts

c.

The nurse is reviewing the medication list for a patient and notes that finasteride (Propecia) 1 mg daily is on the list. This drug is for which of these problems? a. Benign prostatic hypertrophy (BPH) b. Erectile dysfunction c. Alopecia in male patients d. Alopecia in male and female patients.

c. Alopecia in male patients

21. Which action will the nurse include in the plan of care for a patient with right arm lymphedema? a. Check blood pressure (BP) on both right and left arms. b. Avoid isometric exercise on the right arm. c. Assist with application of a compression sleeve. d. Keep the right arm at or below the level of the heart.

c. Assist with application of a compression sleeve.

2. During a well woman physical exam, a 43-year-old patient asks about her risk for breast cancer. Which question is most pertinent for the nurse to ask? a. Do you currently smoke tobacco? b. Have you ever had a breast injury? c. At what age did you start having menstrual periods? d. Is there a family history of fibrocystic breast changes?

c. At what age did you start having menstrual periods?

13. A patient newly diagnosed with stage I breast cancer is discussing treatment options with the nurse. Which statement by the patient indicates that additional teaching may be needed? a. There are several options that I can consider for treating the cancer. b. I will probably need radiation to the breast after having the surgery. c. Mastectomy is the best choice to decrease the chance of cancer recurrence. d. I can probably have reconstructive surgery at the same time as a mastectomy.

c. Mastectomy is the best choice to decrease the chance of cancer recurrence.

The nurse is performing an assessment of a patient who is asking for a prescription for sildenafil (Viagra). Which finding would be a contraindication to its use? a. 65 years of age b. History of thyroid disease c. Medication list that includes nitrates d. Medication list that includes saw palmetto

c. Medication list that includes nitrates

9. Which nursing action should be included in the plan of care for a patient returning to the surgical unit following a left modified radical mastectomy with dissection of axillary lymph nodes? a. Obtain a permanent breast prosthesis before the patient is discharged from the hospital. b. Teach the patient to use the ordered patient-controlled analgesia (PCA) every 10 minutes. c. Post a sign at the bedside warning against venipunctures or blood pressures in the left arm. d. Insist that the patient examine the surgical incision when the initial dressings are removed.

c. Post a sign at the bedside warning against venipunctures or blood pressures in the left arm.

5. A 53-year-old woman at menopause is discussing the use of hormone therapy (HT) with the nurse. Which information about the risk of breast cancer will the nurse provide? a. HT is a safe therapy for menopausal symptoms if there is no family history ofBRCA genes. b. HT does not appear to increase the risk for breast cancer unless there are other risk factors. c. The patient and her health care provider must weigh the benefits of HT against the risks of breast cancer. d. Natural herbs are as effective as estrogen in relieving symptoms without increasing the risk of breast cancer.

c. The patient and her health care provider must weigh the benefits of HT against the risks of breast cancer.

25. The nurse is caring for a 52-year-old patient with breast cancer who is receiving chemotherapy with doxorubicin (Adriamycin) and cyclophosphamide (Cytoxan). Which assessment finding is most important to communicate to the health care provider? a. The patient complains of fatigue. b. The patient eats only 25% of meals. c. The patients apical pulse is irregular. d. The patients white blood cell (WBC) count is 5000/L.

c. The patients apical pulse is irregular.

A patient is asking about the use of saw palmetto for prostate health. The nurse tells him that drugs that interact with saw palmetto include: a. acetaminophen (Tylenol). b. nitrates. c. nonsteroidal antiinflammatory drugs. d. antihypertensive drugs.

c. nonsteroidal antiinflammatory drugs.

8. After a 48-year-old patient has had a modified radical mastectomy, the pathology report identifies the tumor as an estrogen-receptor positive adenocarcinoma. The nurse will plan to teach the patient about a. estradiol (Estrace). b. raloxifene (Evista). c. tamoxifen (Nolvadex). d. trastuzumab (Herceptin).

c. tamoxifen (Nolvadex).

What is the most common breast mass in women? a) Mastitis b) Duct ectasia c) Fibroadenoma d) Intraductal papilloma

c: A fibroadenoma is a benign mass in the breast. It is the most common breast mass among women.

A nurse is caring for a female client undergoing radiation therapy after her breast surgery. The client is refusing to eat and states she does not have a desire to eat at this time. Which action should the nurse do first? a) Assess the client's BMI b) Notify the health care provider c) Continue to monitor the client d) Begin parenteral nutrition

c: A nurse would monitor for signs of anorexia as it is a likely side effect of radiation therapy, along with swelling and heaviness of the breast, local edema, inflammation, and sunburn-like skin changes. The nurse would continue to monitor the client since this is a common, expected side effect of radiation

The nurse is teaching a client with mastitis about care measures. Which client statement indicates effective teaching? a) "I should avoid pumping my breasts until the infection is cleared." b) "I need to wear a breast shield to keep the area moist." c) "I can use warm soaks to my breast to relieve the discomfort." d) "I can stop the antibiotic after 5 days."

c: Applying warm soaks to the breast or letting warm water from the shower flow over the breast can help to relieve some of the discomfort. Breast shields should be avoided because they trap breast milk and moisture around the nipple. The client needs to continue antibiotic therapy as prescribed for the entire treatment period, usually 10 days. The client can express milk with a breast pump until the infection resolves sufficiently to resume breast feeding

A client with breast cancer is scheduled to undergo chemotherapy with aromatase inhibitors. Which of the following best reflects the rationale for using this group of drugs? a) They block progesterone dependent tumors from growing b) They stimulate the immune system to attack a protein common in many tumors c) They lower the level of estrogen in the body blocking the tumor's ability to use it d) They attach to endogenous protein receptors to slow the growth of cancerous cells

c: Aromatase inhibitors lower the level of estrogen in the body thereby interfering with the ability of hormone-sensitive tumors to use estrogen for growth. Antiprogestin drug such as mifepristone, blocks progesterone-dependent breast cancers. The monoclonal antibody, trastuzumab attaches to protein receptors to slow the growth of cancer cells. A breast cancer vaccine is under investigation in Italy. This vaccine stimulates the immune system to attack a protein called mammoglobin A, which is found in 80% of breast cancer tumors

A client is scheduled to have a screening mammogram. When teaching the woman about this test, which instruction would be most appropriate for the nurse to include? a) "You can leave your necklace on, but you'll need to take off your watch and earrings." b) "You'll need to take a narcotic analgesic for any discomfort afterwards." c) "Don't use deodorant that day because it may interfere with the x-ray image." d) "Schedule the procedure just before your menses."

c: Deodorants and powder can appear on the x-ray film as calcium spots. The procedure should be scheduled just after menses, when breast tenderness is reduced. All jewelry must be removed from around the woman's neck because metal can cause distortions on the film image. Acetaminophen or aspirin can relieve any discomfort after the procedure

What is the most common type of breast cancer, accounting for approximately 85% of cases? a) Tubular carcinoma b) Medullary carcinoma c) Invasive ductal carcinoma d) Invasive lobular carcinoma

c: Invasive ductal carcinoma is the most common type of breast cancer. Invasive lobular carcinoma is the second most common type of breast cancer and accounts for approximately 10% to 15% of all cases

A nursing diagnosis of Risk for impaired tissue integrity would be most appropriate for which client? a) Client with a vaginal packing in place b) Client with endometriosis c) Client having reconstructive breast surgery d) Client taking oral contraceptives

c: Reconstructive breast surgery places the client at risk for insufficient blood supply to the muscle graft and skin, which can lead to tissue necrosis. Endometriosis or oral contraceptives aren't generally associated with altered tissue perfusion. Pressure from vaginal packing can sometimes put pressure on the bladder neck and interfere with voiding

A 62-year-old female client arrives at a health care facility reporting skin redness in the breast area, along with skin edema. The physician suspects inflammatory breast cancer. For which symptom of inflammatory breast cancer should the nurse assess? a) Palpable mobile cysts b) Induced nipple discharge c) Increased warmth of the breast d) Palpable papilloma

c: Skin edema, redness, and warmth of the breast are symptoms of inflammatory breast cancer. Induced discharge is an indication of benign breast conditions, which are noncancerous. Cancer involves spontaneous nipple discharge. Papillomas and palpable mobile cysts are characteristics of fibroadenomas, intraductal papilloma, and mammary duct ectasia, which are benign breast conditions and are noncancerous

After teaching a client with mastitis about caring for her breasts, the nurse determines that the client has understood the instructions when she states which of the following? a) " Until the infection is gone, I can't take a tub bath or shower." b) "I should avoid wearing a bra because it would increase the pain." c) "I will express milk with a breast pump until infection is resolved." d) "I need to apply cold soaks to the breast several times a day."

c: The client statement about expressing milk with a breast pump until the infection is resolved indicates effective teaching. The client should apply warm soaks to the breast or let warm water from a shower flow over the breast. The nurse should advise the client to wear a supportive brassiere. The client should bathe or shower regularly

The nurse is developing a presentation for a local women's health center about breast cancer. Which of the following would the nurse include as being the most common type of breast cancer? a) Inflammatory breast cancer b) Medullary carcinoma c) Invasive ductal carcinoma d) Infiltrating ductal carcinoma

c: The most common malignancy is invasive ductal carcinoma (85%), followed by infiltrating ductal carcinoma (75%), tubular carcinoma, colloid carcinoma, medullary, and inflammatory breast cancer, the rarest but most aggressive form of breast cancer

During a follow-up visit, a female client who underwent a mastectomy asks the nurse if she can work in her backyard or at least do some household work. Which suggestion would be most appropriate? a) Increase the frequency of follow-up visits if she does works b) Avoid working in the garden or yard altogether c) Wear gloves and protective clothing to avoid any injuries d) Avoid household chores for at least 6 to 9 months

c: The nurse should recommend that the client wear gloves when doing backyard work or housework to prevent injuries that may heal slowly or become infected. Working whether it be in the backyard or doing some household chores can be helpful in promoting feelings of usefulness, thereby enhancing the client's coping abilities and self-esteem. She could be advised to follow up more frequently however, this would not help prevent any untoward injury

The nurse is giving an educational event for a local women's group on self-examination of the breast. The nurse tells the women's group to raise their arms and inspect their breasts in a mirror. A member of the women's group asks the nurse why she needs to do this. What is the nurse's best response? a) "If you feel pain you will need to inspect it." b) "It will give you greater visibility." c) "It will help to observe for dimpling." d) "Everyone is different in assessing the breast."

c: The primary reason for raising the arms is to detect any dimpling. To elicit skin dimpling or retraction that may otherwise go undetected, the examiner instructs the patient to raise both arms overhead. This maneuver normally elevates both breasts equally. Pain does not always occur with breast masses. The assessment of the breast should be uniform.

A woman who is 6 months postpartum calls the clinic and reports flu-like symptoms, an elevated temperature, and pain and redness throughout her left breast. What would the nurse first suspect? a) Duct ectasia b) Inflammatory breast cancer c) Mastitis d) Intraductal papilloma

c: These symptoms most closely resemble mastitis. Mastitis usually occurs in the postpartum period while the woman is still breastfeeding. Mastitis is usually unilateral and is seen as a red, painful breast with elevated temperature and flu-like symptoms

While performing a clinical breast examination, the nurse notes a firm and rubbery nodule that is well circumscribed and moves freely. How should the nurse counsel the patient? a) "This is a normal breast finding, and you don't have to worry about it." b) "You may have breast cancer." c) "It's most likely a fibroadenoma, but we may need to do a biopsy."

c: This description most closely matches a fibroadenoma, but diagnostic imaging and even biopsy are warranted to confirm and rule out a cancerous tumor. The nurse should never tell the patient that she may have cancer because this will only cause anxiety

An OB/GYN nurse is teaching a new female client how to perform breast self-examination. Which of the following is a recommended step for this examination? a) Standing up, press gently in small circular motions around an imaginary clock face b) In the shower, place hands on hips, bend forward and examine breasts c) To examine the right breast, put a pillow or folded towel under the right shoulder d) Before a mirror, inspect the breasts with arms raised, and then extended sideways

c: To examine the right breast, the client should put a pillow or folded towel under the right shoulder and place the right hand behind the head. With the left hand, fingers flat, press gently in small circular motions around an imaginary clock face. Before a mirror, the client should inspect the breast with arms at side, then raise hands above the head and inspect the breasts

What diagnostic for breast cancer uses a 14 gauge needle, local anesthetic and a spring loaded device used for a more definitive diagnosis, gets actual tissue, best used for larger tumors close to the skin surface?

core biopsy

The nurse is caring for a group of breast cancer survivors post mastectomy. When developing a list of instructions of points to avoid, which point is highlighted? a) Applying cream to breast b) Arm exercises on affected side c) Wearing loose fitting shirts d) No lifting greater than 15 lb

d) No lifting greater than 15 lb Correct Explanation: Of the list citing things to avoid, the correct option to avoid is lifting more than 15 lb. This fact is important and needs to be highlighted. Tight-fitting or constrictive clothing is to be avoided. The client is encouraged to place cream on any area of the breast which is dry. Arm exercises are routinely completed on the affected arm.

A 51-year-old woman has recently had a unilateral, right total mastectomy and axillary node dissection for the treatment of breast cancer. What nursing intervention should the nurse include in the patient's care? a. immobilize the patient's right arm until post-op day 3 b. maintain the patient's right arm in a dependent position when at rest c. administer diuretics prophylactically for the prevention of lymphedema d. promote gradually increasing mobility as soon as possible following surgery

d.

A modified radical mastectomy has been scheduled for your patient with breast cancer. Postoperatively, to restore arm function on the affected side, you would: a. apply heating pads or blankets to increase circulation b. place daily ice packs to minimize the risk of lymphedema c. teach passive exercises with the affected arm in a dependent position d. emphasize regular exercises for the affected shoulder to increase range of motion

d.

In teaching a patient who wants to perform BSE, you inform her that the technique involves both the palpation of the breast tissue and: a. palpation of the cervical lymph nodes b. hard squeezing of the breast tissue c. a mammogram to evaluate breast tissue d. inspection of the breasts for any changes

d.

You are caring for a young woman who has painful fibrocystic breast changes. Management of this patient would include: a. scheduling a biopsy to rule out malignant changes b. teaching that symptoms will probably subside if she stops using oral contraceptives c. preparing her for surgical removal of the lumps, since they will become larger and more painful d. explaining that restrictions of coffee and chocolate and supplements of vitamin E may relieve some discomfort

d.

27. The outpatient clinic receives telephone calls from four patients. Which patient should the nurse call back first? a. 57-year-old with ductal ectasia who has sticky multicolored nipple discharge and severe nipple itching b. 21-year-old with a family history of breast cancer who wants to discuss genetic testing for the BRCA gene c. 40-year-old who still has left side chest and arm pain 2 months after a left modified radical mastectomy d. 50-year-old with stage 2 breast cancer who is receiving doxorubicin (Adriamycin) and has ankle swelling and fatigue

d. 50-year-old with stage 2 breast cancer who is receiving doxorubicin (Adriamycin) and has ankle swelling and fatigue

24. A patient has had left-sided lumpectomy (breast-conservation surgery) and an axillary lymph node dissection. Which nursing intervention is appropriate to delegate to a licensed practical/vocational nurse (LPN/LVN)? a. Teaching the patient how to avoid injury to the left arm b. Assessing the patients range of motion for the left arm c. Evaluating the patients understanding of instructions about drain care d. Administering an analgesic 30 minutes before scheduled arm exercises

d. Administering an analgesic 30 minutes before scheduled arm exercises

19. The nurse is admitting a patient scheduled this morning for lumpectomy and axillary lymph node dissection. Which action should the nurse take first? a. Teach the patient how to deep breathe and cough. b. Discuss options for postoperative pain management. c. Explain the postdischarge care of the axillary drains. d. Ask the patient to describe what she knows about the surgery.

d. Ask the patient to describe what she knows about the surgery.

20. When the nurse is working in the womens health care clinic, which action is appropriate to take? a. Teach a healthy 30-year-old about the need for an annual mammogram. b. Discuss scheduling an annual clinical breast examination with a 22-year-old. c. Explain to a 60-year-old that mammography frequency can be reduced to every 3 years. d. Teach a 28-year-old with a BRCA-1 mutation about magnetic resonance imaging (MRI).

d. Teach a 28-year-old with a BRCA-1 mutation about magnetic resonance imaging (MRI).

In which of the follow patients would the nurse first suspect cancer? a) A 30-year-old with a mobile, rubbery, firm, well-circumscribed, nontender lump b) A 25-year-old with multiple small, round, and smooth lesions on both breasts that are painful during menstruation c) A 40-year-old with nipple retraction and a watery discharge d) A 45-year-old with thickening in one breast with nipple irritation and retraction and a pink discharge

d: The 25-year-old most likely has fibrocystic breast changes. The 30-year-old most likely has a fibroadenoma. The 40-year-old most likely has an intraductal papilloma. Further assessment is needed to confirm each of these, but this is what the nurse would first suspect

1. The nurse teaching a young womens community service group about breast self-examination (BSE) will include that a. BSE will reduce the risk of dying from breast cancer. b. BSE should be done daily while taking a bath or shower. c. annual mammograms should be scheduled in addition to BSE. d. performing BSE after the menstrual period is more comfortable.

d. performing BSE after the menstrual period is more comfortable.

The nurse is teaching a patient about the possible adverse effect of priapism, which may occur when taking erectile dysfunction drugs. The nurse emphasizes that, if this occurs, the most important action is to a. stay in bed until the erection ceases. b. apply an ice pack for 30 minutes. c. turn toward his left side and rest. d. seek medical attention immediately.

d. seek medical attention immediately.

16. The nurse will anticipate teaching a 56-year-old patient who is diagnosed with lobular carcinoma in situ (LCIS) about a. lumpectomy. b. lymphatic mapping. c. MammaPrint testing. d. tamoxifen (Nolvadex).

d. tamoxifen (Nolvadex).

7. The nurse will teach a patient with metastatic breast cancer who has a new prescription for trastuzumab (Herceptin) that a. hot flashes may occur with the medication. b. serum electrolyte levels will be drawn monthly. c. the patient will need frequent eye examinations. d. the patient should call if she notices ankle swelling.

d. the patient should call if she notices ankle swelling.

What does the American Cancer Society (ACS) say about breast self-examination (BSE)? a) If a patient has mammography every year, she needs to perform a BSE only every 6 months. b) As long as a patient has a clinical breast examination every 1 to 3 years, a BSE is recommended only every 6 months. c) To decrease mortality from breast cancer, a BSE should be performed every month. d) A monthly BSE is optional.

d: According to the American Cancer Society, a monthly BSE is now optional because the most recent research demonstrates that monthly BSEs do not decrease the mortality associated with breast cancer

A female client with metastatic breast disease is receiving trastuzumab as part of her immunotherapy. The client has nausea, fatigue, diarrhea, appears jaundice, and has a distended abdomen. What would the nurse do next? a) Decrease the trastuzumab infusion rate b) Continue to monitor the client c) Assess the client's white blood cell count d) Notify the healthcare provider

d: Adverse effects of trastuzumab include cardiac toxicity, vascular thrombosis, hepatic failure, fever, chills, nausea, vomiting, and pain with first infusion. The nurse should monitor for these adverse effects with the first infusion of trastuzumab. The nurse would notify the health care provider since the client is showing signs of hepatic failure.

A breast tumor is most likely found in what part of the breast? a) At the areola b) Near the nipple c) The upper inner quadrant d) The upper outer quadrant

d: Although they can occur throughout the breast, breast tumors are most frequently located in the upper outer quadrant of the breast.

The nurse is teaching a client with mastitis about care measures. Which client statement indicates effective teaching? a) "I need to wear a breast shield to keep the area moist." b) "I should avoid pumping my breasts until the infection is cleared." c) "I can stop the antibiotic after 5 days." d) "I can use warm soaks to my breast to relieve the discomfort."

d: Applying warm soaks to the breast or letting warm water from the shower flow over the breast can help to relieve some of the discomfort. Breast shields should be avoided because they trap breast milk and moisture around the nipple. The client needs to continue antibiotic therapy as prescribed for the entire treatment period, usually 10 days. The client can express milk with a breast pump until the infection resolves sufficiently to resume breast feeding

A client arrives at the health care facility reporting a lump that she felt during her breast self-examination. Upon diagnosis, the physician suspects fibroadenomas. Which question should the nurse ask when assessing the client? a) "Do you consume foods high in fat?" b) "Do you smoke regularly?" c) "Are you taking oral contraceptives?" d) "Are you lactating?"

d: As the physician suspects fibroadenomas, it is important for the nurse to know whether the client is pregnant or lactating since the incidence of fibroadenomas is more frequent among pregnant and lactating women. Taking oral contraceptives assists a client with fibrocystic breast changes, but is not necessary for a client with fibroadenomas. Fibroadenomas usually occur in women between 20 and 30 years of age. Smoking and a high-fat diet will make the client more susceptible to cancer, not fibroadenomas.

The nurse is preparing a client for breast biopsy of a lump. The nurse explains that a needle with a cutting edge will be inserted into a lump and rotated to remove a core sample. What type of breast biopsy is this client undergoing? a) Aspiration b) Incisional biopsy c) Excisional biopsy d) Needle biopsy

d: During a needle biopsy, a needle with a cutting edge will be inserted into a lump and rotated to remove a core sample. With aspiration, cells from a lump are drawn into a syringe. In excisional biopsy, an entire lump is removed and analyzed, and, in incisional biopsy, part of a lump is removed as a sample

How should the nurse counsel a postpartum patient on how to prevent mastitis? a) "Be sure to keep your breasts covered when you are not feeding or pumping." b) "Sterilize your bottles and pump equipment after each use." c) "If you notice that your breast is warm, hard, or red, stop feeding on that side and pump from that breast instead." d) "Wash your hands thoroughly, and let your breasts dry after each feeding."

d: Handwashing is one of the best ways to prevent infection. If the woman feels that her breast is warm, hard, or red, she should increase the amount of breastfeeding from that side. It is not necessary to sterilize bottles and pumping equipment after each use. Normal dish washing is sufficient. Keeping the breasts exposed to the air to dry will aid in preventing infection

The nurse is providing care to a client who has had surgery as treatment for breast cancer. The nurse would be alert for the development of which of the following? a) Fibroadenoma b) Breast abscess c) Fibrocystic breast disease d) Lymphedema

d: Lymphedema occurs in some women after breast cancer surgery. It causes disfigurement and increases the lifetime potential for infection and poor healing. Fibrocystic breast disease and fibroadenoma are two benign breast conditions that occur usually in premenopausal woman. Breast abscess is the infectious and inflammatory breast condition that is common among breast-feeding mothers

A nurse is preparing a presentation for a health fair about preventing breast cancer. Which of the following would the nurse include? a) Opting for estrogen only replacement therapy during menopause b) Increasing the intake of caffeinated drinks c) Delaying childbearing until after age 30 d) Maintaining an ideal weight

d: Maintaining an ideal weight decreases the risk of breast cancer. Having no children or having children after age 30 is associated with an increased risk for breast cancer. Some breast tumors are hormone dependent, such that estrogen (or progesterone) enhances tumor growth. Women are advised to avoid the consumption of alcohol, not caffeine, because alcohol correlates with an increased risk of breast cancer

The nurse explains to a client with a family history of breast cancer the difference between benign and malignant neoplasms. Which of the following is a characteristic of a malignant neoplasm? a) Round or oval shape b) Movable c) Smooth border d) Irregular shape and hard

d: Malignant lesions are more likely to be irregularly shaped and hard, and often show secondary signs, such as enlarged lymph nodes in the axillary area, breast asymmetry, nipple retraction, bloody discharge, dimpling, or elevation of one breast. Benign lesions tend to be round or oval with a smooth border and usually show no secondary signs. Furthermore, benign lesions are likely to be movable

Which breast disorder is NOT benign? a) Fibroadenomas b) Fibrocystic breasts c) Duct ectasia d) Paget's disease

d: Paget's disease of the nipple is a rare cancer of the nipple and surrounding skin. Fibrocystic breasts, fibroadenomas, and duct ectasia are all benign conditions of the breast

During a recent visit to the clinic a woman presents with erythema of the nipple and areola on the right breast. She states this started several weeks ago and she was fearful of what would be found. The nurse suspects a) Acute mastitis b) Peau d'orange c) Nipple inversion d) Paget's disease

d: Paget's disease presents with erythema of the nipple and areola. Peau d'orange is associated with breast cancer and is caused by interference with lymphatic drainage. Nipple inversion is considered normal if long-standing; if it is associated with fibrosis and is a recent development, malignancy is suspected. Acute mastitis is associated with lactation but may occur at any age

When counseling a patient on how to prepare for her first mammogram, what should the nurse include? a) "Prepare yourself for a 30- to 45-minute procedure." b) "Schedule the test during or just before your period." c) "Tylenol or Motrin won't help with the discomfort from the procedure." d) "Don't wear any powder, deodorant, or jewelery."

d: Powder, deodorant, and jewelery can distort the images on film and should not be worn. The procedure should be scheduled just after the menses, when the breasts are the least tender. Pain relievers such as acetaminophen and ibuprofen may ease discomfort during the procedure and thereafter. Mammograms are 5- to 10-minute procedures

What are nursing responsibilities for the post op breast cancer patient?

teach-elevate affected side with distal joint higher than proximal joint, NO BP, injections for venipunctures on affected side, watch for S & S of edems on affected arm (may occur post or years later), limited arm exercises 24 hrs post op, adduction and external rotation exercises after wound has healed

A female client is diagnosed with breast abscess. Although she has been allowed to breast-feed her newborn, she decides to terminate breast-feeding. Which of the following would be most appropriate in this situation? a) Encourage the client to include protein content in the diet b) Reduce the frequency of removing and reapplying the dressings c) Instruct the client to wear a loose-fitting bra d) Assist the client to pump the breasts to remove breast milk

d: The nurse should help the client pump the breasts and remove breast milk to prevent engorgement. Because the client has decided to terminate breast-feeding, the client should wear a tight-fitting bra. Including protein content in the diet would be unrelated to the client's current situation

A female client who has a 2-month-old infant arrives at a health care facility reporting flu-like symptoms with fever and chills. When examining the breast, the nurse observes an increase in warmth. Which instruction should the nurse provide the client to help her cope with the condition? a) Avoid changing positions while nursing b) Apply cold compresses to the affected breast c) Avoid breastfeeding for 1 month d) Increase fluid intake

d: The nurse should instruct the client with mastitis to increase her fluid intake. A client with mastitis is instructed to continue breastfeeding as tolerated and to frequently change positions while nursing. The nurse should also instruct the client to apply warm, not cold, compresses to the affected breast area or to take a warm shower before breastfeeding

After teaching a group of students about the signs and symptoms of breast cancer, the instructor determines that additional teaching is needed when the group identifies which of the following? a) Nipple retraction b) Painless mass c) Peau d'orange skin d) Breast symmetry

d: The primary sign of breast cancer is a painless mass in the breast. Other signs of breast cancer include a bloody discharge from the nipple, a dimpling of the skin over the lesion, retraction of the nipple, peau d'orange (orange peel) appearance of the skin, and a difference in size between the breasts.

While interviewing a woman before her yearly examination, the nurse learns that she stopped breastfeeding over a year ago but occasionally has pain on one side behind her left nipple not related to her period. Upon examination, the nurse does not palpate any lumps but finds the left nipple retracted with a greenish discharge. What would the nurse first suspect? a) Inflammatory breast cancer b) Paget's disease c) Mastitis d) Duct ectasia

d: These symptoms most closely resemble duct ectasia, which is unilateral, is seen at or near the nipple, and often has a greenish discharge. Cancer cannot be entirely ruled out, but the nurse's first thought should be duct ectasia

A client reports lumpy, tender breasts, particularly during the week before menses. She reports pain that often dissipates after the onset of menses. The nurse suspects the client has fibrocystic breast changes. Which should the nurse do next? a) Have the client follow up in 1 week b) Schedule the client for cryoablation c) Determine if the client has had a mammography d) Perform a breast examination

d: To determine if the client is experiencing fibrocystic breast changes, the nurse must first examine the client's breasts. It is not important to know if the client has a mammography at this time. Cryoabation is done to remove a tumor

A client with breast cancer is to receive immunotherapy. The nurse would expect the client to receive which of the following? a) Raloxifene b) Tamoxifen c) Anastrozole d) Trastuzumab

d: Trastuzumab is the first monoclonal antibody approved for breast cancer treatment. Tamoxifen is a selective estrogen receptor modulator used to prevent further spread of breast cancer in women with ER-positive breast cancer. Raloxifene is a selective estrogen receptor modulator used to prevent further spread of breast cancer; it was originally used solely for prevention and treatment of osteoporosis. Anastrozole is an aromatase inhibitor that interferes with the conversion of androgens to estrogens; it is usually given to women with advanced breast cancer or cancers that recur despite the use of tamoxifen

What are pre op nursing diagnoses for the patient undergoing breast cancer surgery?

deficient knowledge, anxiety, fear, risk for ineffective coping, decisional conflict

What does the sentinel node biopsy do to the breast area?

disfigures it

What is the goal of breast conservation treatment?

excise tumor

What diagnostic for breast cancer is used to remove a portion of the mass to be put through studies and a pathological assessment on a palpable mass, the entire mass pluse a 1 cm margin around it will be removed?

excisional biopsy (AKA lumpectomy)

A nurse is teaching a client who suspects that she has a lump in her breast. The nurse instructs the client that a diagnosis of breast cancer is confirmed by: a) mammography. b) breast self-examination. c) fine needle aspiration. d) chest X-ray.

fine needle aspiration. Correct Explanation: Fine needle aspiration and biopsy provide cells for histologic examination to confirm a diagnosis of cancer. A breast self-examination, if done regularly, is the most reliable method for detecting breast lumps early. Mammography is used to detect tumors that are too small to palpate. Chest X-rays can be used to pinpoint rib metastasis.

What diagnostic for breast cancer is well tolerated, uses local anesthesia and a 20-25 gauge needle to remove serum?

fine needle biopsy

What is the most commonly encountered breast condition in the male?

gynecomastia

What is male breast gland enlargement of glandular tissues immediately below and surrounding the nipple?

gynecomasty

What is the best way to know your teaching for BSE has been effective?

have the patient demonstrate

What should be included in a health history for breast disorders?

history of disease, gynecological and obstetric history, present meds, past and present hormonal therapy, contraceptions used, fertility treatments

A nurse is instructing a premenopausal woman about breast self-examination. The nurse should tell the client to do her self-examination: a) on the same day each month. b) immediately after her menses. c) on the first day of the menstrual cycle. d) at the end of her menstrual cycle.

immediately after her menses. Premenopausal women should do their self-examination immediately after the menstrual period, when the breasts are least tender and least lumpy. On the first and last days of the cycle, the woman's breasts are still very tender. Postmenopausal women, because their bodies lack fluctuation of hormone levels, should select one particular day of the month to do breast self-examination.

When is the best time for BSE?

in shower, 5-7 days after menses (can have fibroids or calcium deposits that are more prominent during menses)

What is it when a portion of the mass is removed and put through studies?

incisional biopsy

Why should a patient inspect their breasts in a mirror?

it will help to observe for dimpling

What should the nurse assess post op breast cancer surgery?

maintain pressure dressing, assess wound drain, encourage woman to look at the incision, information about chemo if needed, info about radiation (may cause skin changes) if needed, psychological concerns (altered body image, altered sexuality, fear of disease outcome)

What helps to show abnormalities in breasts and detects non-palpable nodules?

mammogram

What should be done to the dressing post op?

mark for drainage

What is used to treat invasive cancer by removing the entire breast tissue including the nipple and a portion of the lymph nodes?

modified radical mastectomy

How many people are affected by breast cancer?

more than 125,000 and 1450 men with 40,000 deaths annually

Your patient is receiving adjuvant chemotherapy for breast cancer. Which of the following is most likely her node status and tumor size? a) node negative, tumor size 0.3 cm b) node negative, tumor size 1.2 cm c) node negative, tumor size 0.2 cm d) node negative, tumor size 0.5 cm

node negative, tumor size 1.2 cm Explanation: Adjuvant chemotherapy is recommended for patients who have positive lymph nodes or who have invasive tumors greater that 1 cm in size, regardless of nodal status.

What is the #1 risk factor for men with breast cancer?

obesity

What are nursing diagnosis for a post op breast cancer patient?

pain, disturbed sensory perception, disturbed body image, self care deficit, risk for sexual dysfunction, deficient knowledge

What should be included in the assessment of a patient undergoing breast cancer surgery?

physical assessment, psychological assessment, pain assessment, teaching needs, post surgery instructions (know what to look for and report-bleeding, infection signs)

What can cause a great deal of anxiety or loss of sexual attraction to a spouse concerning the breast?

potential disfigurement

Why should a post op breast cancer patient not put their arms over their head?

pulls tissue

What may be used before breast cancer surgery to shrink the cancer so there's less tissue to be removed?

radiation

What is the nonsurgical management of breast cancer?

radiation, chemo, hormonal therapy, targeted therapy

What is the 1st lymph node involved in breast cancer?

sentinel node

On discharge, a client who underwent left modified radical mastectomy expresses relief that "the cancer" has been treated. When discussing this issue with the client, the nurse should stress that she: a) should schedule a follow-up appointment in 6 months. b) is lucky that the cancer was caught in time. c) should continue to perform breast self-examination on her right breast. d) will have irregular menses.

should continue to perform breast self-examination on her right breast. Having breast cancer on her left side puts the client more at risk for cancer on the opposite side and chest wall. Therefore, the nurse should stress the importance of monthly breast self-examinations and annual mammograms. Although the tumor was found, it was large enough to require a mastectomy, and could put the patient at risk for metastasis. Follow-up appointments should be monthly for the first few months and then scheduled at the direction of her health care provider. Modified radical mastectomy shouldn't affect the menstrual cycle.

What is used either on specific areas or a certain cycle of the cell cycle of the chemo regimen?

targeted therapy

Why is it important to check under the arms during a BSE?

that's where lymph nodes are located

What distinguishes fluid filled cysts from cancer and other lesions?

ultransonography

A client is being discharged to home following a modified radical mastectomy. The nurse is providing discharge instructions and making arrangements for home care. Which of the following interventions will be included in her instructions? Select all that apply. a) Advise the client that blood pressure measurements, injections, blood donations, and IV infusions are lifelong restrictions on the side of the mastectomy. b) Encourage the client to wear restrictive clothing on the affected side. c) Recommend wearing gloves while doing yard or housework. d) Advise the client to use a disposable razor for shaving axillary hair.

• Advise the client that blood pressure measurements, injections, blood donations, and IV infusions are lifelong restrictions on the side of the mastectomy. • Recommend wearing gloves while doing yard or housework. Explanation: Because of impaired lymphatic flow, blood pressure measurements, injections, blood donations, and IV infusions in the affected arm are contraindicated. Gloves are worn to prevent injuries that may heal slowly or become infected. Advise the use of an electric razor for shaving axillary hair because it reduces the likelihood of cutting the skin, leading to infection. Discourage sleeping on the affected arm or wearing constrictive clothing that impairs circulation.


Kaugnay na mga set ng pag-aaral

male and female reproductive system quizzes

View Set

Fed Goc chapter 9,10,11,12, Government: Chapters 6-11, Government - Chapter 11 Quiz

View Set

3.4 Given a scenario, install and configure wireless security settings.

View Set

Concept Physic Unit 1 Chapter 2 - Linear Motion

View Set

History Chapter 10 Essay questions

View Set

Psychology Chapter 7 In Class Test - Stephen Kittrell

View Set